Download as pdf or txt
Download as pdf or txt
You are on page 1of 166

Priscilla’s Medicine

Table of Contents
SHORT CASES IN MEDICINE ...........................
..........................................
.............................
............................
............................
............................
............................
............................
............................
............................
................
.. 5
MEDICINE = CARDIO SHORTS.............................................................
...........................................................................................
.............................................................
..............................................................
......................................................
....................... 5
MEDICINE = RESPI SHORTS ...........................................................
.........................................................................................
.............................................................
..............................................................
...........................................................
............................ 9
MEDICINE = RENAL SHORTS ..............................................................
............................................................................................
.............................................................
..............................................................
....................................................
.....................14
MEDICINE = ENDOCRINE SHORTS ..........................................................
.........................................................................................
.............................................................
.............................................................
...............................................
................15
MEDICINE = HANDS SHORTS ..............................................................
............................................................................................
.............................................................
..............................................................
....................................................
.....................16
ENDOCRINE ...........................
.........................................
............................
............................
............................
.............................
.............................
............................
............................
............................
............................
............................
................
.. 17
MEDICINE (THYROID) = PHYSICAL EXAMINATION ............................................................
..........................................................................................
.............................................................
...............................................
................17
MEDICINE (THYROID) = INTRODUCTION ............................................................
...........................................................................................
.............................................................
..............................................................
................................20
MEDICINE (THYROID) = THYROID LUMPS ..........................................................
.........................................................................................
.............................................................
..............................................................
................................23
MEDICINE (THYROID) = MANAGEMENT OF HYPERTHYROIDISM .............................................................
............................................................................................
....................................................
.....................23
SURGERY (THYROID) = HYPERTHYROIDISM ............................................................
...........................................................................................
.............................................................
.........................................................
...........................26
MEDICINE (THYROID) = HYPOTHYROIDISM ............................................................
...........................................................................................
.............................................................
.........................................................
...........................30
SURGICAL (THYROID) = THYROID CARCINOMA ...........................................................
..........................................................................................
..............................................................
....................................................
.....................33
SURGICAL (THYROID) = THYROIDECTOMY .........................................................
........................................................................................
.............................................................
..............................................................
................................36
RHEUMATOLOGY ............................
..........................................
............................
............................
............................
............................
............................
............................
............................
............................
.............................
...................
.... 38
MEDICINE (RHEUMATOLOGY) = RHEUMATOID ARTHRITIS SONG ...........................................................
..........................................................................................
....................................................
.....................38
MEDICINE (RHEUMATOLOGY) = GENERAL POINTS ABOUT ARTHRITIS .............................................................
............................................................................................
..........................................
...........39
MEDICINE (RHEUMATOLOGY) = SYSTEMIC LUPUS ERYTHEMATOSUS (SLE)........................................................
........................................................................................
.....................................
.....40
MEDICINE (RHEUMATOLOGY) = GALS SCREEN ...........................................................
..........................................................................................
..............................................................
....................................................
.....................46
MEDICINE (RHEUMATOLOGY) = RHEUMATOID ARTHRITIS ............................................................
..........................................................................................
..............................................................
................................48
MEDICINE (RHEUMATOLOGY) = EXAMINATION OF RHEUMATOID HANDS ............................................................
............................................................................................
.....................................
.....56
MEDICINE (RHEUMATOLOGY) = CASE STUDY .........................................................
........................................................................................
..............................................................
.........................................................
..........................59
MEDICINE (RHEUMATOLOGY) = CLERKING OF RHEUMATOID ARTHRITIS ............................................................
............................................................................................
.....................................
.....62
MEDICINE (RHEUMATOLOGY) = SCLERODERMA LONG CASE .........................................................
.......................................................................................
..............................................................
................................63
MEDICINE (RHEUMATOLOGY) = GOUT ...........................................................
..........................................................................................
.............................................................
..............................................................
.....................................
.....66
MEDICINE (RHEUMATOLOGY) = GOUT HISTORY TAKING ..........................................................
........................................................................................
..............................................................
.....................................
.....68
MEDICINE (RHEUMATOLOGY) = CHRONIC TOPHACEOUS GOUT (SHORT CASE) .........................................................
.........................................................................................
................................71
DIABETES ..........................
........................................
............................
.............................
.............................
............................
............................
............................
............................
............................
............................
............................
.....................
.......72
MEDICINE (DIABETES) = HISTORY TAKING .............................................................
............................................................................................
.............................................................
.........................................................
...........................72
MEDICINE (DIABETES) = DIETARY ADVICE .............................................................
............................................................................................
.............................................................
.........................................................
...........................74
MEDICINE (DIABETES) = COUNSELING A NEWLY DIAGNOSED DIABETIC ..........................................................
.........................................................................................
..........................................
...........75
MEDICINE (DIABETES) = DIABETES MANIFESTATIONS .............................................................
...........................................................................................
..............................................................
.....................................
.....76
MEDICINE (DIABETES) = DIABETES MELLITUS ...........................................................
..........................................................................................
..............................................................
....................................................
.....................77
MEDICINE (DIABETES) = HYPOGLYCEMIA ..........................................................
.........................................................................................
.............................................................
..............................................................
................................79
MEDICINE (DIABETES) = DIAGNOSIS OF DM ..........................................................
.........................................................................................
.............................................................
.........................................................
...........................81
RENAL MEDICINE ............................
..........................................
............................
............................
............................
............................
............................
............................
............................
............................
.............................
...................
.... 83
MEDICINE (RENAL) = NEPHROTIC SYNDROME HISTORY TAKING ...........................................................
..........................................................................................
....................................................
.....................83
MEDICINE (RENAL) = NEPHROTIC SYNDROME ............................................................
...........................................................................................
..............................................................
....................................................
.....................85
MEDICINE (RENAL) = SECONDARY HYPERTENSION ..............................................................
............................................................................................
.............................................................
..........................................
...........89
MEDICINE (RENAL) = DIALYSIS MODALITIES ..........................................................
.........................................................................................
.............................................................
.........................................................
...........................90
MEDICINE (RENAL) = RENAL TRANSPLANT (MAJOR RISKS) .........................................................
.......................................................................................
..............................................................
................................92
MEDICINE (RENAL) = ADULT POLYCYSTIC KIDNEY DISEASE (APKD) ............................................................
...........................................................................................
..........................................
...........92
1
MEDICINE (RENAL) = URINARY TRACT INFECTION / PYELONEPHRITIS ..........................................................
.........................................................................................
..........................................
...........94
MEDICINE (RENAL) = ASSESSING VOLUME STATUS ..........................................................
........................................................................................
.............................................................
...............................................
................96
MEDICINE (RENAL) = FLUID AND ELECTROLYTES (ACID- BASE DISORDERS) ...........................................................
...........................................................................................
................................98
MEDICINE (RENAL) = RESPIRATORY DISORDERS.............................................................
............................................................................................
..............................................................
............................................
............. 102
MEDICINE (RENAL) = RENAL TUBULAR ACIDOSIS (RTA) .............................................................
...........................................................................................
...........................................................
.............................103
MEDICINE (RENAL) = POTASSIUM DISORDERS ............................................................
...........................................................................................
..............................................................
.................................................
.................. 105
MEDICINE (RENAL) = HYPONATRAEMIA ............................................................
...........................................................................................
.............................................................
...........................................................
............................. 108
MEDICINE (RENAL) = HYPERNATRAEMIA ..........................................................
.........................................................................................
.............................................................
...........................................................
............................. 111
GASTROLOGY..........................
........................................
............................
.............................
.............................
............................
............................
............................
............................
............................
............................
.........................
........... 113
MEDICINE (GIT) = HISTORY TAKING: GIT (GENERAL) .............................................................
...........................................................................................
..............................................................
.................................... 113
MEDICINE (GIT) = PHYSICAL EXAMINATION : GIT ............................................................
..........................................................................................
.............................................................
............................................
............. 115
MEDICINE (GIT) = ISSUES FOR DISCUSSION ............................................................
...........................................................................................
.............................................................
......................................................
........................ 119
MEDICINE (GIT) = APPROACH TO ASCITES .............................................................
............................................................................................
.............................................................
......................................................
........................ 122
MEDICINE (GIT) = ASCITES..........................................................
........................................................................................
.............................................................
..............................................................
......................................................
....................... 124
MEDICINE (GIT) = CHRONIC LIVER DISEASE AND LIVER CIRRHOSIS ............................................................
...........................................................................................
............................................
............. 128
MEDICINE (GIT) = HEPATOMEGALY ..........................................................
.........................................................................................
.............................................................
.............................................................
.......................................
........ 132
MEDICINE (GIT) = JAUNDICE (HISTORY-TAKING)............................................................
...........................................................................................
..............................................................
............................................
............. 135
MEDICINE (GIT) = APPROACH TO JAUNDICE ..........................................................
.........................................................................................
.............................................................
......................................................
........................ 137
MEDICINE (GIT) = ACUTE HEPATITIS ............................................................
...........................................................................................
.............................................................
..............................................................
.................................... 143
MEDICINE (GIT) = VIRAL HEPATITIS .............................................................
............................................................................................
.............................................................
..............................................................
.................................... 144
MEDICINE (GIT) = ALCOHOLIC LIVER DISEASE ...........................................................
..........................................................................................
..............................................................
.................................................
.................. 152
MEDICINE (GIT) = AUTOIMMUNE HEPATITIS.........................................................
........................................................................................
..............................................................
......................................................
....................... 153
MEDICINE (GIT) = METABOLIC LIVER DISEASE ..........................................................
.........................................................................................
..............................................................
.................................................
.................. 154
MEDICINE (GIT) = WILSON’S DISEASE (HEPATOLENTICULAR DISORDER) ..........................................................
..........................................................................................
.................................... 156
SURGERY (GIT) = OBSTRUCTIVE JAUNDICE .............................................................
............................................................................................
.............................................................
......................................................
........................ 158
MEDICINE (GIT) = LIVER FAILURE ............................................................
...........................................................................................
.............................................................
.............................................................
.......................................
........ 163
MEDICINE (GIT) = PORTAL HYPERTENSION ...........................................................
..........................................................................................
.............................................................
......................................................
........................ 165
MEDICINE (GIT) = CHRONIC DIARRHEA .............................................................
............................................................................................
.............................................................
...........................................................
............................. 167
MEDICINE (GIT) = INFLAMMATORY BOWEL DISEASE ..........................................................
........................................................................................
.............................................................
.......................................
........ 171
REPIRATORY MEDICINE ..........................
.........................................
.............................
............................
............................
............................
............................
............................
............................
............................
..................
.... 179
MEDICINE (RESPI) = HISTORY TAKING: RESPIRATORY SYSTEM (GENERAL) ...........................................................
........................................................................................
.............................179
MEDICINE (RESPI) = PHYSICAL EXAMINATION: RESPIRATORY SYSTEM ..........................................................
.........................................................................................
.......................................
........ 182
MEDICINE (RESPI) = HAEMOPTYSIS ..........................................................
.........................................................................................
.............................................................
.............................................................
.......................................
........ 188
MEDICINE (RESPI) = DYSPNOEA ............................................................
...........................................................................................
.............................................................
.............................................................
............................................
............. 189
MEDICINE (RESPI) = APPROACH TO CHEST PAIN AND DYSPNEA ............................................................
...........................................................................................
.................................................
.................. 191
MEDICINE (RESPI) = PULMONARY FIBROSIS ...........................................................
..........................................................................................
.............................................................
......................................................
........................ 193
MEDICINE (RESPI) = COPD ..............................................................
............................................................................................
.............................................................
..............................................................
.................................................
.................. 194
MEDICINE (RESPI) = BRONCHIECTASIS ..........................................................
.........................................................................................
.............................................................
..............................................................
.................................... 199
MEDICINE (RESPI) = COR PULMONALE ..........................................................
.........................................................................................
.............................................................
..............................................................
.................................... 203
MEDICINE (RESPI) = RESPIRATORY INFECTIONS: TUBERCULOSIS ..........................................................
.........................................................................................
.................................................
.................. 205
MEDICINE (RESPI) = PANCOAST TUMOUR-UPPER LOBE LUNG CA..........................................................
.........................................................................................
.................................................
.................. 211
MEDICINE (RESPI) = PLEURAL EFFUSION ...........................................................
..........................................................................................
.............................................................
...........................................................
............................. 212
MEDICINE (RESPI) =PNEUMOTHORAX ...........................................................
..........................................................................................
.............................................................
..............................................................
.................................... 216
MEDICINE (RESPI) = RESPIRATORY FAILURE .........................................................
........................................................................................
..............................................................
......................................................
....................... 220
MEDICINE (RESPI) = SYSTEMIC APPROACH TO CXR ..............................................................
............................................................................................
.............................................................
.......................................
........ 222
MEDICINE (RESPI) = MEDIASTINAL MASSES ...........................................................
..........................................................................................
.............................................................
......................................................
........................ 223
CARDIO VASCULAR SYSTEM ..........................
.........................................
.............................
............................
............................
............................
............................
............................
............................
.........................
........... 224
MEDICINE (CVS) = HISTORY TAKING: CVS...................................................
CVS..................................................................................
.............................................................
..............................................................
.................................... 224
MEDICINE (CVS) = PHYSICAL EXAMINATION: CVS ..........................................................
........................................................................................
.............................................................
............................................
............. 227
MEDICINE (CVS) = ISSUES FOR DISCUSSION ............................................................
...........................................................................................
.............................................................
......................................................
........................ 231
MEDICINE (CVS) = APPROACH TO CHEST PAIN ...........................................................
..........................................................................................
..............................................................
.................................................
.................. 236
MEDICINE (CVS) = HO ON CALL ............................................................
...........................................................................................
.............................................................
.............................................................
............................................
............. 240
2
MEDICINE (CVS) = ISCHAEMIC HEART DISEASE (HISTORY) .........................................................
.......................................................................................
...........................................................
.............................242
MEDICINE (CVS) = ANGINA PECTORIS ...........................................................
..........................................................................................
.............................................................
..............................................................
.................................... 244
MEDICINE (CVS) = ISCHAEMIC HEART DISEASE (HISTORY) .........................................................
.......................................................................................
...........................................................
.............................248
MEDICINE (CVS) = ACUTE CORONARY SYNDROME (ACS) ............................................................
..........................................................................................
...........................................................
.............................250
MEDICINE (CVS) = CONGESTIVE CARDIAC FAILURE (CCF)...........................................................
.........................................................................................
...........................................................
............................. 257
MEDICINE (CVS) = PATHOPHYSIOLOGY OF DYSPNOEA ..............................................................
............................................................................................
..............................................................
.................................... 264
MEDICINE (CVS) = PROGNOSTIC FACTORS OF HYPERTENSION .........................................................
.......................................................................................
......................................................
........................264
MEDICINE (CVS) = HYPERTENSION ...........................................................
..........................................................................................
.............................................................
.............................................................
.......................................
........ 266
MEDICINE (CVS) = ANTI HYPERTENSIVE MEDICATION .............................................................
...........................................................................................
..............................................................
.................................... 273
MEDICINE (CVS) = GUIDELINES FOR SELECTING DRUG TREATMENT OF HYPERTENSION ............................................................ ....................................................................
........277
MEDICINE (CVS) = LIPIDS ............................................................
..........................................................................................
.............................................................
.............................................................
......................................................
........................ 278
MEDICINE (CVS) = MYOCARDITIS..............................................................
.............................................................................................
.............................................................
.............................................................
.......................................
........ 282
MEDICINE (CVS) = CARDIOMYOPATHY ..........................................................
.........................................................................................
.............................................................
..............................................................
.................................... 284
MEDICINE (CVS) = TAKAYASU ARTERITIS .........................................................
........................................................................................
.............................................................
...........................................................
............................. 286
MEDICINE (CVS) = VALVULAR HEART DISEASE ...........................................................
..........................................................................................
..............................................................
.................................................
.................. 287
MEDICINE (CVS) = VALVULAR HEART DISEASE ...........................................................
..........................................................................................
..............................................................
.................................................
.................. 289
MEDICINE (CVS) = PROSTHETIC HEART VALVES.........................................................
........................................................................................
..............................................................
.................................................
.................. 294
MEDICINE (CVS) = INFECTIVE ENDOCARTITIS ............................................................
...........................................................................................
..............................................................
.................................................
.................. 296
PRISCILLA’S MEDICINE ADD-ON ............................
..........................................
............................
............................
............................
............................
............................
............................
.............................
................. 300
SURGERY (THYROID) = INVESTIGATIONS ............................................................
...........................................................................................
.............................................................
...........................................................
............................. 300
SURGERY (THYROID) = SHORT CASES ............................................................
...........................................................................................
.............................................................
..............................................................
.................................... 302
SURGERY (THYROID) = CONGENITAL ANOMALIES............................................................
...........................................................................................
..............................................................
............................................
............. 303
MEDICINE (RHEUMATOLOGY) = APPROACH TO THE RHEUMATOLOGICAL CASE ...........................................................
...................................................................................
........................304
MEDICINE (RHEUMATOLOGY) = DERMATOMYOSITIS AND POLYMYOSITIS ...........................................................
...........................................................................................
.................................... 309
MEDICINE (RHEUMATOLOGY) = HISTORY-TAKING ...........................................................
.........................................................................................
.............................................................
............................................
............. 313
MEDICINE (RHEUMATOLOGY) = HAND ..........................................................
.........................................................................................
.............................................................
..............................................................
.................................... 315
MEDICINE (RHEUMATOLOGY) = HANDS & WRISTS, SHOULDER, C-SPINE, HIP ..........................................................
.......................................................................................
.............................329
MEDICINE (DIABETES) = DIABETIC KETOACIDOSIS (DKA) ...........................................................
.........................................................................................
...........................................................
............................. 332
MEDICINE (DIABETES) = HYPEROSMOLAR HYPERGLYCAEMIC NON -KETOTIC (HHNK) STATE .......................................................... ..........................................................336
MEDICINE (DIABETES) = MANAGEMENT OF DIABETES MELLITUS .........................................................
........................................................................................
.................................................
.................. 338
MEDICINE (ENDOCRINE) = CUSHING’S SYNDROME ...........................................................
.........................................................................................
.............................................................
............................................
............. 349
MEDICINE (ENDOCRINE) = ACROMEGALY ..........................................................
.........................................................................................
.............................................................
...........................................................
............................. 354
MEDICINE (ENDOCRINE) = ADDISON’S DISEASE (CHRONIC 1  ADRENAL INSUFFICIENCY ) ..........................................................
0 ..................................................................
........ 358
MEDICINE (ENDOCRINE) = HYPO-PITUITARISM ..........................................................
.........................................................................................
..............................................................
.................................................
.................. 360
MEDICINE (ENDOCRINE) = GYNAECOMASTIA .........................................................
........................................................................................
..............................................................
......................................................
....................... 363
MEDICINE (RENAL) = ACUTE RENAL FAILURE ............................................................
...........................................................................................
..............................................................
.................................................
.................. 364
MEDICINE (RENAL) = CHRONIC RENAL FAILURE .............................................................
............................................................................................
..............................................................
............................................
............. 368
MEDICINE (RENAL) = CRF WITH FLUID OVERLOAD ..............................................................
............................................................................................
.............................................................
.......................................
........ 376
MEDICINE (RENAL) = BALLOTABLE KIDNEYS .........................................................
........................................................................................
..............................................................
......................................................
....................... 377
MEDICINE (RENAL) = TRANSPLANTED KIDNEY ..........................................................
.........................................................................................
..............................................................
.................................................
.................. 379
MEDICINE (RENAL) = APPROACH TO OLIGURIA ...........................................................
..........................................................................................
..............................................................
.................................................
.................. 383
MEDICINE (RENAL) = APPROACH TO PROTEINURIA ..............................................................
............................................................................................
.............................................................
.......................................
........ 384
MEDICINE (RENAL) = HAEMATURIA ..........................................................
.........................................................................................
.............................................................
.............................................................
.......................................
........ 387
MEDICINE (RENAL) = GLOMERULONEPHRITIS ............................................................
...........................................................................................
..............................................................
.................................................
.................. 390
MEDICINE (RENAL) = DGIM RENAL TRANSPLANT ..........................................................
........................................................................................
.............................................................
............................................
............. 396
MEDICINE (GIT) = HEPATOSPLENOMEGALY ...........................................................
..........................................................................................
.............................................................
......................................................
........................ 398
MEDICINE (RESPI) = GENERAL APPROACH TO A HISTORY OF SHORTNESS OF BREATH. ......................................................................
...................................................................... 399
MEDICINE (RESPI) = ACUTE RESPIRATORY DISTRESS SYNDROME (ARDS) ........................................................
........................................................................................
.................................... 400
MEDICINE (RESPI) = SYSTEMIC APPROACH TO CXR..............................................................
............................................................................................
.............................................................
.......................................
........ 402
MEDICINE (RESPI) = LUNG CANCER ..........................................................
.........................................................................................
.............................................................
.............................................................
.......................................
........ 406
MEDICINE (RESPI) = INFECTIONS – TUBERCULOSIS ..............................................................
............................................................................................
.............................................................
.......................................
........ 411
MEDICINE (RESPI) = PNEUMONIA ..............................................................
.............................................................................................
.............................................................
.............................................................
.......................................
........ 416
MEDICINE (RESPI) = ASTHMA ..........................................................
........................................................................................
.............................................................
..............................................................
.................................................
.................. 421
3
MEDICINE (RESPI) = PULMONARY EMBOLISM .............................................................
............................................................................................
..............................................................
.................................................
.................. 427

Acknowledgements

Written by:
Dr Priscilla Phoon & her team of original authors

Transcribed by:
YLLSOM Class of Medicine 2013

Special Thanks to the following people for helping with the add-on:
Ong Eng Hui
Chew Bao Li
Steffi Chan
Teo Yi Lyn
Lucy Davis
Grace Lum

Edited and formatted by:


James Lee (Class of 2013)

Last Updated:
20st  Febuary 2011

4
Short Cases in Medicine
Medicine = Cardio shorts

Prosthetic heart valves


Mdm XXX is an elderly Chinese lady who appears to be alert, well, comfortable and orientated at rest. Her
vitals are as follows = HR 80/min, irregularly irregular. There is no RR delay, RF delay or collapsing pulse. RR is
16/min, not tachypneic or dyspnoeic. She does not appear to be in any respiratory distress and is pink on
room air. On general inspection, I note the presence of a mid-line sternotomy scar with no corresponding
saphenous vein harvest site. There are no signs of pallor, jaundice or cyanosis.

On examination of the peripheries, there are no stigmata of infective endocarditis such as clubbing, splinter
haemorrhages, Janeway lesions or Osler nodes. There is no extensive bruising seen over the arms. I looked
for but did not find any evidence of an enlarged goitre or thyroidectomy scar.

On examination of the praecordium, there was a metallic click audible to the unaided ear. A visible apical
impulse was seen in the 6th intercostal space 1cm lateral to the mid-clavicular line. The apex beat was
heaving in nature. There was no parasternal heave or thrills felt over t he base of the heart. On auscultation,
the first heart sound was metallic and sharp in nature. The second heart sound was native. There were no
additional heart sounds. In addition, there was a grade 3/6 PSM heard loudest over the apex with radiation
to the axilla.

This was not associated with signs of right heart failure as the jugular venous pressure was not raised and
there was no peripheral oedema. Auscultation of the lung bases also did not reveal the prese nce of
inspiratory crepitations.

So in summary, Mdm XXX is an elderly chinese lady who has a prosthetic mitral valve replacement. I say this
because
(a) midline sternotomy scar with no corresponding saphenous vein harvest site
(b) metallic click audible to the unaided ear
(c) sharp and metallic first heart sound heard on auscultation

This is most likely due to severe mitral regurgitation


(a) atrial fibrillation
(b) displaced and heaving apex beat
(c) grade 3/6 PSM heard loudest over the apex with radiation to the axilla

This is not complicated by congestive cardiac failure, infective endocarditis, over-anticoagulation or valve
haemolysis

Mitral stenosis
Mdm XXX is an elderly chinese lady who appears to be alert, well, comfortable and orientated at rest. Her
vitals are as follows = HR 80/min, irregularly irregular. There is no RR delay, RF delay or collapsing pulse. RR is
16/min, not tachypneic or dyspnoeic. She does not appear to be in any respiratory distress and is pink on
room air. On general inspection, there are no signs of pallor, jaundice or cyanosis.

On examination of the peripheries, there are no stigmata of infective endocarditis such as clubbing, splinter
haemorrhages, Janeway lesions or Osler nodes. There is no extensive bruising seen over the arms. I looked
for but did not find any evidence of an enlarged goitre or thyroidectomy scar.

On examination of the praecordium, there were no surgical scars or chest wall deformities. The apex beat
was not displaced. It was in the 5 th intercostal space in the mid-clavicular line and was tapping in nature.
There was no parasternal heave or thrills felt over the base of the heart. On auscultation, the first
f irst heart
5
sound was loud and there was an opening snap followed by a mid-diastolic murmur heard best over the
apex which was accentuated with the patient in the left lateral position. There was no PSM heard over the
tricuspid area or Graham-Steell murmur heard over the pulmonary area.

This was not associated with signs of right heart failure as the jugular venous pressure was not raised and
there was no peripheral oedema. Auscultation of the lung bases also did not reveal the presence of
inspiratory crepitations.
So in summary, Mdm XXX is an elderly chinese lady who has mitral stenosis. I say this because
(a) presence of atrial fibrillation
(b) tapping apex beat which is not displaced
(c) opening snap with a MDM heard best over the apex and accentuated by the patient lying in the left
lateral position

This is not complicated by pulmonary hypertension, congestive cardiac failure, infective endocarditis or
over-anticoagulation

# Request to examine = neurological system (pronator drift for h emiplegia)


peripheral pulses (occlusion by emboli)

Mitral regurgitation
Mdm XXX is an elderly chinese lady who appears to be alert, well, comfortable and orientated at rest. Her
vitals are as follows = HR 80/min, regular. There is no RR delay, RF delay or collapsing pulse. RR is 16/min, not
tachypneic or dyspnoeic. She does not appear to be in any respiratory distress and is pink on room air. On
general inspection, there are no signs of pallor, jaundice or cyanosis.

On examination of the peripheries, there are no stigmata of infective endocarditis such as clubbing, splinter
haemorrhages, Janeway lesions or Osler nodes.

On examination of the praecordium, there were no surgical scars or chest wall deformities. The apex beat
was displaced in the 6 th intercostal space 1cm lateral to the mid-clavicular line and was heaving in nature.
There was no parasternal heave or thrills felt over the base of the heart. On auscultation, the first and
second heart sounds were heard. There was no 3rd heart sound. In addition, there was a grade 3/6 PSM
heard loudest over the apex with radiation to the axilla. There was no radiation to the carotids.

This was not associated with signs of right heart failure as the jugular venous pressure was not raised and
there was no peripheral oedema. Auscultation of the lung bases also did not reveal the pre sence of
inspiratory crepitations.

So in summary, Mdm XXX is an elderly chinese lady who has mitral regurgitation. I say this because
(a) displaced apex beat which is heaving in nature
(b) grade 3/6 PSM heard loudest over the apex with radiation to the axilla

This is not complicated by congestive cardiac failure or infective endocarditis

Aortic stenosis
Mdm XXX is an elderly chinese lady who appears to be alert, well, comfortable and orientated at rest. Her
vitals are as follows = HR 80/min, regular. There is no RR delay, RF delay or collapsing pulse. However, I note
that the pulse is of low-volume and slow-rising in nature. RR is 16/min, n ot tachypneic or dyspnoeic. She
does not appear to be in any respiratory distress and is pink on room air. On general inspection, there are
no signs of pallor, jaundice or cyanosis.

On examination of the peripheries, there are no stigmata of infective endocarditis such as clubbing, splinter
haemorrhages, Janeway lesions or Osler nodes.

On examination of the praecordium, there were no surgical scars or chest wall deformities. The ape x beat is

6
not displaced and is thrusting
thrusting in nature. There was no parasternal heave or thrills felt over the base of
of the
heart. On auscultation, the first and second heart sounds were heard. There was no 4  heart sound. In
th

addition, there was a grade 3/6 ESM heard loudest over the aortic area with radiation to the carotids
which was accentuated by forced expiration.

This was not associated with signs of right heart failure as the jugular venous pressure was not raised and
there was no peripheral oedema. Auscultation of the lung bases also did not reveal the presence of
inspiratory crepitations.

So in summary, Mdm XXX is an elderly chinese lady who has aortic stenosis. I say this because
(a) low-volume slow-rising
slow-rising pulse (‘anacrotic pulse’)
(b) apex beat is not displaced and is thrusting in nature
(c) grade 3/6 ESM heard loudest over the aortic area with radiation to the carotids and accentuated by
forced expiration

This is not complicated by congestive cardiac failure or infective endocarditis

# Request = BP (narrow pulse pressure)

Aortic regurgitation
Mdm XXX is an elderly chinese lady who appears to be alert, well, comfortable and orientated at rest. Her
vitals are as follows = HR 80/min, regular. There is a collapsing pulse noted but no RR delay or RF delay. RR is
16/min, not tachypneic or dyspnoeic. She does not appear to be in any respiratory distress and is pink on
room air. On general inspection, there are no signs of pallor, jaundice or cyanosis.

On examination of the peripheries, there are no stigmata of infective endocarditis such as clubbing, splinter
haemorrhages, Janeway lesions or Osler nodes.

On examination of the praecordium, there were no surgical scars or chest wall deformities. The apex beat is
displaced in the 6 th intercostal 1cm lateral to the mid-clavicular line and
and is heaving in nature. There was no
parasternal heave or thrills felt over the base of the heart. On auscultation, the first and second heart
sounds were heard. There was no 3rd heart sound. In addition, there was a grade 2/6 EDM heard loudest
over the upper left sternal edge which was accentuated with forced expiration. There was no Austin-Flint
murmur detected.

This was not associated with signs of right heart failure as the jugular venous pressure was not raised and
there was no peripheral oedema. Auscultation of the lung bases also did not reveal the presence of
inspiratory crepitations.

So in summary, Mdm XXX is an elderly chinese lady who has aortic regurgitation. I say this because
(a) collapsing pulse
(b) displaced apex beat which is heaving in nature
(c) grade 2/6 EDM heard loudest over the upper left sternal edge and accentuated by forced expiration

This is not complicated by congestive cardiac failure or infective endocarditis

# Request = BP (wide pulse pressure; Hill’s sign)


other features of AR

7
Tricuspid regurgitation
Mr XXX is a young chinese gentleman who appears to be alert, well, comfortable and orientated at rest. His
vitals are as follows = HR 80/min, regular. There is no RR delay, RF delay or collapsing pulse. RR is 16/min, not
tachypneic or dyspnoeic. He does not appear to be in any respiratory distress and is pink on room a ir. On
general inspection, there are no signs of pallor or cyanosis. However, he appears to be jaundiced.

On examination of the peripheries, there are no stigmata of infective endocarditis such as clubbing, splinter
haemorrhages, Janeway lesions or Osler nodes. I did not note the presence of needle tracks in the cubital
fossae.

On examination of the praecordium, there are no surgical scars or chest wall deformities. The apex beat is
not displaced and is normal in nature. There was a parasternal heave detected but no thrills were felt over
the base of the heart. On auscultation, the first and second heart sounds were heard. There was no loud
P2. In addition, there was a grade 3/6 PSM heard l oudest over the lower left sternal edge which was
accentuated with forced inspiration. I did not hear a MDM which might be suggestive of mitral stenosis.

This is associated with signs of right heart failure as the jugular venous pressure was raised till the level of the
mid-neck with giant v waves seen. There was also bilateral lower limb pitting oedema till the level of the
knees. However, auscultation of the lung bases also did not reveal the presence of inspiratory crepitations.
So in summary, Mr XXX is a young chinese gentleman who has tricuspid regurgitation. I say this because
(a) jaundiced
(b) parasternal heave but with no other signs of pulmonary hypertension
(c) grade 3/6 PSM heard loudest over the lower left sternal edge and accentuated by forced inspiration
(d) signs of right heart failure with raised JVP, giant v waves and lower limb pitting oedema

This is not complicated by left heart failure or infective endocarditis

# Request = abdomen (pulsatile liver, hepatomegaly, splenomegaly)


respiratory system (COPD, bronchiectasis, pulmonary fibrosis)

8
Medicine = Respi shorts

Bronchiectasis
(after examination) I would like to complete my examination by requesting for the vitals, sputum mug as
well as to examine the patient for a raised JVP and splenomegaly.

Mdm XXX is a middle-aged chinese lady who appears to be alert at rest. Her vitals are as follows: HR 80/min
regular and not bounding. She appears to be in respiratory distress as evidenced by tachypnoea with a RR
of 24/min, on supplemental oxygen via nasal prongs at 2L/min, use of accessory muscles of r espiration as
well as the presence of intercostal retractions. However, there is no cyanosis or terminal asterixis. I also note
the presence of intravenous antibiotics hanging by the drip-stand suggesting that there is an underlying
infective process going on. There is also no sputum mug or bronchodilators by the bedside. On general
inspection, she does not appear to be cachexic. There are no signs of pallor or jaundice.

On examination of the peripheries, I note digital clubbing. However, there are no signs of t ar stains, wasting
of the intrinsic hand muscles or swelling and pain over the wrist joints. There are no signs suggestive of
Horner’s syndrome. There is no displacement of the trachea or apex beat.

On examination of the chest, there are no surgical scars or chest wall deformities. The main physical
findings on examination are coarse pan-inspiratory crepitations heard t hroughout the posterior chest which
do not clear with coughing. This is associated with decreased air-entry and chest expansion, resonant
percussion note and a normal vocal resonance.

(infective exacerbation) In addition, there are also signs suggestive of consolidation in the ri ght lower third
of the posterior chest. I say this because there is decreased chest expansion, dullness to percussion,
decreased air-entry with bronchial breath sounds and i ncreased vocal resonance.

There is no cervical lymphadenopathy or signs of pulmonary hypertension. There was no parasternal heave
or palpable P2 detected. I would have liked to examine the neck for a raised JVP but I note that the
patient does not have lower limb oedema.

So in summary, Mdm XXX has features suggestive of an infective exacerbation of bronchiectasis. I say this
because
(a) digital clubbing
(b) coarse pan-inspiratory crepitations that do not clear with coughing
She is currently in respiratory distress but her condition is not complicated by pulmonary hypertension or cor
pulmonale

My differentials are
(a) infective exacerbation of COPD  signs of hyperinflation, expiratory rhonchi, prolonged expiratory
phase
(b) pulmonary fibrosis  dry cough, steroid toxicity, fine end-inspiratory crepitations

Pleural effusion
(after examination) I would like to complete my examination by requesting for the vitals and sputum mug.

Mr XXX is an elderly chinese gentleman who appears to be alert at rest. His vitals are as follows: HR 80/min,
regular and not bounding, RR 16/min, not tachypneic or dyspnoeic. He does not appear to be in any
respiratory distress and is pink on room air. I do not note the presence of a sputum mug or bronchodilators
by the bedside. On general inspection, he does not appear to be cachexic. There are no signs of pallor,
 jaundice or cyanosis.

On examination of the peripheries, there is no evidence of digital cl ubbing, tar stains, wasting of the intrinsic
hand muscles or pain and swelling over the wrist joints. Ther e are no features suggestive of Horner’s
syndrome. There is also no displacement of the trachea or apex beat.

9
On examination of the chest, I did not note any surgical scars or chest wall deformities. The main physical
findings are that of a right-sided pleural effusion. I say this because there is decreased chest expansion over
the right lower third of the posterior chest associated with stony dull percussion, decreased breath sounds
as well as decreased vocal resonance. There was no cervical lymphadenopathy.

I looked for but did not find any underlying aetiology. In particular, there were no other abnormal chest
findings, hand deformities, characteristic malar rash or stigmata of chronic liver and renal disease. I would
have liked to examine the cardiovascular system in detail but I note that there is no lower limb oedema.

So in summary, Mr XXX is an elderly chinese gentleman who has a right-sided pleural effusion. I say this
because the right lower chest
(a) decreased chest expansion
(b) stony dull percussion note
(c) decreased air entry
(d) decreased vocal resonance
This is likely to be a small effusion as there is no mediastinal displacement. He is currently not in respiratory
distress

Pulmonary fibrosis
(after examination) I would like to complete my examination by requesting for the vitals and sputum mug
as well as to examine the patient for a raised JVP

Mdm XXX is a middle-aged chinese lady who appears to be alert at rest. Her vitals are as follows: HR
80/min, regular and not bounding. She appears to be in respiratory distress as evidenced by tachypnoea
with a RR of 24/min, on supplemental oxygen via nasal prongs at 2L/min, use of accessory muscles of
respiration as well as the presence of intercostal retractions. There is also evidence of central cyanosis.
However, there is no terminal asterixis. On general inspection, she does not appear to be cachexic. There
are no signs of pallor or jaundice.

On examination of the peripheries, I note the presence of digital clubbing. However, there are no tar stains,
wasting of the intrinsic hand muscles or tenderness and swelling over the wrist joints. There are also no signs
suggestive of Horner’s syndrome. The trachea and apex beat are not displaced.

On examination of the chest, there are no surgical scars or chest wall deformities. The main physical
findings are suggestive of bibasal pulmonary fibrosis. I say this because there is dullness to percussion over
the lung bases associated with decreased air-entry and fine end-inspiratory crepitations that do not clear
with coughing. Vocal resonance is normal. There is no cervical lymphadenopathy.

There is no evidence of pulmonary hypertension as there was no parsternal heave or palpable P2


detected. I would have liked to examine the patient for a raised JVP but I note that there is no lower limb
oedema.

So in summary, Mdm XXX is an elderly chinese lady who has bilateral lower lobe fibrosis. I say this because
of
(a) digital clubbing
(b) bibasal fine end-inspiratory crepitations which do not clear with coughing
She is currently in respiratory distress as evidenced by tachypnoea and central cyanosis. However, her
condition is not complicated by pulmonary hypertension or cor pulmonale.

My differentials are
(a) congestive cardiac failure with pulmonary oedema  no clubbing, evidence of fluid overload,
crepitations clear with coughing
(b) bronchiectasis  productive cough, coarse pan-inspiratory  expiratory crepitations

10
Chronic obstructive pulmonary disease
(after examination) I would like to complete my examination by requesting for the vitals and sputum mug
as well as to examine the patient for liver ptosis and raised JVP

Mr XXX is an elderly chinese gentleman who appears to be alert at rest. His vitals are as follows: HR 80/min,
regular and not bounding. He appears to be in respiratory distress as evidenced by tachypnoea with a RR
of 24/min, on supplemental oxygen via nasal prongs at 2L/min and use of accessory muscles of r espiration.
However, he does not appear cyanosed nor is there terminal asterixis. I note the presence of intravenous
antibiotics hanging by the drip-stand suggesting that there is an underlying infective process going on.
However, there is no sputum mug or bronchodilators by the bedside. On general inspection, he does not
appear to be cachexic. There are no signs of pallor or jaundice.

On examination of the peripheries, there is no sign of digital clubbing, tar stains, wasting of the intrinsic
hand muscles or tenderness and swelling over the wrist joints. There are also no signs suggestive of Horner’s
syndrome. The trachea and apex beat are not displaced.

On examination of the chest, there are no surgical scars or chest wall deformities. However, there are signs
of hyperinflation as evidenced by
(a) barrel-shaped chest = increased antero-posterior diameter cg lateral diameter
(b) decreased chest expansion
(c) resonant percussion note
(d) loss of cardiac and liver dullness
(e) decreased air-entry associated with expiratory wheeze and prolonged expiratory phase
(f) decreased vocal resonance

There is no cervical lymphadenopathy or signs of pulmonary hypertension as there was no parasternal


heave and palpable P2 detected. I would have liked to examine the patient for a raised JVP and I note
that there is unlikely to be right heart failure as there is no lower limb oedema

So in summary, Mr XXX is an elderly chinese gentleman who has evidence suggestive of an infective
exacerbation of COPD. I say this because
(a) signs of hyperinflation
(b) decreased air-entry, expiratory wheeze and prolonged expiratory phase
He is currently in respiratory distress but his condition is not complicated by pulmonary hypertension or cor
pulmonale

My differentials are
(a) infective exacerbation of bronchial asthma
(b) infective exacerbation of bronchiectasis  clubbing, coarse-inspiratory crepitations

Consolidation
(after examination) I would like to complete my examination by requesting for the vitals and sputum mug

Mr XXX is an elderly chinese gentleman who appears to be alert at rest. His vitals are as follows: HR 80/min,
regular and not bounding, RR 16/min not tachypneic or dyspnoeic. He does not appear to be in any
respiratory distress and is pink on room air. There is an intravenous antibiotic hanging on the drip-stand
suggesting an underlying infective process. On general inspection, Mr XXX does not appear to be
cachexic. There are/are no signs of pallor, cyanosis or jaundice.

On examination of the peripheries, there is no sign of digital clubbing, tar stains, wasting of the intrinsic
hand muscles or tenderness and swelling over the wrist joints. There are also no signs suggestive of Horner’s
syndrome. The trachea and apex beat are not displaced.

On examination of the chest, there are no surgical scars or chest wall deformities. The main physical
findings are in the lower 1/3 of the right posterior chest which is suggestive of consolidation. I say this

11
because there is decreased chest expansion, dullness to pe rcussion, decreased air-entry associated with
coarse pan-inspiratory crepitations, bronchial breathing as well as increased vocal resonance. There is no
cervical lymphadenopathy.

So in summary, Mr XXX is an elderly chinese gentleman who has evidence suggestive of consolidation in
the lower 1/3 of the right posterior chest. I say this because
(a) decreased chest expansion
(b) dullness to percussion
(c) decreased air-entry, coarse pan-inspiratory crepitations, bronchial breathing
(d) increased vocal resonance
He is currently not in respiratory distress.

Collapse
(after examination) I would like to complete my examination by requesting for the vitals and sputum mug

Mr XXX is an elderly chinese gentleman who appears to be alert at rest. His vitals are as follows: HR 80/min,
regular and not bounding, RR 16/min not tachypneic or dyspnoeic. He does not appear to be in any
respiratory distress and is pink on room air. On general inspection, Mr XXX appears to be cachexic. There
are no signs of pallor, cyanosis or jaundice.

On examination of the peripheries, there is no sign of digital clubbing, tar stains, wasting of the intrinsic
hand muscles or tenderness and swelling over the wrist joints. There are also no signs suggestive of Horner’s
syndrome. There is tracheal deviation to the right with no mediastinal displacement.

On examination of the chest, there are no surgical scars or chest wall deformities. The main physical
findings are in the upper 1/3 of the right anterior chest suggestive of an upper lobe collapse. I say this
because of right tracheal deviation, flattening of the right chest wall, decreased chest expansion, dullness
to percussion, decreased air-entry as well as decreased vocal resonance. There is no cervical
lymphadenopathy.

So in summary, Mr XXX is an elderly chinese gentleman who has evidence suggestive of a right upper lobe
collapse. I say this because
(a) right tracheal deviation
(b) flattening of right chest wall
(c) decreased chest expansion
(d) dullness to percussion
(e) decreased air-entry and vocal resonance
He is currently not in respiratory distress.

# important to exclude malignancy


# Brock’s syndrome = collapse due to compression of right middle lobe bronchus by enlarged lymph node

12
Lung cancer
(after examination) I would like to complete my examination by requesting for the vitals and sputum mug

Mr XXX is an elderly chinese gentleman who appears to be alert at rest. His vitals are as follows: HR 80/min,
regular and not bounding, RR 16/min not tachypneic or dyspnoeic. He does not appear to be in any
respiratory distress and is pink on room air. On g eneral inspection, I note that he is cachexic. There are/are
no signs of pallor or jaundice.

On examination of the peripheries, I note the presence of digital clubbing as well as hypertrophic
pulmonary osteoarthropathy. However, there are no tar stains, wasting of the intrinsic hand muscles or
features suggestive of Horner’s syndrome.

On examination of the chest, there are no surgical scars or chest wall deformities. The main physical
findings are that of a collapse-consolidation over the right upper 1/3 of the posterior chest as well as a
right-sided pleural effusion involving the lower 2/3 of the posterior chest. I say this because
(a) collapse-consolidation
- tracheal deviation to the right
- dull percussion note
- decreased air-entry with no adventitious sounds
- increased vocal resonance
(b) pleural effusion
- decreased chest expansion over the right lower chest
- stony dull percussion note
- decreased air-entry with no adventitious sounds
- decreased vocal resonance
- likely to be moderate in size as the apex beat is slightly displaced in the 6 th intercostal space 1cm lateral
to the mid-clavicular line

In addition, multiple small enlarged cervical lymph nodes were found bilaterally ranging from 1-2 cm in
length. They were non-tender, firm, matted and relatively immobile.

So in summary, Mr XXX has multiple chest findings i ncluding a right upper lobe collapse-consolidation as
well as a right-sided pleural effusion. He most likely has a right lung malignancy. This is supported by the
findings of cachexia, pallor, clubbing, HPOA as well as cervical lymphadenopathy.

I would like to examine the patient for hepatomegaly, focal neurological deficits and to percuss the
vertebral column for tenderness.

13
Medicine = Renal shorts

Polycystic kidney disease


I would like to complete by doing a per- rectal examination and requesting for the patient’s vitals esp the
BP. In addition, I would like to examine the cardiovascular system for MVP and the neurological system for a
focal neurological deficit.

Mr XXX is a young Chinese gentleman who appears to be alert and comfortable. His vitals are as follows =
HR ____, RR ____. On examination, he has bilateral ballotable kidneys most likely due to adult polycystic
kidney disease and is in ESRF on haemodialysis.

(confirm findings) I say this because on examination of the abdomen, I note a distinct fullness over the left
and right flanks. On palpation, there were bilateral ovoid masses measuring ___ cm by ___ cm, non -tender
and firm. I was able to get above the masses and they did not move with respiration. No splenic notch was
felt. The masses were ballotable and a band of resonance was detected on percussion.

(aetiology) I looked for but did not find any hepatosplenomegaly. There was no apparent focal
neurological deficit as Mr XXX was able to move all 4 limbs. However, I would like to confirm this by doing a
detailed neurological examination. In addition, I did not note the presence of diabetic dermopathy.

(complications) Functionally, Mr XXX is in ESRF as evidenced by his sallow appearance and conjunctival
pallor. In addition, I also note the presence of an AVF in the left cubital fossa with a palpable thrill and signs
of recent cannulation. However, Mr XXX does not appear to be uraemic as there are no signs of brui sing,
scratch marks or terminal asterixis. He is also not in fluid overload as t here is no lower limb oedema, ascites
and he is able to lie flat in bed with no signs of respiratory distress.

(summary) In summary, Mr XXX is a young Chinese gentleman who m ost likely has adult polycystic kidney
disease. I say this because of the presence of bilateral ballotable kidneys. This is complicated by end-stage
renal failure and Mr XXX is currently being managed by haemodialysis. He is not in uraemia or fluid
overload.

Transplanted kidney
Mdm XXX is a (age)(race)(gender) who has a transplanted kidney and is on immunosuppressive therapy

I say this because she has a J-shaped scar in her left iliac fossa, overlying a rounded mass x cm by x cm
which is non tender and firm to touch. There is no hepatosplenomegaly or ascites noted

In addition, she also has evidence of immunosuppression with a characteristic rounded facies, central
obesity, violaceous abdominal striae, oral thrush, gum hypertrophy, bruising and thin skin

I looked for but was unable to find any signs suggestive of t he aetiology of end stage renal failure such as
bilaterally enlarged ballotable kidneys and diabetic dermopathy

Functionally, I note that she has a left arteriovenous fistula with a palpable thrill. There are no signs of recent
cannulation which suggests that the graft is functioning well. This is supported by the fact that she does no t
have any evidence of uraemia. She does not appear sallow and there are no signs of bruising, scratch
marks or asterixis. She is also not in fluid overload as she is able to lie flat in bed with no signs of respiratory
distress and there are no signs of ascites or lower limb oedema

14
Medicine = Endocrine shorts

Cushing’s syndrome
Mdm XXX is a middle-aged Chinese lady who appears to be alert and comfortable at rest. On general
inspection, I note that she has Cushingnoid features as evidenced by
(a) characteristic rounded facies with facial plethora, hirsutism and acne
(b) central deposition of adiposity with thick violaceous abdominal striae
(c) supraclavicular and dorsal fat pads
(d) skin atrophy, bruising, proximal myopathy
(e) cataracts
(f) oral thrush

During the examination, I looked for but did/did not find any evidence of
(a) deforming arthropathy  RA, SLE
(b) characteristic malar rash  SLE
(c) clubbing or tar stains  small cell lung ca
(d) expiratory rhonchi or fine end-inspiratory bibasal crepitations  asthma, COPD, IPF
(e) transplanted kidney/liver

There are no hypocount scars over the finger-t ips or diabetic dermopathy which may suggest the presence
of DM as a complication. However, I would like to confirm this by performing a urine dipstick to look for
glycosuria. In addition, I would like to take the BP for hypertension.

To end off my examination


(a) visual field  bitemporal hemianopia (pituitary adenoma)
(b) fundoscopy  cataracts
diabetic/hypertensive retinopathy
papilloedema/optic atrophy (SOL in optic chiasm)

Thyrotoxicosis
I would like to complete my examination by requesting for the patient’s vitals, performing Pemberton’s sign
 , checking for hyper-reflexia as well as performing a cardiovascular examination looking out for signs of
congestive cardiac failure.

Miss XXX is a young Chinese lady who appears to be alert and comfortable at rest. She does not appear to
be agitated or nervous. On general inspection, I note that she has a diffuse anterior neck swelling. This is
most likely the thyroid gland as it moves with swallowing but not with tongue protrusion. There are no
overlying skin changes, dilated veins or previous surgical scars.

On palpation, the thyroid gland measured 10cm by 5cm in dimensions. There was no increased warmth or
palpable thrill. It was non-tender, firm in consistency and had a smooth and regular surface. It was not
attached to overlying skin or underlying muscle. There was no cervical lymphadenopathy or displacement
of the carotids and trachea. Retrosternal extension is unlikely as the inf erior border of the gland was well
felt. In addition, there was no dullness to percussion over the manubrium. There was an audible bruit heard
over both lobes on auscultation.

Miss XXX is likely to be in thyrotoxicosis. I say this because she is in sinus tachycardia with a HR of 120/min. In
addition, she has warm and sweaty palms, palmar erythema as well as fine tremors. I did not note the
presences of thyroid acropathy, proximal myopathy or pre-tibial myoxedema. Furthermore, Miss XXX has
also features of thyroid eye disease as evidenced by lid retraction, exophthalmos and lid lag. However,
there is no proptosis, chemosis, limitation in eye movement or lagophthalmos.

So in summary, Miss XXX is a young Chinese lady who most likely has Graves’ disease complicated by
thyroid eye disease and is currently in thyrotoxicosis.

15
Laboratory findings
1. FBC
 NCNC anemia = anemia of chronic disease
Hemolytic anaemia -> reticulocyte count, hepa toglobin, LDH, direct Coomb’s test 
 Leucopenia/lymphopenia
 Thrombocytopenia
2. ESR, CRP
 ESR = raised
 CRP = normal (consider infection if raised)
3. PT/PTT
 Prolonged aPTT in anti phospholipid syndrome
4. U/E/Cr
 Renal impairment
 Proceed to do urine dipstick, UFEME, urine c/s , urine phase contrast, 24 hr CCT/UTP, urine PCR, renal
biopsy
5. Autoimmune markers
 ANA = sensitive but not specific
 Anti-dsDNA = specific
 Anti-Sm = specific
 Anti-Rho and anti-La (complete heart block in neonate)
 anti phospholipid Ab (lupus anticoagulant, anticardiolipin Ab)
6. Monitor disease activity
 anti-dsDNA = high
 serum complement = low C3 and C4
High C3 degradation product
 ESR = high (do CRP to distinguish lupus flare from infection)

Management
General measures
 Avoid sunlight = carry umbrella, wear sun block
 Wear warm socks and gloves for Raynaud’s phenomenon
 Avoid drug provocation (penicillin, sulphonamides)
Pharmacotherapy
 No curative therapy
 Different modalities
(a) Joint symptoms = NSAIDs
(b) Skin symptoms/joint symptoms not controlled by NSAIDs = hydroxychloroquine (annual eye check for
maculopathy)
(c) Renal involvement = steroid and pulsed IV cyclophosphamide
(d) Severe episodes = high dose prednisolone, cytotoxics (azathioprine, cyclophosphamide, methotrexate)
(e) Chronic disease = low dose prednisolone

Prognosis
 poor prognostic factors
(a) Renal disease (esp class IV)
(b) Hypertension
(C) male
(d) Young age
43
(e) APLS
(f) High disease activity
 prognosis = 90% 5 year survival
80% 10 year survival

Pregnancy and SLE


 Avoid during active disease (esp with sig organ impairment) due to high risk of spontaneous miscarriage
and exacerbation of SLE
 Should wait until disease has been quiescent for at least six months before attempting pregnancy
 Management of patients with active lupus = corticosteroids, NSAIDs and hydroxychloroquine
 Cyclophosphamide and methotrexate are contraindicated
 Azathioprine can be used cautiously
 Patients with migraine headaches, Raynaud ’s phenomenon, history of phlebitis or APL Ab should not be
treated with OCPs (increases risk of thrombosis)

History taking
1. Malar rash
2. Discoid rash
3. Photosensitivity
4. Alopecia, dry eyes and mouth, oral ulcers
5. Gangrene of fingers, Raynaud ’s phenomenon
6. Chest pain, dyspnoea
7. Joint pain
8. Seizures
9. Change in urinary frequency and volume, haematuria, frothy urine, loin pain
10. Anaemia = pallor, chest pain, palpitation, fatigue, giddiness, dyspnoea, jaundice
Leukopenia = susceptibility to infections
Thrombocytopenia = gum bleeding, easy bruising, menorrhagia
APLS = history of recurrent spontaneous abortion, DVT/PE, AMI, CVA
11. Constitutional (fever, LOA, LOW, malaise)

Examination
“This patient most likely has SLE as evidenced by the butterfly rash affecting the nose bridge but sparing
the nasolabial folds.”

Proceed with the following:


General appearance
 Weight loss (due to chronic inflammation)
 Cushingnoid appearance (due to steroid therapy)
Hands
 Nails (splinter hemorrhages, nail-fold infarcts)
 Gangrene (vasculitis
 Palmar erythema
 Raynaud’s phenomenon (white-blue-red)
 Arthropathy
Arms
 Livedo reticularis (bluish purple streaks without discrete borders)

44
 Purpura (vasculitis or autoimmune thrombocytopenia)
 Proximal myopathy (due to disease or steroid use)
Face
 Conjunctiva pallor
 Mouth ulcers
 Alopecia
Chest
 CVS = pericardial rub
 Lungs = pleural rub, pleural effusion, pulmonary fibrosis
Abdomen
 Mild splenomegaly +/-hepatomegaly
Legs
 Vasculitic rash
 Lower limb pitting edema (due to lupus nephritis)

Request to look at:


1. Vitals = temperature and BP
2. Urine dipstick = proteinuria haematuria

45
Medicine (Rheumatology) = GALS screen
Gait, Arms, Legs, Spine => look at appearance and movement

History
1. Have you had any pain or stiffness in your muscles, joints or back?
 Cardinal symptoms of rheumatic disease
2. Can you dress yourself completely without any difficulty?
 ADL = assessing functional problem of UL
3. Can you walk up and down stairs without any difficulty?
 ADL = assessing functional problem of LL

Physical examination (examine patient wearing his underwear only)


1. Gait
 Ask patient to stand
 Ease of transfer from chair/ lying position to standing position
 Get patient to walk, turn around and walk back
 Symmetry and smoothness of movement (legs, arm swing, pelvic tilting)
 Normal stride length
 Ability to turn quickly
2. Spine
 Inspection (from the back and side)
 Start from the back
 Scoliosis
 Symmetry of paraspinal muscles and girdle muscles
 Symmetrical pelvic position and level iliac crests
 Inspect from the side
 Excessive thoracic kyphosis
 Loss/excessive lumbar lordosis
 Movement
 Squeeze midpoint of supraspinatus muscle -> hyperalgesic response of fibromyalgia
 Schober’s test = measure of lumbar excursion
 From front
 Lateral flexion of C-spine (Place your ear on your shoulder)
3. Arms
 Place arms behind head
 Measure of shoulder abduction and external rotation
 Observe movement at glenohumeral, acromioclavicular and sternoclavicular joints
 Place arms by the side with palms facing outwards
 Full elbow extension
 Normal girdle muscle bulk and symmetry
 Bend elbows at 90ᵒ and pronate/supinate
 Wrist flexion and extension
 Lift elbows up
 Subcutaneous nodules
 Movement
 Clench fists and test grip strength

46
 Fingers on thumb
Measurement of fine movements
 Squeeze across MCP joints
 Early arthritis = pain and tenderness on squeezing before other abnormalities seen
4. Legs
 Inspection
 Leg
 Deformities
 Knee
 Bulk of quadriceps muscle
 Loss of parapatellar fossae
 Feet
 Callus formation = abnormal weight bearing
 Movement
 Fully flex knee and hip joint
 Place hand on knee joint to feel for crepitus
 Internally and externally rotate hip joint
 Squeeze across MTP joints
 Early arthritis = pain and tenderness on squeezing before other abnormalities seen

47
Medicine (Rheumatology) = Rheumatoid Arthritis

Overview
 Description
► Systemic chronic inflammatory disease affecting multiple tissues but principally attacking joints to produce a non- suppurative proliferating
synovitis that frequently progresses to destroy articular cartilage and underlying bones with resulting disabling arthritis.
 Epidemiology
► Very common = ∼ 1% (higher in smokers)
► Female > Males (3:1)
► Peak incidence = 4th/5th decades of life
 Pathogenesis
► Initiation by an arthritogenic antigen with subsequent autoimmune reaction in which T cells release c ytokines and inflammatory mediators that
ultimately destroy the joint.
► Causative microbial triggers are unknown but suspects include EBV, Borrelia species, Mycoplasma species, retrovirus and mycobacterium.

Principles of Diagnosis

 History
► Arthritis
- Classically, swollen, painful, stiff hands and feet worse in the morning
- Chronic inflammatory joint disease with relapsing and remitting course
- Insidious onset with joint pain and early morning stiffness
- Symmetrical polyarthropathy = PIPJ, MCPJ, wrist, MTPJ and knees (spares distal DIPJ)
- Joints progressively enlarge → limited ROM and complete ankylosis (stiffness due to abnormal adhesion and rigidity of the bones of the joint )
► Constitutional symptoms
- LoA, LoW, fatigue, fever, rash
- Anemia → chest pain, SOB, giddiness, palpitations, fatigue
► Extra-articular involvement
- Skin = Raynaud’s phenomenon, rash
- Head and Neck = red eyes, dry eyes and mouth (Sjögren’s syndrome)
- Pulmonary and Cardiac = chest pain, SOB
- CNs = numbness, parasthesiae, weakness
► Atypical presentations
- Palindromic = acute recurrent, relapsing, remittent arthritis usually affecting 1 large joint for a few hours, with symptom-free intervals of days
– months between attacks. (‘Was I Saw!’ → wrist, ankle, shoulder, IPJ)
- Persistent monoarthritis
- Systemic = pericarditis, pleurisy, LoW, constitutional symptoms
- Acute onset of widespread arthritis
48
 Extra-articular involvement
1. Eyes
 Sclera – episcleritis, scleritis, scleromalacia, scleromalacia perforans
 Conjunctiva – pallor, keratoconjunctivitis sicca (Sjögren’s syndrome)
 Lens – cataracts from steroid use
 Extra-Ocular Muscles – mononeuritis multiplex, myasthenia 2º penicillamine, EOM tendon synovitis
 Fundi – maculopathy from hydroxycholoroquine use
2. Head and Neck
 Mouth – ulcers from DMARD treatment, dry mouth and enlarged parotids (Sjögren’s)
 TMJ – crepitus
 Neck – tenderness, muscle spasm, limited ROM (atlanto axial sublux, basilar invagination by dens protrusion,
subcervical spine)
3. Respiratory system
 Upper airway – cricoarytenitis
 Pleura – pleural effusion, pleurisy
 Bronchioles – bronchiolitis obliterans and organizing pneumonia (BOOP)
 Parenchyma – lower lobe pulmonary fibrosis, penumonitis, rheumatoid nodules
 Infiltration – Caplan’s (rheumatoid nodules in periphery of lung fields a/w coal worker’s pneumoconiosis)
4. CVS
 Pericarditis
 Aortic/mitral regurgitation
5. Lympadenopathy
6. GIT
 Splenomegaly (5%)
 Felty’s syndrome (1%) = RA w/splenomegaly and hypersplenism →anemia, leukopenia, thrombocytopenia and leg ulcers (ameliorated by
splenectomy)
 Methotrexate use → hepatomegaly
7. Upper Limb
 Vasculitis = nail-fold infarcts, splinter hemorrhage, telangiectasia, Raynaud’s phenomenon
 Subcutaneous nodules (indicates seropositivity and more aggressive arthritis, found on flexor and myocardium)
 Entrapment neuropathy
8. Lower Limb
 Hip – limited ROM
 Knees – quadriceps wasting, synovial effusion, flexion contracture, genu valgus deformity, Baker’s cysts in popliteal fossae
 Lower Leg – leg ulcers, calf swelling (ruptured Baker’s cyst), peripheral neuropathy, mononeuritis multiplex
 Ankle – limited ROM, nodules on Achilles tendon
 Feet – foot drop (peroneal nerve entrapment), MTPJ (swelling, subluxation)
 Differentials
49
Diagnosis of RA made when ≥4 criteria are met (93% sensitivity and 90% specificity)

 Morning stiffness >1 hours

 Arthritis of ≥3 joints Fluid-filled presence of soft tissue swelling in


the following: wrist, PIP, MCP, elbow, knee,
ankle, MTP

 Arthritis of hand joints Wrist, MCP, or PIP joints among the


≥6 weeks
symptomatic joints observe

 Arthritis is symmetrical Right and left joints involved for one or more of
the following: wrist, PIP, MCP, knee, MTP, elbow,
ankle

 Rheumatoid nodules Subcutaneous nodules in regions surrounding


joints, flexor/extensor surfaces, or bony
prominences, sacrum, Achilles, sclera

 Rheumatoid factor +ve

 Radiological changes Hand and wrist films

51
Complications
 Complications of disease
‐ Increased risk of IHD and l ymphoma
‐ Ruptured tendons
‐ Joint destruction and resultant disability
‐ Cervical myelopathy
‐ Amyloidosis → proteinuria, nephritic syndrome and renal failure
 Side effects of therapy
‐ Dyspepsia, BGIT, asthma (NSAIDs)
‐ Renal impairment (NSAIDs, penicillamine)
‐ Proteinuria (gold salts, penicillamine)
‐ Anemia (NSAIDs)
‐ Bone marrow depression (DMARDs)
 5 causes of anemia in RA
1. Anemia of chronic disease
2. Iron deficiency anemia
 BGIT due to NSAIDs use
3. Megaloblastic anemia
 Increased cellular turnover (folate acid ‘deficiency’)
 Methotrexate use
 Pernicious anemia
4. Hypersplenism
 2º to Felty’s syndrome
5. Aplastic
 BM suppression due to gold and penicillamine use

Assessment of disease severity


 Symptoms
‐ Duration of morning stiffness
‐ Pain score
‐ Severity of fatigue
 P/E
‐ Number of swollen joints
‐ Number of tender joints
‐ Degree of swelling ± tenderness
‐ Extra-articular disease
 Lab values
‐ ↑ESR and CRP = active disease, infection, Amyloidosis, Sjögren’s disease

52
‐ Anemia
‐ Rh factor titres = correlates with likelihood that patient has extra-articular disease (not activity of arthritis)
‐ Inflammatory joint fluid = high polymorph count, low complement, fibrin
 Imaging
‐ Progressive bony erosions on serial X-ray films
‐ Low bone marrow density
Sjögren’s syndrome
‐ Connective tissue disorder a/w dry eyes(keratoconjunctivitis sicca) and dry mouth (xerostoma)
‐ May be a/w autoimmune thyroid disease, MG or autoimmune liver disease
‐ Ix = Schirmer filter paper test (crude measure of tear production; <5mm, N → at least 15mm after 5 mins)
‐ Tx = artificial tears, artificial saliva and NSAIDs

Principles of Management

 Treatment modalities

Symptomatic relief

Paracetamol Pain

NSAIDs Pain Give with PPI or H2-R blocker


- Aspirin, Ibuprofen CI = BGIT, PUD, Asthma S/E = BGIT, interstitial nephritis,
- COX 2 inhibitor(Arcoxia), if bronchoconstriction
elderly
DMARDS (mono or combination therapy)
- Start if persistent synovitis > 6 weeks
- Slow onset of action (may take weeks to months)
Hydroxychloroquine Mild disease S/E = maculopathy, rash, N/V/D, ototoxicity,
aggravates psoriasis

Sulphaslazine Moderate disease S/E = N/V/D, rash, BM depression, oral ulcers,


SJS
 Monitor LFT & FBC
Methotrexate 1st  choice for severe CI = pregnancy, liver disease, G6PD deficiency
disease Do not take with alcohol!!
S/E = N/V/D, lower lobe pulmonary fibrosis,
-Better tolerated
53
transaminitis, ↑no. of rheumatoid nodules
 Give with folate to reduce GI S/E
 Monitor LFT
Lefluonomide Inhibits activated T Takes months to work
cells S/E = BM depression, proteinuria, rash,
hepatitis
 Monitor LFT, FBC, urinalysis
Corticosteroids Indications S/E = metabolic, cosmetic, cataracts,
osteoporosis
- vasculitis  Monitor BSL & BP
- severe disease
Rebound disease common on stopping steroids
- exacerbations not
responding to other
drugs
Azathioprine, Cyclosporin A, Severe disease with AZP = BM depression, transaminitis, oncogenic
cyclophosphamide failure of other CSP = gingival hypertrophy, HPT, renal
therapies impairment

Anti-cytokine therapy
-suppress disease activity only during treatment → relapse on discontinuation

Infliximab (against TNF) Progressive RA after 2 S/E = N/V/D, rash, infection (TB reactivation),
Etanercept (against TNF receptor) DMARDs failure neutralizing antibodies

Surgery To improve function, relieve pain and prevent complications

- Synovectomy & decompression of wrist & tendon sheaths


- tendon repair and transfer
- Arthrodesis
- Osteotomy
- Joint replacement
Paramedical services Physiotherapy
Occupational therapy = adaptive aids, orthoses (eg wrist splints), ADL
training

Others Patient education and Support groups

54
*drugs causing cytopenias = warn patient to stop meds and consult doctor if sore throat develops.
 Clinical course
 Variable
- most have fluctuating disease with the greatest progression during the initial 4-5 years
 Most develop deforming and destructive arthritis after 15-20 years
 Life expectancy reduced by 3-7 years
 Poor prognostic factors
1. Female
2. Older age of onset (>60 YO)
3. Systemic features: LoW, extra-articular manifestations
4. Vasculitis
5. Early bone erosions
6. Rheumatoid nodules
7. Persistent disease activity > 12 months
8. Insidious onset
9. HLA-DR 4 linkage
10. Rh factor > 1 in 512

55
Medicine (Rheumatology) = Examination of rheumatoid hands
Approach to RA short case
 Introduce yourself
 Sit patient at edge of bed, remove accessories, roll up sleeves and place hands on pillow (watch action)
 General inspection – Cushingoid appearance
 Hands
1. LOOK
‐ Palms down
 Deformities = symmetrical polyarthropathy involving small joints of the hand (sparing DIPJ)
Z-deformity of thumb, Swan neck and Boutonniere deformity of fingers
Ulnar deviation of fingers
Volar subluxation of MCPJ
Radial deviation of wrist
Dorsal subluxation of ulna at carpal joint → prominent radial styloid process
 Swelling = Rheumatoid nodules over extensor surfaces (never on IPJ)
 Discoloration = Erythema (active disease)
 Wasting of intrinsic muscles (guttering)
 Nails = Telangiectasia, nail fold infarcts, splinter hemorrhages, nail bed pallor, longitudinal ridging, thickening, pitting, onycholysis (rule out
psoriasis)
‐ Palms up
 Wasting of thenar and hypothenar eminences
 Palmar erythema
 Carpal tunnel release scar (over distal palmar crease)
2. FEEL
‐ Increased warmth (run back of hand across patient’s dorsum)
‐Wrist = Tenderness (suggestive of synovitis → active disease)
Joint effusion (soft and boggy → synovitis)
Synovial thickening
Piano key sign = springs back into position when pressed
‐ MCPJ = Tenderness
Joint effusion (bulge sign)
Subluxation
‐ PIPJ = Tenderness
Joint effusion
3. MOVE
56
‐ Clench fists tightly and release = trigger finger
‐ Place palms on pillow and lift fingers off = dropped finger (tendon rupture/slipped off into gutter)
‐ ‘push against wall’ position = finger drop (PIN palsy due to inflammation around wrist)
‐ Wrist flexion and extension = limited ROM
‐ Fold arms across chest = subcutaneous nodules over elbows, psoriatic skin plaques
4. NEUROLOGICAL
‐ Radial nerve = EPL, sensation over 1st  dorsal web space
‐ Median nerve = FPL, FDP of index finger, APB sensation over lateral palm and 3½ fingers, Tinel’s sign (CTS)
‐ Ulnar nerve = FDP of little finger, finger abduction, sensation over medial palm and 1½ finger
5. FUNCTION
‐ Power grip
‐ Unbutton clothes
‐ Write
‐ Hold a cup of water
6. REQUEST
‐ Feet for similar changes
‐ TMJ for crepitus
‐ Neck for tenderness
‐ Eyes = Episcleritis, scleritis
‐ Lymphadenopathy
‐ Lungs = Pleural effusion, end-inspiratory fine crepitations (pulmonary fibrosis), nodules
‐ CVS = aortic regurgitation
‐ Request to look at temperature chart, offer to take BP and perform urine dipstick test
Format for presentation
a) Describe deformities
b) Disease activity → active or quiescent
c) Functional status
d) Request to examine
e) Diagnosis and differentials

57
Renal Medicine
Medicine (Renal) = Nephrotic Syndrome History Taking
Name/Age/Race/Gender/Occupation
Past Medical History
Date of admission

Presenting Complaint
1. Lower limb oedema
 When did it start?
 Bilateral/unilateral?
 Getting progressively worse?
 Worse in the evening? Better in the morning?

2. Associated with
 Abdominal distension? Can clothes still fit?
 Increase in weight?
 SOB? Exertional dyspnoea/Orthopnea/Paroxysmal nocturnal dyspnoea?
 Periorbital/Facial oedema? (esp so in the morning)

3. Aetiology
 Renal
o Frothy urine, oliguria, concentrated urine (signs of proteinuria)
o Haematuria (Nephritic syndrome)
o Fever, URTI symptoms (trigger, post infectious glomerulonephritis)
o Diarrhoea (IgA nephropathy)
o History of Hepatitis B/C infection
o Recent drug intake
o Joint pain, rashes (autoimmune)
o Polyuria, polydipsia, polyphagia, LOW (DM)
 CVS
o Chest pain, SOB, palpitations, giddiness/syncope, diaphoresis, nausea/vomiting
 GIT
o LOA, LOW, lethargy, jaundice, pruritus, easy bruisability (chronic liver disease)
o Mucoid/bloody stools, alternating constipation and diarrhoea (inflammatory bowel disease)
4. Complications
 Spontaneous bacterial peritonitis (fever, abdominal pain)
 Hypovolemia (abdominal pain, vomiting, dizziness)

5. Management prior and during admission


6. Is this the first time that this happened? Describe prior episodes.

History of presenting complaint


1. When was nephrotic syndrome diagnosed?
 Presenting complaint
 Investigations done (renal ultrasound, renal biopsy)
 Cause of nephrotic syndrome (if biopsy was done  likely glomerulonephritis)
 Followed up with whom? Frequency of follow up? Compliance to follow up? Investigations done at
every follow up? Annual investigations?
 Medications?
o Steroids, cyclophosphamide, chlorambucil, levamisole, cyclosporine A
o Compliance with medications?
o Side effects: obesity, hypertension, cataracts, osteoporosis, increased susceptibility to infections,
cosmetic changes, gastritis, diabetes
 Fluid and dietary restrictions
o Fluids: as desired
o Diet: no refined sugars, no fat (if patient is on steroids), less protein
 Level of control
o Number of relapses? Number of hospitalisations?
o For each relapse  Presentation? Triggers? Treatment?
o When was the last episode?
 Monitoring
o How often?
o Records in nephrotic diary?
o Do you know what to do when proteinuria is found?
o Indications for admission?

3. Complications
 Hypovolemia (abdominal pain, vomiting, giddiness)
 Acute renal failure
 Thromboembolism
o Was any blood clot found?
o Treatment with heparin/warfarin only if symptomatic or immobile
 Increased susceptibility to infections
 Spontaneous bacterial peritonitis
o History of abdominal pain of fever treatment?
o Pneumococcal vaccinations?
o Prophylactic antibiotics?
 Hyperlipidemia
o On statins?

Past Medical History


1. Other medical problems
2. Previous hospitalisations
3. Previous surgeries

Drug History
1. Drug allergies

Social History
1. Smoking
2. Alcohol drinking
3. Family set-up? Main caregiver?
4. Finances
5. Have to miss a lot of work?
Medicine (Renal) = Nephrotic Syndrome
*in an oedematous patient  always test for proteinuria and investigate for nephrotic syndrome if
albustick >= 2+
Definition
1. Clinical entity
2. Characterized by classical triad
 Proteinuria (> 3g/1.73m3/day)
 Hypoalbuminemia (<30g/L)
 Oedema
3. Usually associated with hyperlipidemia and lipiduria
4. Hypertension, haematuria and azotemia are rare (characteristic of nephritic syndrome)

 Aetiology
1. Primary glomerulonephritis (usually non-proliferative glomerulonephritis)
 Minimal change disease
 Focal segmental glomerulosclerosis
 Membranous glomerulonephritis
 Membranoproliferative glomerulonephritis

2. Secondary glomerulonephritis
 Vascular (Henoch-Schönlein Purpura)
 Infective (hepatitis B/C, malaria, HIV, post streptococcal)
 Drugs (captopril, TCM, NSAIDS, gold, penicillamine)
 Autoimmune (SLE)
 Metabolic (diabetes)
 Infiltrative (Amyloidosis)
 Neoplasia (multiple myeloma, lymphoma)

Causes
1. Children
 Minimal change disease (80%)
2. Adults
 Minimal change disease (30%)
 Focal global sclerosis (21%)
 Mesangial proliferative glomerulonephritis (25%)
 Membranous glomerulonephritis (12%)
 Focal segmental glomerulosclerosis

Pathogenesis
1. Derangement in glomerular capillary walls  proteinuria  hypoalbuminemia
2. Loss of oncotic pressure  generalised oedema
3. Drop in plasma volume  diminished glomerular filtration rate  compensatory rise in aldosterone 
promotes retention of salt and water by kidneys  further aggravates oedema

Clinical signs
1. Oedema (periorbital, facial, lower limb, genitalia, sacral)
2. Pleural effusion
3. Ascites
4. Xanthelasma
5. Leuchonychia
Complications
1. Hypovolemia
 Presents with abdominal pain, vomiting and giddiness
 Pathogenesis: third space loss results in insufficient blood volume in vessels to maintain adequate
blood pressure  leads to peripheral vasoconstriction and urinary Na+ retention
 Indicators: decreased urinary Na+, increased hematocrit/urea/creatinine
 Management: IV 20% albumin

2. Acute renal failure (as a result of hypovolemia)


3. Thromboembolism
 Occurs in 10-40% of patients
 Pathogenesis
o Urinary loss of anti-thrombin III
o Increased synthesis of clotting factors and fibrinogen
o Increased hematocrit and hence increased viscosity
 Presents as deep vein thrombosis, pulmonary embolism, renal vein thrombosis, saggital sinus
thrombosis
 Management: prophylactic warfarin/heparin if patient is immobile

Renal Vein Thrombosis


 More common in membranous glomerulonephritis than other forms (6-8%)
 Clinical features (usually asymptomatic): loin pain, hematuria, proteinuria, bllotable kidneys,
renal impairment
 35% have concomitant pulmonary embolism
 Diagnosed by Doppler ultrasound, renal angiography, spiral CT, MRI
 Treatment by anticoagulants with warfarin for 3-6months

4. Increased susceptibility to infections


 Pathogenesis
o Urinary loss of immunoglobulins
o T cell abnormalities
o Use of immunosuppresants
 Presents as peritonitis (causative agent is Streptococcus pneumonia), urinary tract infection,
septicaemia
 Management
o Pneumococcal vaccination
o Prophylactic penicillin

5. Hyperlipidemia
 Pathogenesis
o Lipoprotein synthesis triggered by hypoalbuminemia
o Abnormal transport of circulating lipid particles
o Impaired breakdown of lipoproteins
 Usually improves with resolution of nephrotic syndrome
 Increases the risk of ischemic heart disease and arthrosclerosis
 Management
o Statins (if prolonged)

6. Negative nitrogen balance


o LOA

Investigations
1. Confirm diagnosis of nephrotic syndrome
 Urine dipstick
o Proteinuria
o Haematuria
o Glycosuria
 UFEME and urine cultures to rule out urinary tract infection as a cause of proteinuria
 Liver function test to check for hypoalbuminemia
 Fasting lipid panel to check for hyperlipidemia

2. Rule out other causes


 ECG, cardiac enzymes, beta natriuretic peptide, Chest X-ray to rule out congestive cardiac failure
 Urea/Electrolytes/Creatinine to rule out renal impairment and test for severity of hypovolemia
(increased urea and creatinine)
 Liver function test to rule out deranged liver function
3. Aetiology
 Full blood count
o Anaemia, lymphopenia, thrombocytopenia  SLE
o Increased hematocrit  Severe hypovolemia
 Erythrocyte sedimentation rate, C-reactive protein  Underlying inflammatory condition
 Anti-nuclear antibody (ANA), anti-dsDNA, C3, C4  SLE
 Anti Neutrophil Cytoplasmic Antibody (ANCA), anti-glomerular basement membrane (anti-GBM) 
vasculitis
 Fasting glucose, HbA1c  Diabetes
 Hepatitis B/C serology: if Hepatitis B/C is the cause of nephrotic syndrome, there is a need to know
before starting steroids in view of decompensation
 Anti-streptolysin O titres  Post-streptococcal glomerulonephritis
 Renal ultrasound: need to know anatomy before doing biopsy
 Renal biopsy

Indications for Renal Biopsy


 Atypical features: gross hematuria, hypertension, renal impairment, persistently low serum
complement, poorly selective proteinuria, proteinuria > 1g/day
 Family history of glomerulonephritis
 Steroid resistance
 Steroid dependantpatient with unacceptable steroid toxicity

 Acute management
1. Fluid restriction to < 1L/day
2. Low salt and low protein diet
3. Place on I/O charting
4. Daily weights and albustick
5. Monitor vitals Q4hourly (inform if systolic BP is <100mmHg or Hr >100/min)
6. Symptomatic treatment
 IV Lasix (furosemide) and PO Span K (aim for 1kg loss/day) with/without spironolactone (K+ sparing
diuretic) and IV 20% albumin  symptomatic treatment for hypotension
 Thromboembolic deterrent (TED) stockings, anticoagulants  prevent thromboembolic event
o Consider ambulatory problems/immobility
o Consider risk factors for deep vein thrombosis (DVT)
o Consider severe proteinuria with low albumin

Chronic management
1. Immunosuppression
2. PO furosemide with Span K and low salt diet only if oedematous
3. Monitoring at home with albustick and educate patient on how to escalate therapy and when to admit
4. Prevention of infections
 Pneumococcal vaccination
 Prophylactic antibiotics
 Prompt treatment of infections
 NO LIVE ATTENUATED VACCINES (especially if on steroids)

Immunosuppresants
Immunosuppressive therapy is used for minimal change disease
1. Corticosteroids
 High dose prednisolone (1mg/kg/day)
 80% remission rate achieved by 16 weeks
 Regime
o High dose prednisolone continued for 1 week after remission is achieved
o Taper dose over 6 months, and subsequently discontinue
o Can give alternate day prednisolone during tapering to mi nimise side effects
 Complications
o Cosmetic changes: moon-like facies, hirsutism, acne, central obesity, buffalo hump,
supraclavicular fat pads
o Metabolic: obesity, diabetes, hypertension
o Endocrine: menstrual irregularities, Addisonian crisis, osteoporosis
o Musculoskeletal: proximal myopathy, aseptic necrosis
o Posterior subcapsular cataracts
o Gastritis/Peptic ulcer disease (PUD)
o Increased catabolism: thin skin, easy bruising, abdominal striae
o Increased susceptibility to infections especially opportunistic ones
o Steroid psychosis

2. Alkylating agents

 Cyclosporine A, cyclophosphamide
o Indicated in frequently relapsing, steroid dependant nephrotic syndrome (clinically significant
cataracts, difficult hypertension, diabetes, and disabling emotional disorders due to cosmetics
appearance)
3. Mycophenolate mofetil
4. Tacrolimus
Medicine (Renal) = Secondary Hypertension
 Indications for screening
o Age of onset <40years old or >55 years old
o Severe or refractory hypertension
o Sudden rise in BP over a previously stable value
 Renal artery stenosis
o Most correctable cause of secondary hypertension
o Presentation
 Patients <30yo with no family history/risk factors
 Patients >55yo presenting with severe hypertension
 Refractory or resistant hypertension (compliant to full dosages of an appropriate 3-drug
regimen including a diuretic)
 Hypertensive emergency
 Acute elevation in plasma Cr after use of ACE-I or ARB
 Unilateral abdominal bruit
o Causes
 Atheroma (elderly male smokers)
 Fibromuscular dysplasia (young females)
o Management
 Balloon angioplasty
 Conn’s syndrome (primary hyperaldosteronism)
o Primary hyperaldosteronism
 Excess pdn of aldosterone independent of RAA system
 Conn’s syndrome (aldosterone -secreting adenoma)
 Primary adrenocortical hyperplasia
 Adrenal carcinoma (rare)
o Secondary hyperaldosteronism
 Decreased renal perfusion (RAS, coarctation of aorta)
 Pregnancy (estrogen-induced increase in rennin)
 Arterial hypovolemia and oedema
o Clinical presentation
 Suspect conn’s syndrome in hypertensive patients with
 Hypokalemia
 Refractory hypertension
 Severe hypertension before 40yo (esp in females)
o Investigations
 U/E/Cr = hypokalemia, hypernatremia
 Plasma rennin and aldosterone = raised aldosterone with low rennin levels
 CT A/P
o Management
 Conn’s syndrome = surgery with pre-op spironolactone
 Hyperplasia = spironolactone/amiloride
 Pheochromocytoma
o Composed of chromaffin cells found in adrenal medulla which synthesize and release
catecholamines
o Rule of 10s
 10% rise in association with several familial syndromes (MEN 2, NF 1, von Hippel-
Lindau syndrome)
 10% are extra-adrenal
 10% are bilateral
 10% are biologically malignant
o Clinical presentation
 Abrupt onset of hypertension  hypertensive emergency
 Symptoms
 Episodic headaches, palpitations, diaphoresis, postural giddiness
 Ascites
1. Place left middle finger on umbilicus and start persussing to the other end (should be resonant out to the t he
flanks); dullness in the flanks means that there is at least 2L of ascitic fluid
2. If there is dullness before the flanks, ask patient to lie on right lateral position and wait for 30-60s for fluid
equilibration. Percuss form site of dullness back towards midline. If site of dullness becomes resonant in
this position, there is ascites. Percuss for new position of dullness (fluid level in this position)
3. Further test for ascites: ask patient to place right hand vertically in midline of abdomen, then place your
hand on left abdomen and flick fingers on right abdomen (can feel fluid thrill if there is ascites)
4. In this position, palpate for spleen
5. Check for sacral edema and scars (bone marrow biopsy → ? myeloproliferative disease)

Kidneys
1. Bimanual palpation; place left hand under patient and right hand on the abdomen and push left hand
upwards twice during inspiration (do 2 time on each side)
a. If kidney enlarge, the righ hand will feel something hitting it
b. Enlarged kidney bulges forwards; perinephric abscess bulges backwards; transplanted kidneys
palpable in either iliac fossa

 Ausculatation
1. Listen for bowel sounds (tinkling and hyper active = IO; absent over 3min period = paralytic ileus)
2. Listen for renal bruits
3. Hepatic arterial bruit = alcoholic hepatitis, HCC, liver mets
4. Abdominal venous hum = portal hypertension

Groin
1. Palpate for enlarge inguinal lymph nodes
2. Ask patient to cough to detect inguinal hernias
3. Inspect for testicular atrophy (CLD)

Legs
1. Look for bruising, scratch marks and edema (press thumb against the back of malleolus, look at patient’s
face for pain)
2. Inspect toe nails for clubbing, cyanosis, pallor, leukonychia

End
1. Sit the patient up
a. Hepatic asterixis for 15s (hepatic encephalopathy)
encephalopathy)
b. Cervical lymphadenopathy
lymphadenopathy
c. Parotid/ submandibular gland enlargement)
2. Tell the examiners that you would like to complete the examination by doing a PR exam, taking blood
pressure and temperature. If patient has hepatomegaly,
h epatomegaly, should examine JVP. If ascites/ pedal edema
present, check for pleural effusion
3. Thank patient for his help and dress him up properly

Template for presentation


On general inspection, the patient appears to be alert, comfortable, orientated and well at rest. There are no
signs of respiratory distress and the patient does not appear to be in any pain. The vital signs are stable (HR =...,
RR= ..., afebrile)

Examination of the peripheries did not show any signs of jaundice, pallor cyanosis, dehydration or stigmata of
chronic liver disease such as clubbing, leuconychia, palmar erythema, spider naevi, gynaecomastia etc.

Inspection of the abdomen did not reveal any surgical scars, abdominal distension, distended vein or any
visible masses or pulsations. The abdomen was symmetrical and moved well with respiration. On superficial
palplation, the abdomen was soft and non-tender with no guarding or rigidity of abdominal wall muscles. The
the mid-clavicular line. The surface of the liver was smooth with no nodules felt. It was non pulsatile and no
bruits were heard over the liver. The spleen and the kidneys were non-palpable. No ascites was detected. Bowel
sounds were normal and no renal bruits were heard.

Inguinal lympadenopathy and cough impulses suggestive of inguinal hernia were not detected in the groin.
Lower limb and sacral edema were absent. There was no hepatic flap.

In summary, the patient has features of _________ as evidenced by _________


Medicine (GIT) = Issues for discussion
1. Signs of chronic liver disease
a. Hands = clubbing, leuconychia, palmar erythema, asterixis, bruising, petechiae
b. Face= jaundice, fetor hepaticus
c. Chest = spider naevi, gyanecomastia, loss of axillary hair, wasting of pectoral muscles
d. Abdomen = portal hypertension (ascites, caput medusa, splenomegaly), sacral edema
e. Groin = testicular atrophy
f. Legs= edema

2. Peutz-Jeghers syndrome
a. Autosomal dominant condition
b. Features
i. Freckle like spots (discrete brown-black lesions) around the mouth, buccal mucosa, fingers,
toes
ii. Harmatomas of the small bowel(50%) and colon(30%) →can present with bleeding +
interssusception, increased incidence of GI adenocarcinoma

3. Rendu-Osler-Weber syndrome
a. Autosomal dominant condition
b. Multiple small tenlangiectasiae present on lips, tongue, and skin
c. GI features = chronic blood loss, torrential bleeding, liver AV malformation

4. Hepatic flap
a. Refers to jerky, irregular flexion-extension movement at the wrist and MCP joints often
accompanied by lateral movements of the fingers
b. Mechanism = interference with the inflow of joint position sense information to hte reticular
formation in brainstem resulting in rhythmical lapses of postural muscle tone
c. Characteristics = usually bilateral, absent at rest, brought by sustanined posture, not synchronous
on each side, absent when coma suepervenes
d. Characteristic but not diagnostic of liver failure (can also occur in cardiac, respiratory, and renal
failure; also in metabolic encephalopathy – hypoglycaemia, hypokalemia, hypomagnesaemia,
barbiturate intoxication)

5. Spider naevi
a. Consist of a central arteriole form which radiate numerous small vessels
b. Usual distribution is the area drained by SVC→ found on the arms, neck and chest wall
c. Pressure applied with a pointed object to the central arteriole causes blanching of the whole lesion
with rapid refilling from the centre to the periphery on release of pressure
d. Differentials:
i. Campbell de Morgan spots (flat/ slightly elevated red circular lesions which occur on the
abdomen or chest wall; do not blanch on pressure)
ii. Venours stars (due to elevated venous pressure; found overlying main tributary to a large
vein; occur on dorsum of feet, legs, back and lower chest; not obliterated by pressure; blood
flow from periphery to centre of lesion)
iii. Hereditary haemorrhagic telangiectasia

6. Troisier’s sign = presence of a large left supraclavicular lymph node with gastric carcinoma

7. Causes of abdominal distention (6Fs)

a. Fat = umbilicus buried in fat


b. Fluid = shifting dullness, fluid thrill, eversion of umbilicus, tense abdominal wall and flanks
c. Faeces = mass in the left lower quadreant, indentable, non-tender
d. Flatus
e. Fetus = umbilicus pushed upwards
8. Sister Joseph nodule = metastatic tumor deposit in the umbilicus (antatomical region where the peritoneum
is closest to the skin)
9. Cullen’s sign = blue black discolouration of umbilicus (extensive haemoperitoneum, acute pancreatitis)

10. Liver
a. Normal liver span = <13 cm as measure in the mid-clavicular line
b. Pulsatile liver = tricuspid regurgitation, HCC
c. Tender liver = hepatitis, rapid liver enlargement, hepatic abscess, cholangitis

11. Gallbladder
a. Courvosier’s law= if the gallbladder is enlarged and the patient is jaundiced, unlikely to be
gallstones; gallbladder with stones is usually chronically fibrosed (small)

Gallbladder enlargement
With jaundice
1. Carcinoma of head of pancreas
2. Carcinoma of ampulla of Vater
3. Gallstones in CBD
4. Carcinoma of the gallbladder
Without jaundice
1. Mucocele/ empyema of gallbladder
2. Carcinoma of the gallbladder
3. Acute cholecysitits

12. Splenomegaly

Causes of splenomegaly
Vascular Portal hypertension
Infective Viral hepatitis , EBV, CMV
Bacterial (SBE)
Protozoal (malaria, kala-azar)
Trauma Haematoma
Autoimmune SLE
RA (felty’s syndrome)
Metabolic Storage disorders (Gaucher, Neimann-Pick, glycogen storage, lipid storage)
Infiltrative Amyloidosis
Sarcodosis
Neoplastic CML (invasive)
Myelofibrosis (massive)
Lymphoma
Lymphoproliferative disorders
Polycythemia ruba vera (massive)
Haematological Chronic haemolytic anemia, (spherocytosis, G6PD deficiency, thalassaemia)

13. Causes of hepatosplenomegaly


a. Chronic liver disease with portal hypertension
b. Infective = acute viral hepatitis, infectious mononucleosis, CMV, malaria
c. Autoimmune = SLE, RA
d. Metabolic = storage disorders
e. Infiltrative= amyloidosis, sarcoidosis
f. Neoplastic =myeloproliferative disorders, lymphoma, lymphoproliferative disorders
g. Haematological = Chronic haemolytic anemia, (spherocytosis, G6PD deficiency, thalassaemia)

14. Distinguishing features between a large left kidney and splenomegaly


a. Spleen descends inferomedially on inspiration while kidney descends downwards
c. Dull percussion note over spleen but resonant over kidney due to gas filled fowel loops
d. Kidney is ballotable but not the spleen
e. Cannot get above the spleen (can get above the kidney)
f. Friction rub may be heard over the spleen but never over the kidney (too posterior)

15. Succession splash


a. May be present in gastric outlet obstruction
b. Procedure
i. Explain to patien what is going to happen
ii. Grasp one iliac crest with each hand
iii. Place stethoscope close to epigastrium
iv. Shake patient from side to side
c. Ensure that the patient has not ingeted any fluids just prior to examination (at least 3 horus before)

16. Anterior abdominal wall masses


 Ask pateitn to fold arms across the upper chest and sit halfway up
o Intraabominal mass will disappear/ decrease in size
o Anterior abdominal mass will become more prominent/ remain unchanged
 Lipoma
 Sebaceous cyst
 Dermal fibroma
 Malignant deposits = melanoma, carcinoma
 Epigastric hernia
 Umbilical hernia
 Incisional hernia
 Rectus sheath divarication/ haematoma

17. Per-rectal examination


a. Explain to the patient the purpose of the examination and the procedure
b. Relax the patient and lie him on the left lateral side with the legs drawn up
c. Put on gloves and always use lubrication
d. Approach the rectum from the inferior aspect

 INSPECT THE PERIANAL AREA


o Skin tags,
o Protruding polyps
o Haemorrhoids
o Anal fissures (most often seen at 6 & 12 o’clock; often accompanined by sentinel tags)
 ASSESS ANAL WINK (anocutaneous reflex) = contraction of the anus on stroking the perianal region
 INSERT INDEX FINGER
o Assess anal tone: tight → anal stenosis ; loose → lower spinal lesion
o Ask patient to squeeze your finger
o Rectal masses
o Prostate enlargement
o Local tenderness e.g. retrocaecal appendix
 ON WITHDRWAL
o Fresh PR bleed or melena
Medicine (GIT) = Approach to ascites
Ascites is the effusion and accumulation of serous fluid in the abdominal cavity.
Symptoms and Differential Diagnoses of Ascites
1. Causes of a distended abdomen:
 Fat, faeces, fluid, flatus, fetus, filthy big tumour
2. Causes of ascites
 Chronic Liver Disease/ Cirrhosis (Commonest Cause)
 Chronic alcoholism
 Viral hepatitis
 Cardiac Failure
 Chronic Renal Failure
 Nephrogenic ascites secondary to dialysis
 Nephrotic syndrome
 Enlarged Lymph Nodes
 Primary and Metastatic
 Intra abdominal mass
 Malignancy : Primary and Metastatic
 Others :
 Tuberculosis
 SLE
 Pancreatitis
 Constrictive pericarditis

History
1. Past medical history : To identify the system responsible for the ascites
Any past medical history of any disorder like coronary artery disease, hypertension, alcohol abuse. Is
the patient on any drug that can cause cardiac, hepatic or renal disease? Does the patient have renal
failure or go for dialysis. If suspicious, a history of HIV and TB should be obtained.
2. Ascites: Alcohol history, Hepatitis B status, any intra-abdominal masses and their associated symptoms.
3. Past medical history : Hepatitis Vaccinations, any recent drugs used
4. Any associated early satiety and shortness of breath

Physical Examination
General Appearance
 Does the patient have any stigmata of chronic liver disease?
 Is the patient on oxygen?
 Does the appear to be in any respiratory disease?

Vital Signs
 Any tachycardia
 Respiratory rate for tachypnea
 Blood pressure measurement for hypertension

CVS Examination
 Checking for raised JVP will provide great yield here as it would indicate heart failure
 Other signs to pick up will include displaced apex beat, gallop rhythm, bibasal crepitations and any
possible aetiology for heart failure like valvular heart disease

Abdominal Examination
 Ask for site of most intense pain first. Palpate for presence or organomegaly. The live in alcoholic
cirrhosis is unlikely to be enlarged. However, other stigmata of chronic liver disease can be sought for
like caput medusae.
 Percuss for ascites and shifting dullness
 Palpate for any suspicious intra abdominal masses
 Palpate for hepatosplenomegaly in portal hypertension

Lower Limbs
 Check for presence of pitting edema
Medicine (GIT) = Ascites
Introduction
 Pathological accumulation of fluid in the peritoneal cavity = clinically detectable when > 500mls
 Pathogenesis
 Under filling theory = inappropriate fluid sequestration within splancnic vascular bed secondary
to portal hypertension > decreased intravascular volume > kidneys retain more Na+ and water by
activating RAA system
 Overflow theory = primary renal retention of Na+ and water
 Complications
a) Peritonitis
b) Dyspnea secondary to splinting of diaphragm
c) Pre-renal failure secondary to intravascular volume depletion
d) Early satiety

 Aetiology
Transudate vs Exudate

Transudative Exudative
Cardiovascular Infection
 Congestive cardiac failure  TB peritonitis
 Right heart failure
 Constrictive pericarditis
 IVC obstruction
 Portal/hepatic vein obstruction
Renal Inflammation
 Acute renal failure  Pancreatitis
 Chronic renal failure
 End stage renal failure
 Nephritic syndrome
 Nephrotic syndrome
GI Intra-abdominal malignancy
 Chronic liver disease  Pancreatic/gastric/colonic ca
 Malnutrition  Ovarian ca
 Protein-losing enteropathy  Metastasis to liver
 Metastasis to peritonium

Generalised vs Localised

Generalised Localised
Cardiovascular Vascular
 Congestive cardiac failure  Portal HPT
 Right heart failure a) IVC obstruction
 Constrictive pericarditis b) Budd-chiari syndrome
c) veno-occlusive disease
d) Liver cirrhosis
e) Portal/splenic vein obstruction
Renal Infection
 Acute renal failure  TB peritonitis
 Chronic renal failure
 End stage renal failure
 Nephritic syndrome
 Nephrotic syndrome
GI Inflammation
 Malnutrition
 Protein-losing enteropathy
Intra-abdominal malignancy
 Pancreatic/gastric/colonic ca
 Ovarian ca
 Metastasis to liver
 Metastasis to peritonium

History
Name/age/race/gender/occupation
Drug allergy
Past medical history
Date of admission
Presenting complaint
Symptoms
1. Abdominal distension
 duration
 acute/gradual
 quantity = how many inches? , weight gain
 associated with LL edema, SOB (exertional, orthopnea, PND), faciel edema
 fever and abdominal pain and diarrhea > SBP

Aetiology
1. CVS
 History of heart disease
 Chest pain, sob, diaphoresis
2. Renal
 Urine output (oliguria, appears concentrated)
 Hematuria and frothy urine
3. GIT
 Chronic bloody diarrhea
 LOA, LOW
 History of liver disease – jaundice, easy bruising, pruritis changes in uring and stool, fatigue
4. Infection
 History of TB
5. Inflammation
 Acute epigasttric pain radiating to the back
6. Malignancy
 LOA, LOW, fever, fatigue, abdominal pain, jaundice, recent changes in bowel habits,
haematochezia, melena, irregular menstrual bleeding

Underlying etiology for liver cirrhosis


Triggers for present episode
1. BGIT
2. Sepsis
3. Recent drug intake
4. Recent surgery/trauma

Complications (usually of CLD and cirrhosis)


1. Bleeding varices = hematemesis, haematochezia, melena
2. Encephalopathy = lethargy, drowsiness, confusion, personality changes
Systemic review
Management prior and during admission
Has this happened before? Describe prior experiences
Differentials
 Fat
 Fetus
 Flatus
 Faeces
 Fluid
 Filthy big tumour

Investigations
ECG, Cardiac enzymes, CXR = CCF
Urine
 Urine dipstick = proteinuria, hematuria
 UFEME
 Urine phase contrast microscopy
 Urine PCR or 24hr UTP

Bloods
 FBC = WBC ↑ (SBP), HCT (hypovolemia)
 U/E/Cr = renal impairment
 LFT = liver impairment, albumin
 ESR, CRP

Imaging
 U/S Hepatobiliary system (HBS)
 CTAP

Microbiology
 Abdominal paracentesis
 Both diagnostic and therapeutic
 Clinical parameters
 Appearance (straw coloured; turbid = pyogenic, TB; bloody = malignant, TB; chylous =
pancreatitis)
 Clinical chemistry (cell count and differential; protein; albumin; glucose; amylase)
 Gram stain, microscopy, c/s, AFB smear, TB c/s
 Fluid cytology
 Serum-ascitic albumin gradient = serum albumin – ascitic albumin
 Correlates directly with portal pressure
 Transudate = gradient > 1.2 g/dl
 Excudate = gradient < 1.2 g/dl
 May be associated with a right pleural effusion via trans-diaphragmmatic lymphatics =
subpulmonic effusion
- Management = fluid restriction, strict I/O charting, vitals monitoring, IV albumin 20% with
diuretics

Management
Non-pharmacological
 Fluid restriction (1 L/day)
Low salt diet
 Strict I/O charing and daily weights
 Monitor vitals 4hourly – inform doctor if SBP < 100mmHg or HR > 100 bpm

Pharmacological
 IV diuretic therapy = frusemide  spironolactone
 Albumin 20% only if patient is hypotensive – to bring back fluid from 3rd space

Watch out for


a) Hypokalemia
 diuretic therapy
 abdominal paracentensis ( >2-3 L a day may cause hypovolemia and hypohalemia)
 may also precipitate hepactic encephalopathy
b) dehydration
 over vigorous dieresis
 negative fluid balance not more than 1L a day

Diuretic-resistant ascites
 therapeutic abdominal paracentesis with IV albumin 20%
 TIPPS
 Liver transplant

Approach to ascites
 Abdominal masses
 Epigastric mass (gastric ca)
 RIF mass (ovarian ca, cecal ca)
 LIF mass (desceding colonic ca, sigmoid ca)
 If patient is jaundiced
a) Signs of CLD – liver cirrhosis Cx portal HPT
b) Minimal signs of CLD + smooth tender hepatomegaly = budd-chiari syndrome
+ craggy liver = intra-abdominal malignancy with liver/peritoneal mets
 Feel for supraclavicular LAD
 If patient is not jaundiced and no signs of CLD
a) Leuconychia – nephritic syndrome
b) Raised JVP – constrictive pericarditis or right heart failure
 If all negative
a) TB peritonitis = chest examination
b) Carcinomatosis peritoneii = paracentesis + FNAC to see malignant cells
Medicine (GIT) = Chronic liver disease and Liver cirrhosis
Chronic liver disease
 Liver disease persisting >6 months based on LFT and histology

Liver cirrhosis
Strict criteria
a. Diffuse fibrosis
o Occurs in portal tracts, central veins and space of Disse
o Inflammation stimulates stellate cells in space of Disse  transforms into myofibroblasts
o Extension of fibrosis from space of Disse to other parts of lobule causes sinusoids to separate
from hepatocytes
o Venulization = sinusoids converted from fenestrated endothelial channels with free exchange of
solutes to high pressures and fast-flowing channels without such exchange
o Shunting of blood directly from portan vein to central vein = no detoxification of metabolites
hepatocytes derived of nutrients
b. Nodule formation = consisting of regenerating hepatocytes
c. Disruption of tissue architecture = bridging fibrosis and shunt formation

Results in subdivision of liver into nodules of regenerating hepatocytes surrounded by scar tissue
Aetiology
 Vascular
o Tricuspid regurgitation/right heart failure (cardiac cirrhosis)
o Veno-occlusive disease
o Budd Chiari syndrome
 Infective
o Hep B and C infection
 Toxin
o Chronic alcoholism
o Drugs (methotrexate, amiodarone)
o alfatoxin
 Autoimmune
o Autoimmune hepatitis
o Primary biliary cirrhosis
o Primary sclerosing cholangitis
o Secondary biliary cirrhosis (RPC or chronic CBD stones)
 Metabolic
o Wilson’s disease
o Secondary haemochromatosis (DO NOT mention HH as gene not found locally)
o Alpha1 antitrypsin deficiency
 Cryptogenic

History
Aetiology
 Vascular
o History of heart failure
 Infective
o Personal history of Hep B/C infection
o History of Hep B vaccination
o Maternal history of Hep B infection
o History of blood transfusion, IVDA, CSW contact, tattooing
 Toxin
o History of chronic alcoholism
o History of cytotoxic drug ingestion
 Autoimmune
o History of rash, joint pain and swelling
o Family history of liver disease

Complications
 Hypoalbuminaemia
o Lover limb edema
o Abdominal distension +abdominal pain/fever (SBP)
 Bilirubin
o Jaundice
o pruritus
 Coagulopathy
o Easy bruisability
o Petechiae  may present like ITP
o Mucocutaneous bleeding
o menorrhagia
 Bleeding varices - Haematemesis, haematochezia, melena
 Encephalopathy - lethargy drowiness, confusion, sleep-wake i nversion, personality change
 Hepatorenal syndrome - oligouria
 HCC
o LOA, LOW, fever, fatigue
o Regular f/u done? Annual U/S and AFP?

Physical examination
 CLD = jaundice, clubbing, leuconychia, palmar erythema, bruising, scratch marks, fetor hepaticus, spider
naevi, gynaecomastia, loss of axillary hair, testicular atrophy, lower limb edema
 Portal HPT = dilated veins, ascites, splenomegaly
 Encephalopathy = terminal asterixis
 HCC = hard and craggy hepatomegaly
 Alcoholism = duputyren’s contracture, parotid enlargement 
 Hep B/C infection = tattoos, IV needle marks

Investigations
Aims
 Confirm diagnosis
 Look for underlying aetiology
 Assess severity
 Look for complications

Bloods
 FBC
o Hb decrease, WBC decrease, pII decrease (hypersplenism)
o WBC increase (SBP)
o Hb decrease (folate/iron-deficiency anemia)
o MCV decrease (alcoholism)
 LFT
o Bilirubin increase
o AST>ALT (alcoholic liver disease)
o GGT increase (alcohol liver disease)
o ALP increase (biliary obstruction)
o Albumin (marker of synthetic function; malnutrition)
 PT/PTT = prolonged PT (marker of synthetic function)
 U/E/Cr = hepatorenal syndrome
 Hepatitis serology
 Autoimmune screen
 Tumor markers = AFP

Imaging
 U/S HBS
o Development of HCC
o Biliary obbstruction

Liver biopsy/ERCP
Assessing severity
Child’s Pugh score
 Originally used to predict peri-operative morality
 Now used for
o Evaluating prognosis of liver cirrhosis
o Management (determine treatment required, necessity of liver transplant)

Measure 1 point 2 points 3 points


Albumin >35 28-35 <28
Bilirubin <34 34-50 >50
Coagulopathy (PT) 1-4s 4-6s >6s
Distension (ascites) None Mild Severe
Encephalopathy none Grade 1-2 Grade 3-5

Points Class Life expectancy Peri-operative


mortality
5-6 A 15-20 yrs 10%
7-9 B Candidate for 30%
transplant
10-15 C 1-3months 82%

Management
Ascites/Lower limb edema
 Fluid restriction
 Low-salt diet
 Strict I/O charting and daily weights
 IV diuretics = Furosemide +/- spironolactone
 Abdominal paracentensis (intermittent peritoneal taps)
o Therapeutic = relieves SOB
o Diagnostic = rule out peritonitis
 Peritoneovenous shunt

Spontaneous bacterial peritonitis


 Usually caused by S pneumoniae
 Investigations = abdominal paracentesis
 Empirical Abx = IV rocephine 1g OM
 Prophylactic Abx = PO Ciprofloxacin 250mg BD X 3/12

Portal HPT
 Results in = varices, ascites and spleomegaly
 Varices
o Portal gastropathy (watermelon stomach = strips of dark red and light red)
o Esophageal varices
o Caput medusae
o Haemorrhoids
 Management
o Propanolol 20mg BD
o TIPPS (does nt reduce mortality)
o OGD with banding/sclerotherapy
Hepatic encephalopathy
 Identify triggers and treat
o Increase protein/urea load = excess dietary protein, constipation, BGIT, uraemia
o Infective = sepsis, HDV infection
o Drug-induced = alcohol binge, anti-depressants, narcotics, sedatives
o Trauma = surgery, porto systemic shunts, paracentensis (>3-5L will result in hypoK)
o Metabolic = hypokalemia
o Neoplastic = HCC
 Low-protein diet
 Constipation = fleet enema STAT, lactulose
 Neomycin = decreases bacterial decomposition of urea into ammonia
 Liver transplant

Malnutrition
 Vitamin D and calcium supplements
 IM vit K injections

Pruritus
 Ursodeoxycholic acid

Hepatic decompensation
 Jaundice
 Hepatic flap
 Coagulopathy
 Ascites
 Lower limb edema
Medicine (GIT) = Hepatomegaly
Causes of hepatomegaly
Right heart failure
Constructive pericarditis
Vascular IVC obstruction
Budd-Chiari syndrome (malignancy = myeloproliferative (PRV) and intra abdominal
(HCC, RCC); PNH, IBD, OCP, SLE/APLS)
Bacterial (Salmonella, Shigella)
Infective Viral (hepatitis, CMV, EBV)
Protozoal (malaria, amoebiasis)
Trauma Haematoma
Primary biliary cirrhosis
Primary sclerosing cholangitis
Autoimmune
SLE
RA
Fatty liver (alcohol, DM, obesity, pregnancy, Cushing syndrome, hyperthyroidism,
IBD, steroids, methotexate)
Metabolic
Storage disorders (type 4 glycogen storage disease, Wilson’s disease, cystic fibrosis,
haemochromatosis, 1-antitrypsin deficiency)
Amyloidosis
Infiltrative
Sarcoidosis
Primary (HCC, hepatic adenoma, hepatoblastoma, hemangioma)
Neoplastic
Secondary (metastasis)
Lymphoma (Hodgkin’s, non -Hodgkin’s)
Lymphoproliferative disease
Haematological
Myeloproliferative disease
Chronic haemolytic anaemia (spherocytosis, G6PD deficiency, thalessemia)

Common causes
 Moderate-large liver
a) Malignancy
b) Fatty liver (esp alcoholic liver disease)
c) Myeloproliferative disease
d) Right heart failure
 Hard and knobbly liver
a) Malignancy
b) Macronodular cirrhosis
c) Cystic  APKD, hydatid
d) Granulomatous/gummatous syphilis
e) Amyloidosis

# glass eye + hard knobbly liver = 1° ocular melanoma with liver metastasis
Chronic Liver disease

Area Sign Aetiology to consider


Head and neck Xanthelesma Primary biliary cirrhosis
(females)  Must always consider in a middle-aged lady
with splenomegaly and minimal signs of CLD
Slate-grey Haemochromatosis
(males)
Parotidomegaly Alcoholic liver disease
 Liver is typically big even if cirrhotic
Raised JVP Right heart failure
Constrictive pericarditis
Kayser- Wilson’s disease
fleischer rings
Limb Duputyren’s Alcoholic liver disease
contracture  Liver is typically big even if cirrhotic
Chorea Wilson’s disease
Tattoo Post-viral (likely Hepatitis B)
Pyoderma Ulcerative colitis
gangrenosum
Lungs Lower zone 1-antitrypsin deficiency
emphysema

-thalessemia major
Short stature
Hyperpigmented
Overall
Thalessemic facies (frontal bossing, flat nosebridge,
maxillary hyperplasia)
Looks younger for age
Pituitary haemosiderosis Hypopigmented areolae
Loss of axillary hair
JVP v wave
Cardiac haemosiderosis Pulsatile liver
Lower limb edema
Hypocount marks on fingers
Pancreatic haemosiderosis
Diabetic dermopathy
Intervention Splenectomy
Request Gonadal examination

Specific conditions
Haemochromatosis
Request to examine for:
a) Arthropathy pseudogout
b) CCFcardiomyopathy
c) Testicular atrophy  pituitary involvement
d) Glycosuria with urine dipstick  DM
Wilson’s Disease
Area Sign Interpretation
Overall Short stature Rickets secondary to
proximal RTA
Eyes Ptosis Penicillamine-induced MG
Pallor and jaundice Coomb’s negative
haemolytic anaemia
Kayser-Fleischer rings Copper deposits in
Descemet’s membrane of
cornea predominantly at 12
and 6 o’clock positions. Can
also occur in PBC and
cryptogenic cirrhosis. Look
out for sunflower cataract 
as well
Face Malar Rash Penicillamine-induced
lupus
Upper limb Small hand joint arthritis Penicillamine-induced
lupus
Tremor/chorea Extrapyramidal syndrome
Lower limb Swollen knees Pseudogout
Request urinalysis for glycosuria (proximal RTA)

Chronic UC
CLD + pyoderma gangrenosum = chronic UC and
a) Cirrhosis
b) Chronic active hepatitis
c) Primary sclerosing cholangitis
d) Cholangiocarcinoma
e) Metastatic colorectal cancer

Request to examine
a) Joints sacroilitis, ankylosing spondylitis, peripheral large joint arthritis
b) Skin  erythema nodosum, pyoderma gangrenosum
c) Mouth  aphthous ulcers
d) Ocular  uveitis, iritis, episcleritis

Template for presentation (a)


“In summary, this patient has hepatomegaly likely secondary to chronic liver disease. I say this because
a) Evidence of stigmata of CLD found on peripheral examination
b) Presence of hepatomegaly measuring ___cm along the mid c lavicular line and of ___consistency
The aetiology is likely to be secondary to Hep B virus (high local endemicity) or alcohol (duputyren’s
contracture, parotidomegaly). This is/is not complicated by portal hypertension and hypoalbuminemia.”

Template for presentation (b)


“In summary this patient has hepatosplenomegaly likely secondary to liver cirrhosis complicated by portal
hypertension. I say this because of
a) Evidence of stigmata of CLD found on peripheral examination
b) Presence of hepatomegaly measuring __cm along the mid-clavicular line and of ___consistency
c) Presence of splenomegaly measuring __cm
d) Features suggestive of portal HPT
This is/is not complicated by hypoalbuminaemia or hepatic encephalopathy”
Medicine (GIT) = Jaundice (history-taking)
Name/age/ethnicity/gender
Occupation
Date of admission

Presenting complaint
1. Jaundice
- Duration
- Onset=acute or gradual
- Skin and eyes affected?
- Progression – getting better, worsening, fluctuating (periampullary ca, gallstones)
2. Obstructive
- Tea-coloured urine
- Acholic stools
- Steatorrhoea
- Pruritus
- Bleeding tendencies (gum bleeding, easy bruising)
3. Abdominal pain (epigastric/RHC pain) = obstructive/hepatic jaundice
4. Fever (a/w chills and rigors)
5. LOA, LOW, malaise
6. Nausea/vomiting
7. Changes in bowel habit (?CRC with liver mets)
8. Melena/PR bleeding (necrosis of periampullary ca? CRC with liver mets, portal HPT)
9. Abdominal distension and lower limb oedema
Aetiology
1. Pre-hepatic = symptoms of anaemia (pallor, chest pain, SOB, giddiness, palpitations)
History of G6PD deficiency and thalassaemia
History of recent blood transfusion
2. Hepatic
#infective
- Travel history
- Contact history
- Recent shellfish/seafood ingestion
- History of Hep B/C infection
- Maternal history/family history of Hep B/C
- Sexual history
- History of blood transfusion/tattoo-ing/IVDA
#drugs
- Alcoholism/recent alcoholic binge
- Recent drug/TCM intake
#autoimmune
- Rash, joint pain and swelling
#neoplasia
- Evidence of mets  chest pain, SOB, bone pain
- If primary: change in bowel habits, melena, tenesmus, haemoptysis
3. Post-hepatic
- History of gallstones
- History of epigastric pain radiating to the back (pancreatic ca)
- History of biliary surgery/instrumentation
Complications
1. Liver failure (acute/chronic)
o Mild heterogeneous condition affecting 6% of the population
o Usually detected during recurrent illness or fasting
o No clinical consequences
 Impaired hepatocyte conjugation
- Gilbert syndrome (decreased uridine diphosphate-glucuronosyltransferase)
- Crigler-Najjar syndrome s I/II (lack or deficiency of uridine diphosphate-glucuronosyltransferase)
o Type 1 more fatal than Type 2 (former is unresponsive to phenobarbitone and requires
liver transplant)
o Causes kernicterus with neurological damage
- Hepatocellular disease
o Vascular (CCF, right heart failure,
fai lure, hypotension, Budd-Chiari syndrome)
o Infected (viral hepatitis, EMV, CMV, HSV, dengue, leptospirosis)
o Toxin (alcohol binge, drugs – paracetamol OD, anti-TB, statins, anti-epileptics, OC, TCM)
o Autoimmune (SLE, idiopathic)
o Metabolic (fatty liver, Wilson’s disease, haemochromatosis, α1-antitrypsin deficiency)
o Infiltrative (sarcoidosis, amyloidosis)
o Neoplasia (HCC, liver mets, leukaemia, lymphoma, myeloproliferative disorders,
myelodysplasia)
Predominantly conjugated hyperbilirubinaemia
- Designated when >15% of an elevated serum bilirubin is conjugated
- Conjugated bilirubin is water-soluble, non-toxic, loosely bound to serum albumin and is excreted in
urine
- Typically associated with cholestasis
 Impaired intra-hepatic excretion of bilirubin
- Dubin-Johnson syndrome
o Defect in the transport protein responsible for excretion of bilirubin glucuronides
o Darkly-pigmented liver
- Rotor’s syndrome
o Variant of DJS
o Liver is non-pigmented
- Hepatocellular disease (refer above + TPN)
 Intra-hepatic biliary obstruction
- Primary biliary cirrhosis
- Primary sclerosing cholangitis
o Recurrent pyogenic cholangitis – recurrent febrile episodes of jaundice
o Vicious cycle of intra-hepatic biliary ductal stonesscarringstrictures
- Cholangiocarcinoma
 Extra-hepatic biliary obstruction
- Gallstones in the CBD
- Liver abscess (amoebic, TB, meliodosis, enteric gram –ve bacilli)
- Ca head of pancreas
- Periampullary ca (cholangiocarcinoma,
(cholangiocarcinoma, Ca ampulla of Vater, Ca duodenum)
- Biliary strictures
- Secondary biliary cirrhosis
- Lymphadenopathy at porta hepatis
Clinical features
Pre-hepatic jaundice
Onset Acute
Precipitating factor Usually present
1st  episode Usually not the first time
Urine and stools Dark urine and stools
 Anaemia Usually present, +/- splenomegaly
Progression Usually self-limiting and gradually improves once
precipitating factor is removed
Onset Viralgradual with prodromal symptoms
Drug/alcohol-inducedacute
Pain Dull and usually insignificant
Usually due to stretching of Glisson’s capsule
Fever May occur with viral hepatitis
Low-grade and non-specific
Urine and stools Dark urine and stools
Progression Usually self-limiting (die or get better)
Post-hepatic jaundice
Onset Gallstoneacute
Pain Gallstonebiliary colic (tenderness)
Ca pancreasepigastric pain radiating to the back
Hepatomegaly (Stretching of Glisson’s capsule)congestion
of intra-hepatic biliary spaces
Fever Characteristic of cholangitis (Charcot’s triad – feverRHC
painjaundice)
Spiking with chills and rigors
Urine and stools Dark urine
Acholic stools steatorrhoea
Progression Progressive and relentless
Fluctuating
- periampullary ca (necrosis of tumour may relieve
obstruction)
- Gallstones
Benign (Sx jaundice) Malignant (Sx jaundice)
Onset Gallstoneacute Usually gradual
Constitutional symptoms Usually absent LOA, LOW, fatigue, fever
Pain Acute and colicky Painless jaundice
Ca pancreasepigastric pain radiating to
back
Courvoisier’s sign: non-tender palpable gall bladder is unlikely to be caused
by gallstones, as enlargement of gall bladder is likely in Ca pancreas, not in
gallstones (- too acute).
Cholangitis More Less
- Charcot’s triad -regurgitation of small bowel -relentless tumour growth eventually leads
- Raynaud’s
Raynaud’s contents up biliary tree to complete obstruction
pentad ->90% will have infected bile -No regurgitation of small bowel contents to
cause infection
2-hit phenomenon Usually obstructs biliary tree Obstructs both biliary and pancreatic
only ductssteatorrhoea and LOW
Endocrine insufficiency Absent Worsens existing DM
Newly diagnosed DM
Migratory thrombophlebitis Usually absent Suggests Ca pancreas (esp body and tail)
Progression Self-limiting if gallstone Relentless and progressive
passes Fluctuatingperiampullary ca
Complications
1. Acute jaundice Acute liver failure
- Coagulopathy
- Ascites/LL oedema
- Encephalopathy=forgetful
Encephalopathy=forgetfulness,
ness, confusion, drowsiness
- Hepatorenal syndrome – renal failure due to liver impairment
o Oliguria/anuria (urine becomes lesser and more concentrated(
o Liver unable to detoxify blood either due to porto-systemic shunting or impaired
hepatocyte function
 High levels of circulating endotoxins
 Formation and deposition of immune complexes in the glomeruli
glomerular/tubular dysfunction
- Hypoglycaemia
2. HBS sepsis
3. Malabsorption (protein, fat, vitamins A/D/E/K)
- Protein malnutritionascites, LL oedema pleural effusion
- Hypoglycaemia
- Fat malnutritionsteatorrhoea, LOW, easy bruising
4. Coagulopathy
- Mechanisms
o DIVC (HBS sepsis)
o Lack of production of coagulation factors (liver impairment)
o Lack of absorption of vitamin K (obstructive jaundice, ca pancreas)
- Clinical features
o BGIT (haematemesis, melena, haematochezia)
o Easy bruising
o ICH
o BGUT (haematuria, menorrhagia)
- Management
o correct by giving vit K if PT>3 above upper limit of normal
o Correct within 48 hrscholestasis
o Remains prolonged hepatocellular insufficiency
5. Portal hypertension
- Secondary to long-standing biliary obstruction biliary cirrhosis
- Pathogenesis
o Accumulation of bile pigments within hepatocytes (foamy degeneration)
o Dilated bile canaliculi with green-brown plugs of bile
o Dilatation and proliferation of bile ductules (secondary to bile stasis and back pressure)
o Ruptured canaliculi leading to extravasation of bile into sinusoids
o Inflammationoedema + neutrophilic infiltrateportal tract fibrosis and cirrhosis
- Cx-ascites, splenomegaly, dilated veins (gastro-oesophageal varices, caput medusa, haemorrhoids),
haemorrhoids),
portal hypertensive gastropathy: mucosal changes.

Investigations
Blood:
1. FBC
- WCC (leucocytosis in infections; leucopoenia in biliary cirrhosis and hypersplenism)
- Hb (anaemia if there is haemolysis, bleeding or underlying malignancy)reticulocyte count
- Haptoglobin assay
- PBF
2. U/E/Cr
- Renal impairment (hepatorenal syndrome)
- Serum glucose (hypoglycaemia)
3. LFT
(a) Establish if it is a predominantly unconjugated/conjugated hyperbilirubinaemia
(b) Hepatocyte integrity AST, ALT, LDH
(c) Biliary obstructionALP, GGT
(d) Synthetic function
- albumin, PT/PTT
- ALT>AST in viral hepatitis
- AST>ALT in alcoholic hepatitis
- Raised ALP/GGT in obstructive jaundice
- GGT specific for alcoholic hepatitis
4. PT/PTT
- Check for coagulopathy
5. Serum ammonia = measure of liver function
6. Hepatitis screen
- Anti-HAV Ig M, anti-HAV Ig G
- HBsAg, HBeA (active replication), anti-HBc Ig M, anti-HBc Ig G
- Anti-HCV, HCV RNA
7. Autoimmune markers (anti-ds DNA, ANA, anti-mitochondrial Ab, anti-Sm Ab, ESR, CRP)
8. Tumour markers = α-fetoprotein, CA 19.9, CA 125, CEA
Imaging:
1. AXR
- Gallstones
- Pneumobilia (cholecystenteric fistula, cholangitis with gas-producing organism)
2. U/S HBS
- Dilated intra-hepatic ducts (obstruction)
- Gallstones in gallbladder
- Liver cirrhosis and masses
- Ascites
3. CT AP
- Gallstones in gallbladder or biliary tree
- Liver and pancreatic masses
- Level of biliary obstruction
- Double-duct sign (dilatation of both CBD and pancreatic duct  periampullary ca)
- LAD at porta hepatitis
- Ascites
4. CXR
- Lung primary or mets
5. ERCP (endoscopic retrograde cholangiopancreatography)
cholangiopancreatography)
- If gallstones, lower CBD or pancreatic head pathology suspected
- Diagnostic
o Direct visualisation
o Obtain samples for histology/cytology (periampullary region, pancreatic fluid and bile)
- Therapeutic
o Remove gallstones
o Sphincterotomy
- Stenting of stricture at lower end
o Contraindications
o Gastrectomy (ECRP poses high risk of perforation as stomach is disconnected from
duodenum)
- Complications
o Traumatic pancreatitis
o Pancreatic/biliary sepsis
6. PTC
- If there is dilatation of intra-hepatic ducts or unsuccessful ERCP
- Diagnostic
o Direct visualisation
- Therapeutic
o Insert catheter for drainage
- Contraindications
o Coagulopathy
o Ascites (unable to tamponade liver puncture)
o HBS sepsis
7. MRCP
- If patient has contraindications to ERCP/PTC
8. Liver biopsy
US/CT-guided core liver biopsy
- Determine hepatic causes of jaundice/grade liver tumour
Decompensated liver cirrhosis
 BGIT
 Constipation
 Sepsis
 Drug-inducedalcohol, steroids
 Hep D infection
 HCC
Post-operative jaundice
(Usually occurs in first 3 post-operative weeks)
 Resorption
o Haematoma
o Haemoperitoneum
o Haemolysis of transfused RBCs (shorter t 1/2
1/2)
o G6PD deficiency
 Impaired hepatocellular function
o Halogenated anaesthetics
o Sepsis
o Hepatic ischaemia 2° perioperative hypotension
 Extra-hepatic biliary obstruction
o Biliary stones
o Injury to biliary tree
Medicine (GIT) =  Acute Hepatitis

 Aetiology
 Vascular: Ischemia
 Infective: Viral (hepatitis viruses, CMV, EBV, HSV, dengue)
Bacterial (salmonella, shigella)
Parasitic (malaria)
 Drug induced: Alcohol, paracetamol, TCM, anti-TB drugs (e.g. isonazid, rifampicin, pyrazinamide),
pyrazinamide),
anti-convulsants (e.g. sodium valporate), satins
 Autoimmune: Autoimmune hepatitis
 Metabolic: Wilson’s disease
 Infiltrative:
 Neoplastic: Massive malignant infiltration

 ALT-AST reversal
 Most liver diseases are characterized by greater ALT elevations than AST elevations
 Exception where AST: ALT ≥ 2
o Alcohol
o Drug induced
o Infections (e.g. salmonella, dengue)
Medicine (GIT) = Viral Hepatitis

Introduction
 Viral hepatitis is caused by viruses that cause inflammation to the liver

Spectrum of clinical manifestations


 Asymptomatic/ subclinical infection = serologic evidence
 Acute hepatitis: symptoms are common to all viruses
 Carrier state = asymptomatic individual but harbouring replicating virus
 Chronic hepatitis → liver cirrhosis
Systemic viral infections
 Infectious mononucleosis (EBV)
 CMV
 Yellow fever
 Dengue fever
 Rubella
 Haantan virus
Immunological responses
 Normal → acute hepatitis
 Less adequate → chronic hepatitis
 Inadequate → asymptomatic
 Hyper → fulminant hepatitis

 Acute hepatitis
4 phases
1. Incubation period
 Peak infectivity = last asymptomatic days of incubation period to early days of acute symptoms
2. Symptomatic pre-icteral phase
 Usually precedes development
development of jaundice by a few days
days to 2 weeks
 Non-specific prodromal illness : headache, myalgia, arthralgia, nausea and anorexia
 Vomitting, diarrhea, RHC pain
 May have dark urine and pale stools
 May have physical signs:
i. Liver is often tender but only minimally enlarged
ii. Occasionally, mild splenomegaly and cervical lymphadenopathy ( more frequent in children
or EBV infx)
3. Symptomatic icteric phase
 Mainly conjugated hyperbilirubinaemia
 Common in actue HAV infection; absent in 50% of acute HBV infection; uncommon in acute HCV
 Jaundice may be mild and the diagnosis may be suspected only after finding
f inding abnormal liver blood
tests in the setting
s etting of non-specific symptoms. Symptoms rarely last longer than 3 -6 weeks
4. Convalescence

Chronic hepatitis
 Symptomatic, biochemical
biochemical or serological evidence
evidence of continuing hepatic disease > 6months with
histological evidence of inflammation and necrosis
Aetiology
o Infective = viral hepatitis (HBV, HCV)
o Drugs = chronic alcoholism,
alcoholism, isonazid, methotrexate,
methotrexate, methyldopa, nitrofurantoin
o Autoimmune = autoimmune hepatitis (may be associate with primary biliary cirrhosis and primary
sclerosing cholangitis)
o Metabolic = Wilson’s disease, haemochromatosis, α1 –antitrypsin deficiency
o Spontaneous remission
o Indolent disease without progression
o Rapidly progressive disease → cirrhosis
Causes of death
o Liver cirrhosis
o Liver failure
o Haematemesis
o Hepatocellular carcinoma

Hepatitis A virus
 Epidemiology= usually found in developing world → substandard hygiene & sanitation;
prevelance of seropositivity increases with age
 Caused by picornavirus (ssRNA), 1 serotpe
 Mode of transmission=
o faecal oral route
o food & water borne (e.g. eating partially cooked cockles & oysters/ contaminated food & water)
o person-person
person-person (e.g. sexual oral-anal)
 Incubation period = 4-6 weeks
o HAV appears in faeces before clinical symptoms (usually 2-3 weeks before jaundice & 1 week after
onset of jaundice)
 Clinical presentation
o Asymptomatic (most) = subclinical & milder than HBV infection
o Acute hepatitis= usually bengn and self limiting
o Worse if superimposed on chronic hepatitis
o Does not cause chronic hepatitis
hepatitis or carrier state
 Complications: Fulminant hepatitis (rare)
 Serological picture:
o Transient viraemia → blood borne transmission rare
o IgM with acute infection → fecal shedding ends as IgM increases
o IgG for long term immunity

Prevention
 Avoid eating contaminated food or drinks
 Boiling 5 mins
 Immunization
o Passive immunization with Ig G
 IgG collected from blood of persons who have been exposed to the hepatitis A
 This method of immunization is getting obsolete because of the short supply of immune
globulin and the potential risk of transmission of other infection through blood products
o HAV vaccine
 Given in 2 doses, with the second dose being given 6 - 12 months later. Immunity after
vaccine lasts for 10 - 20 years. Protection against hepatitis A begins 4 weeks after
vaccination
People at risk of HAV
 Persons travelling to or working in countries that have high or intermediate rates of hepatitis A
 Persons who work with hepatitis A virus infected primates or with hepatitis A virus in a
research laboratory should be vaccinated.
 Persons with chronic liver disease eg. chronic
ch ronic hepatitis B carriers as these patients have been reported to
have a higher mortality.

Hepatitis B virus
 Epidemiology: endemic in Africa and Asia;

Microbiology
 Belongs to the Hepadnavirus family
 Has 3 well characterized antigens:
o HBsAg (surface) → stimulates anti-HBs
o HBcAg (core) → stimulates anti-HBc
o HBeAg (core associated) → stimulates anti-HBe
 Dane particle = infectious spherical HBsAg particle containing HBcAg core
 HBeAg arises from the same gene as HBcAg
o c gene has 2 initiation codons= precore
precore and core region
o translation intitated at precore region = HBeAg → signal peptide that facilitates secretion (can be
used as surrogate marker for presence of HBcAG)
o translation initiated at core region = HBcAg → no signal peptide →not secreted into serum
 Nucleocapsid
o circular partially ds DNA
o DNA polymerase with reverse transcriptase activity
o HBcAg → remains in hepatocytes for complete assembly of virions, only detected in liver biopsy
samples

 Incubation period = 3-4months


 Pathogenesis
o Cell-mediated mechanisms
mechanisms = destruction of hepatocytes with viral/ modified surface antigens
o Humoral-mediated
Humoral-mediated mechanisms = GN/ vasculitis from circulating immune complex
 Mode of transmission
o Vertical transmission
o Sexual transmission
o Parententral transmission: blood transfusion, organ transplant, needle-stick injury, IV drug abuser
 Clinical presentation:
o Asymptomatic disease (90%)
o Acute hepatitis
 Fulminant hepatitis rare
o Carrier status (10-15%)
o Chronic hepatitis (5%)
o Liver cirrhosis (3%)
o HCC (1%)
Serology

1) Acute infection with recovery


 HBsAg= appears before onset of symptoms, peaks and declines rapidly, undetectable at 3 -6months
 HBeAg= appears just after HbsAg, indicates active replication (infectiousness)
i. anti-HBe appears after disappearance of HBeAg (indicates waning infection)
 anti-HBc= IgM appears just prior to the onset of infection (indicates acute infection); replaced by
IgG
i. does not protect against re-infection
ii. serves as a surrogate marker for natural HBV infection
 anti-HBs IgG= appears after acute disease is over

2) Acute infection with progression to chronic disease


 Carrier state = presence of HBsAg > 6 months
 Chronic replication of HBV virions = persistent HBsAg, ±HBeAg
± HBeAg and HBV DNA
 Chronic sequel: cirrhosis & HCC
 High risk of becoming a carrier:
i. Age at time of infection
 Infants: 40-50%
 Children: 30-40%
 Adults: 5-10%
ii. Sex – male: female 3:1
iii. Ethnicity – Chinese> Malays> Indians; related to prevalence of female carriers and periantal
infx
iv. Impaired immune responses – transplants, drugs

Markers of past infectivity


 Anti-HBs IgG
 Anti-HBc IgG
 Anti- HBe

HBV mutants
 Pre-core mutant : variant C gene fails to produce HBeAg (–ve HBeAg viraemia); still infections because of
HBcAG
o HBV DNA necessary to detect presence of disease activit y
 S mutants: mutation at ‘a’ epitope (HBsAg –ve viraemia) → vaccine not effective; low frequency in
Singapore

Treatment
 Anti-virals: lamivudine, adefovir
 Interferon-α
 Vaccination

Hepatitis C virus
 Caused by Flavivirus, ssRNA
 Transmission: blood-borne, sexual intercourse
 Incubation period: 6-12 weeks
 Clinical presentation
o Mainly asymptomatic
o Acute hepatitis = general milder than HBV; no effective immunity
o Chronic hepatitis = hallmark of HCV
HCV infection
 60-80% develop chronic hepatitis
 20% go on to develop liver cirrhosis
Acute infection with recovery
recovery
 HCV RNA detectable for 1-3 weeks
during active infection,
 HCV RNA frequently persists
despite neutralizing antibodies (Abs
 present in 50-70% of acute
infection; 30-50% have anti-HCV
Abs after 3-6 weeks)

Chronic infection
 Persistence of HCV RNA despite
neutralising Ab
 Episodic elevations of HCV RNA
and transminases

Treatment
 Ribavirin and IFNα combination therapy → partial efficacy
 No vaccine available; difficult to cover agains the 6 major genotypes
Hepatitis D virus
 Defective ssRNA virus → requires HBsAg coat to infect cells
 HBV serves as helper virus

Clinical presentation
1. Super infection: chronic HBV carrier exposed to HDV → severe hepatitis
2. Co-infection: exposed to HBV & HDV at the same time
a. HBV must become established first to provide HBsAg required for HDV virion production
b. Chronic hepatitis rare
c. Higher rates of fulminant hepatitis (3-4%)

Serology
 HDV RNA appears just before and during early acute symptomatic infection
 IgM anti-HDV = recent HDV exposure
 To differentiate co-infectin and super infection = correlate with HBV markers

Hepatitis E virus
 Calicivirus, ssRNA
 4 genotypes, endemic in India and the Middle East
 Transmission: faecal-oral, water borne
 Incubation period= 4-6 weeks

Clinical presentation
 Acute hepatitis
o Usually self-limiting and benign
o Abs are non-protective
 No chronic state or chronic hepatitis
 High rate of fulminant hepatitis in pregnant women (25% fatal); foetal mortality also high
 No vaccines

Serology
 HEV RNA and HEV virions present in stool and liver before onset of symptoms
 IgM anti-HEV present with rising transaminase → IgG

Hepatitis Screen
 HAV=
o Anti- HAV IgM (acute)
o Anti- HAV IgG (previous infection)
 HBV=
o HBsAG, HBeAG, anti-HBc, IgM (acute)
o Anti-HBs IgG, anti-HBe, anti-HBc IgG (previous infection)
 HCV=
o Anti-HCV IgM (acute)
o Anti-HCV IgG (previous infection)
 CMV = anti-CMV IgM
 EBV = anti EBV IgM
 HSV = anti-HSV IgM, HSV PCR (if patient presents with acute liver failure)
HAV HBV HCV HDV HEV HGV
Agent Icosohedral capsid, Enveloped dsDNA Enveloped ssRNA Enveloped ssRNA Unenveloped ssRNA ssRNA
ssRNA Calicivirus
Picornavirus Hepadnavirus Flavivirus Flavivirus
Transmission
Transmission Faecal-oral Parenteral, close Parenteral, close Parenteral, close Waterborne Parenteral
contact, vertical contact contact
Incubation period 2 – 6 weeks 4 – 26 weeks 2 – 26 weeks 4 – 7 weeks 2 – 8 weeks Unknown
(superinfection)
Carrier state None 0.1 – 1% of blood 0.2 – 1% of blood 1 – 10% of drug Unknown / none 1 – 2% of blood
donors; 90 – 95% of donors; <1% are addicts, donors
those infected at healthy carriers haemophiliacs
birth (vertical
transmission); 1 –
10% infected as
adults (esp. If
immune-
compromised)
Chronic hepatitis None 5 – 10% of acute >60%; half then <5% if co-infection None None
infections (adults); progress to cirrhosis with HBV; 80% upon
90% in infected super infection with
neonates HBV
Fulminant hepatitis 0.1 – 0.4% <1% Rare 3 – 4% in co- 0.3 – 3% Unknown
infection 20% in pregnant
females
Hepatocellular Ca No Yes Yes No increase above Unknown, but None
HBV unlikely
Vaccine available Yes Yes No No No No
Others Acute hepatitis Fulminant hepatitis At present, not
(symptomatic, almost never occurs considered
asymptomatic) with HCV pathogenic

151
Medicine (GIT) = Alcoholic Liver Disease

*clinical spectrum*
1) fatty change/hepatic steatosis
 Clinical features – asymptomatic, mild increase in serum bilirubin & ALP
 Pathology – soft, greasy hepatomegaly
Fat accumulation within hepatocytes
(micro macro-vesicular)
 Seen within days of ingestion on U/S
 Initially reversible

2) acute alcoholic hepatitis


 Clinical features – fever, RHC pain, jaundice
 RHC pain may result from – alcoholic gastritis, pancreatitis or srteching of Glisson’s capsule
 Spectrum may range from self-limiting episode to fulminant hepatitis
 Potentially reversible

3) alcoholic cirrhosis (10-15% of alcoholics)


 Clinical features – features of Chronic Liver Disease and portal HTN
 Irreversible and potentially fatal
 Micronodular cirrhosis

*physical findings of alcoholism*


 Duputyren’s contracture
 Dilated cardiomyopathy – displaced apex beat, signs of CCF
 Cerebellar signs
 Parotidomegaly
 Proximal myopathy
 Peripheral neuropathy
 Dementia
 Wernicke’s encephalopathy – classic triad of:
1) encephalopathy (confusion, loss of short-term memory)
2) ataxia
3) ophthalmoplegia (nystagmus, gaze palsies)
 Korsakoff’s psychosis/syndrome –
1) anterograde amnesia
2) retrograde amnesia
3) confabulation (invented memories which are then taken as true due to gaps in memory sometimes
associated with blackouts)
4) meagre content in conversation
5) apathy
6) lack of insight
*investigations*
 FBC – raised MCV
 LFT – raised ALP, GGT, AST > ALT, raised conjugated bilirubin, decreased albumin
 PT/PTT
 Hepatitis serology
 U/S liver

*causes of death*
 BGIT – haematemesis
 Hepatic encephalopathy
 Hepatorenal syndrome
 Sepsis
 HCC

Medicine (GIT) = Autoimmune Hepatitis

*2 main forms distinguished by type of circulating antibody:


 Type 1 – ANA, anti-SMA (smooth muscle actin) Ab, increased frequency of HLA B8 and
HLA DRw3
 Type 2 – anti-LKM1 (liver kidney microsomal 1) Ab

*epidemiology
 females > males (3:1)
 Premenopausal females

*clinical presentation
 acute hepatitis with autoimmune symptoms (fatigue, arthralgia, myalgia)
 chronic hepatitis
 incidental finding with signs of chronic liver disease

*associated with autoimmune disorders – RA, thyroditis, scleroderma, IBD, pernicious anaemia, IDDM,
AIHA, PSC

*investigations
 FBC – decreased Hb, decreased WBC, decreased platelets (w/ hypersplenism)
 LFT – raised AST
 Hyperglobulinaemia – raised IgG
 Autoimmune screen – ANA, anti-SMA Ab, anti- LKM1 Ab
 Hepatitis serology – negative
 Liver biopsy – mononuclear infiltrates of portal and peri-portal areas,
piecemeal necrosis  fibrosis  cirrhosis

*treatment
(a) immunosuppression – prednisolone, azathioprine (steroid-sparer)
(b) liver transplant
*prognosis – if untreated severe disease:
 40% die within 6 months,
 40% of survivors develop cirrhosis
Medicine (GIT) = Metabolic Liver Disease

1) Non-alcoholic fatty liver and steatohepatitis


*non-alcoholic fatty liver disease (NAFL)*
 Characterized by increased serum transaminases and hepatic steatosis in the ABSENCE
of heavy alcohol consumption
 Associated with: DM, obesity, pregnancy, methotrexate, steroids, Cushing’s syndrome,
hyperthyroidism

*non-alcoholic steatohepatitis (NASH)*


 Characterized by hepatic steatosis and inflammation in the ABSENCE of heavy alcohol
comsumption
 Pathology: hepatocytes containing fat vacuoles
varying amounts of fibrosis
+/- inflammatory infiltrates

2) Haemochromatosis
*excessive iron accumulation with subsequent deposition in various organs esp. liver and pancreas
*primary haemochromatosis (hereditary haemochromatosis)*
 Autosomal recessive inheritance
 Males > females (6:1)
 Females diagnosed later (menstruation offers protection)
 Pathogenesis: unregulated intestinal Fe absorption  excess Fe  direct toxicity via free radical
formation/lipid peroxidation/Fe-DNA interactions
 Usually presents at around 20-30yrs old

Skin pigmentation – slate-grey appearance


 Anterior pituitary gland – adolescent (delayed puberty, hypogonadism)
adult (appears young for age, amenorrhea, impotence/loss of
libido, testicular atrophy)
Pituitary failure – hypopituitarism
Liver – hepatomegaly  cirrhosis  HCC
Pancreas – DM
Heart – cardiomyopathy, arrhythmia
Musculoskeletal – pseudogout

*secondary haemochromatosis*
 Repeated blood transfusions – Thalassemia major, aplastic anaemia, sickle cell disease, myelodysplastic
syndrome, leukaemia, lymphomas
 Increased Fe intake – Fe-dextran injections

*investigations*
 Iron studies – high transferring saturation? High ferritin levels?
 U/S liver – HCC is the commonest cause of death (200x greater risk of getting HCC)
 Liver biopsy (diagnostic) – measure liver stores
 Genetic testing

*management*
Medicine (GIT) = Wilson’s disease (Hepatolenticular Disorder)

*History*
 Hx of consanguinity  Wilson’s disease has an autosomal recessive inheritance
 Considered in any patient younger than 40yrs with unexplained disorder of CNS, hepatitis, chronic
active hepatitis, haemolytic anaemia, unexplained cirrhosis, or has a relative with Wilson’s disease

*Physical Examination*
 Kayser-Fleischer rings: greenish yellow to golden brown pigmentation at the li mbus of the cornea due
to deposition of copper in Descemet’s membrane
 Proceed to look for:
o Jaundice
o Sunflower cataracts
o Hepatomegaly
o Signs of liver failure
o Neurological manifestations: tremor, chorea, mask-like
facies

*Presentation*
 Hepatic – 50% of patients (usually presents in 2nd
decade/children)
1) Acute hepatitis – self limited
2) Parenchymal liver disease (chronic hepatitis) – may follow acute hepatitis or develop
insidiously without prior disease
Indistinguishable from chronic active hepatitis and cirrhosis
3) Cirrhosis – may develop insidiously after a lapse of decades
4) Fulminant hepatitis – generally fatal, characterized by progressive jaundice, ascites,
encephalopathy

 Neuropsychiatric – always accompanied by Kayser-Fleischer rings(usually the presenting complaint in


adults)
1) Acute
-bradykinesia
-behavioural change
-involuntary movements
-liver involvement common
-if untreated  death in 2yrs

2) Chronic
-marked proximal ‘wing -beating’ tremor
-dysarthria, dystonia and rigidity
-choreoathetoid movement
-psychosis, behavioural disorders and
dementia
-if untreated  death in 10yrs
*Discussion*
 Inheritance
o Autosomal recessive; chromosome 13
o A/w family history of consanguinity

 Pathophysiology
o Excessive absorption of Cu from the small intestine with decreased excretion by liver
o Increased tissue deposition esp in brain, cornea, liver and kindey Fanconi’s Syndrome
(glycosuria)
o Cavitation and neuronal loss occurs within the putamen and globus pallidus (basal ganglia)

 Biochemical changes
o Decreased serum ceruloplasmin
o Serum Cu concentration might be high, low or normal
o Increased urinary Cu excretion
o Increased liver Cu content

 Diagnosis
o Kayser-Fleischer rings + serum ceruloplasmin levels < 20mg/l
o Serum ceruloplasmin level < 200mg/l + Cu concentration in liver biopsy sample > 250 µg/g
o MRI (T2) shows thalamic and putaminal hyperintensity

 Clinical stages
o Stage I: asymptomatic accumulation of Cu in liver
o Stage II: asymptomatic or manifests with haemolytic anaemia or liver failure
o Stage III: Cu accumulates in brain
o Stage IV: progressive neurological disease

 Treatment
o Low Cu diet
o Chelating agent, eg. penicillamine.
-side effects: anaphylaxis, skin rash, bone marrow suppression and glomerulonephritis 
alternate treatment: trientine.
-penicillamine has anti-pyridoxine effect, thus pyridoxine given together

o Zinc sulphate: chelates with Cu in gut  slow maintenance treatment


-must not be given with penicillamine or trientine which can chelate zinc and render treatment
ineffective

o Adequate treatment compatible with normal life expectancy


o Genetic screening for rest of the family
o Liver transplant
Surgery (GIT) = Obstructive jaundice
A. Intrahepatic biliary obstruction
*Primary biliary obstruction*
*epidemiology:
 Predominantly affects middle-aged women (~50yrs old)

*pathology
 chronic granulomatous inflammation that destroys interlobular bile ducts  fibrosis  liver cirrhosis

*clinical presentation
 fatigue
 pruritus (main presenting complaint, may precede jaundice by mths/years)
 occasionally jaundice, RHC pain, diarrhoea/steatorrhea

*complications:
 Osteopenia/osteoporosis (due to malabsorption of fat-soluble Vitamin D)
 Coagulopathy (due to malabsorption of fat-soluble Vitamin K)
 Liver cirrhosis  portal HTN  HCC (relative risk=20)
 Increased risk of cancer overall

*clinical signs:
 Stigmata of chronic liver disease: palmar erythema, leukonychia, bruising, scratch marks, spider naevi,
gynaecomastia
 Digital clubbing
 Xanthelasma and xanthomata (over joints, skin folds and trauma sites)
 Signs of portal HTN: dilated veins, hepatosplenomegaly (early stages), ascites
 Request to examine for: proximal muscle weakness (osteomalacia)
peripheral neuropathy

*associated with autoimmune/connective tissue disorders:


 RA
 Systemic sclerosis
 Hashimoto’s thyroiditis
 Sjogren’s syndrome
*investigations:
 LFT
 PT/PTT
 Fasting lipid panel (should see raised TC levels)
 Anti-mitochondrial Ab
 U/S hepatobiliary system
 Liver biopsy
*management
 Nutrition: reduce fat intake, oral calcium, low-fat milk, vitamins A/D/K supplements , mid chain TG
supplements
 Ursodeoxycholic acid (bile salt therapy): partial replacement of water-soluble bile acids may reduce
pruritus and damage to hepatocytes already affected by autoimmune processes
 Pruritus (retention of bile acids with cholestasis: increase in concentration and up-regulation of
endogenous opioid receptors: 1 st  line  cholestyramine, 2nd line  ursodeoxycholic acid, rifampicin, 3 rd
*complications
 Cholangitis  liver abscess  HBS sepsis
 Pancreatitis
 Biliary obstruction  liver cirrhosis  portal HTN
 Increased risk for cholangiocarcinoma
*investigations
 U/S HBS
 CT A/P
 ERCP/MRCP
*management
 Treatment of acute cholangitis
o Take blood cultures  start on IV empirical broad-spectrum antibiotics  definitive antibiotics
once results available
o Biliary drainage – open vs. percutaneous
 Prevention of long-term complications
o General approach:
- removal of stones with regular surveillance (ERCP, percutaneous, surgery, laser)
- surgical resection of affected hepatobiliary segment with biliary-enteric anastomosis

*Cholangiocarcinoma*
* arises from epithelial cells of the intrahepatic and extrahepatic bile ducts
 Histological subtypes: adenocarcinoma (95%)
squamous cell carcinoma (5%)

*classification
 Intrahepatic (10%)
o Least common
 Extrahepatic (90%)
o Peri-hilar (65%)  confluence to upper border of pancreas
o Distal (25%)  upper border of pancreas to ampulla of Vater
^peri-hilar cholangiocarcinoma:
 Most common
 Also called Klatskin tumours (occur at bifurcation of right and left hepatic ducts)
 Bismuth classification: Type 1 (below the confluence)
Type 2 (reaching the confluence)
Type 3 (occluding common hepatic duct and either right/left hepatic duct)
Type 4 (multicentric)

*clinical presentation
 Diagnosis usually made in the 7th decade
 Males > females
 Usually presents late  metastatic at time of presentation
 Insidious onset of jaundice, pruritus, RHC pain
 May present with cholangitis

*associations
 Inflammatory conditions (ulcerative colitis, primary sclerosing cholangitis, RPC)
 Fibropolycystic liver conditions (choledochal cyst, Caroli’s disease)
 Toxin exposure (thorotrast)

*management
 Curative surgery (rarely possible)  wide resection and reconstruction of biliary tree
o Indications:
 inrahepatic tumour confined to 1 lobe of liver
 Extrahepatic tumour
 Patient fit for surgery
o Contraindications:
 bilateral/multifocal intrahepatic disease
 Invasion of portal vein/hepatic artery
 Nodal involvement
 Distant metastasis

 Palliation
o Stenting
o Surgical bypass – cholecystojejunostomy, choledochojejunostomy

 Adjuvant therapy
o Radiotherapy
o Chemotherapy

*dismal prognosis: 15% 5yr survival rate


B. Extrahepatic biliary obstruction

*Biliary strictures*
*aetiology
 Others (5%): impacted gallstone
 Post-traumatic (95%):
o Causes:
 Surgical
 Blunt abdominal trauma
o Classification:
 Early vs. late
 Early: due to technical problems
 Late: due to vascular insufficiency and problems with healing and fibrosis
 Anastomotic vs. non-anastomotic
 Anastomotic
o Due to post-operative oedema and inflammation
o Management: endoscopic balloon dilatation and stenting
o Require life-long surveillance as high recurrence rate
 Non-anastomotic
o Due to vascular insufficiency or recurrence of underlying disease
o More difficult to treat:
-endoscopic balloon dilatation with sphincterectomy and stenting
-surgery (choledochoduodenostomy, choledochojejunostomy, end-to-
end bile duct anastomosis)
*clinical presentation: intermittent cholangitis
*investigations: PTC/ERCP (depends on level of narrowing)
 Cholangitis  liver abscess  HBS sepsis
 Secondary biliary cirrhosis  liver cirrhosis  portal HTN

*Periampullary carcinoma*
*includes: cholangiocarcinoma (involving distal common bile duct)
ampulla of Vater tumour
duodenal adenocarcinoma

*clinical presentation (presents early)


 Obstructive jaundice (intermittent/fluctuating as tumour necrosis periodically re-establishes duct
patency)
 Melaena (as tumour sloughs into duodenum)
 Palpable gallbladder

*metastasis more common than primary tumours

*management: Whipple’s operation

*better prognosis than Ca pancreas


 At time of diagnosis: if 80% are localized and small  resectable for cure
 50% 5 yr survival rates

*Mirizzi syndrome*
*rare cause of obstructive jaundice
*due to lodgement of gallstone in cystic duct/Hartmann pouch causing extrinsic compression of
common bile duct
*aetiology
 Acute/chronic inflammation causing constriction of gallbladder which fuses with and causes secondary
stenosis of common bile duct
 Cholecystocholedochal fistula secondary to direct pressure necrosis of adjacent duct walls

*classification
 Type 1: no fistula present
 Types 2-4: fistula present (depending on size of defect w.r.t. diameter of common bile duct)

*investigations
 U/S HBS
 CT A/P
 ERCP/PTC

*management: surgical (cholecystectomy and closure around T-tube)


History-taking
Name/age/race/gender/occupation
Drug allergy
Past medical history

Presenting complaint
Symptoms
1. Bloody diarrhea
- Duration = chronic if persists more than/equals to 4 weeks
- Baseline and current frequency
- Acute/gradual/congenital onset
- Describe stools = watery
volume of stools

float, foul smelling, hard to flush away (differentiate from steatorrhoea)


color of blood (proximal colon dark red; distal colon bright red)
blood mixed with stools or on top of stools
staining of toilet paper
colour of stools (rule of melena)
- Anal pain and pre-defecation straining = rule out haemorrhoids
- Estimate blood loss = pallor, exertional chest pain and dyspnoea, non-vertiginous
giddiness, fatigue
2. Presence of mucous in stools
3. Urgency
- Faecal incontinence
4. Abdominal pain
- Ask about pain characteristics
- Relieved with defecation

Aetiology
 Infective = fever, abdominal pain, LOW
recent travel and contact history
history of TB
history of antibiotics usage (use of broad spectrum antibiotics = clindamycin,

ampicillin, cephalosporin)
 Trauma = history of radiation
 Autoimmune = oral ulcers, joint pain, back pain, unilateral red and painful eye
 Neoplastic = LOW, LOA, fever, fatigue, nausea/vomiting, abdominal pain, abdominal
distension, recent changes in bowel habit, preceding constipation, jaundice

Complications
 Dehydration = decreased urine output
 Protein losing enteropathy = LL oedema

Systemic Review
Management prior and during admission
Has this happened before?
1. History of IBD
- Duration of disease
- Presenting complaint investigations management
- Currently on follow-up? Compliance
- Current medications = types
recent changes

compliance
side effects
- Control of symptoms = frequency of bloody diarrhea
frequency of exacerbations requiring admission
blood transfusion required
usual precipitating factors
- Triggers for present relapse = stress
intercurrent infections/GE
antibiotics/NSAIDs use
non-compliance to medications
- Complications= red eyes, oral ulcers, back pain, joint pain
toxic megacolon perforation
history of CRC on regular colonoscopy
liver problems (obstructive jaundice)

Past medical history

Drug history

Social history
Smoking
Alcohol drinking
Family set-up and main care-giver
Sexual orientation and contact = HIV can present as diarrhea
Type of housing
Lift-landing
Financial status
Functional level
How has illness affected your life?

Family history

Physical examination
General inspection
 General condition = in pain and distress
toxic-looking
pallor
cachexia
 Cushingnoid features
 Patient ’s vitals
 Hydration status
Peripheries
 Hands = digital clubbing, palmar crease pallor
 Eyes = conjunctival pallor, episcleritis, iritis, conjunctivitis
 Oral cavity = angular stomatitis, oral ulcers
 Abdomen = tenderness, guarding, peritoneal irritation, distended (toxic megacolon),
abdominal mass (thickened bowel loops/intra-abdominal abscess), rectal mass
on PR (CRC)
 Perianal disease = sentinel tags, fistulae,
fistu lae, fissures, abscess
 Lower limbs = ankle oedema (protein-losing enteropathy), erythema nodosum, pyoderma
gangrenosum

Differentials
 Inflammatory bowel disease = ulcerative colitis
 Coorectal cnacer
 Infective colitis = amoebiasis, TB
 Radiation colitis/proctitis
 Ischaemic colitis
 Antibiotic-associated colitis (pseudomembranous
(pseudomembranous colitis casued by C. difficile)

Investigations
Blood
 FBC = low Hb (anemia a/w iron deficiency)
WBC and differential count (chronic enteric infection)
Low platelet (bleeding diasthesis)
Elevated hct (dehydration)
 U/E/Cr = elevated urea > elevated creatinine (dehydration, BGIT)
Electrolyte abnormalities (hypokalaemia and metabolic acidosis)
 LFT = Albumin (LOW, LOA, protein-losing enteropathy)
Elevated conjugated bilirubin + ALP + GGT (primary sclerosing cholangitis a/w IBD)
 PT/PTT = BLEEDING DIASTHESIS
 GXM
 ESR and CRP = raised in acute UC
 Blood c/s (especially in febrile patients with known colitis/CD)

Stools
 Stool studies = gram-staining, culture /sensitivity
Microscopy for ova, cysts and leukocytes
C. difficile toxin

Imaging
 AXR = colitis (toxic megacolon, mucosal oedema aka ‘thumb-printing’, perforation)
 Colonoscopy and biopsy
- UC = confluent lesions most severe in rectum and distal colon
No strictures
- CD = skip lesions, apthoid ulcers and strictures, rectal sparing, perianal disease
- NSAID = microscopic colitis (scope may look normal but still biopsy)
 Barium study/Barium enema/Barium meal (if colonoscopy cannot be done)
- UC = shortened colon, loss of haustrations, lead pipe appearance, featureless colon
- CD = strictures (string-sign), skip lesions, cobble stoning of mucous, deep fissured
ulcerations, fistula, loss of haustrations
Management
 Acute exacerbation
(a) Stabilize patient ’s vitals esp circulation
- Set 2 large-bore IV cannulas
- Take bloods for investigations esp GXM
- Fluid resuscitation if in shock = crystalloids colloids PCT
(b) Keep NBM and maintain on IV hydration. Start I/O charting
(c) IV empirical antibiotics if febrile = Ciprofloxacin or Metronidazole (cover against E. coli)
(d) IV high-dose hydrocortisone x 5/7
If refractory IV cyclosporine (much faster onset than azathioprine)
(e) Monitor vitals q4hrly = inform doctor is SBP <100mmHg or HR>100/min
Place on stool charting

 Surgery if not responsive to conservative treatment after 3-5 days


(a) Fulminant colitis = monitor for fever,
f ever, tachycardia and signs of peritonitis
monitor stool frequencies and volumes
f or toxic megacolon/perforationurgent colectomy
regular AXR for
(b) Perianal disease (fissures, fistula, abscess) = antibiotic cover
surgery

 Long-term management
(a) Pharmacotherapy
1. 5 ASA (5-aminosalicyclic acid)
- prototype = sulfasalazine (5 ASA + sulfapyridine)
- MOA = blocks arachidonic acid metabolism to prostaglandins and leukotrienes
- Routes of administration = oral
suppository (proctitis covers 10-15cm from anal verge)
enema (proctosigmoiditis)
- Topical = very effective for distal disease (up to splenic flexure)
better than steroids
oral = effective for pancolitis
- S/E = nausea, vomiting, headache, rashes, haemolytic anaemia, agranulocytosis

2. Steroids (oral prednisolone, hydrocortisone foam)


- Best drugs to remit acute disease
- May cause Hep B flare
 Chronic Hep B = immunity ineffective ineradicating Hep B, thus, exist in equilibrium
 Immune suppression = HBV DNA +++
 Once immune suppression removed = Hep B immunity recovers and causes a flare (T-cells
destroy hepatocytes

3. Immunosuppressants
Immunosuppressants (azathioprine, methotrexate, cyclosporine, anti-TNF)
- Indications = steroid-dependent
steroid-dependent
Steroid toxicity
- Anti-TNF may reactivate pTB, thus, do CXR and mantoux test

4. Oral antibiotics for perianal disease (CD)

Introduction of remission
Mild disease  Oral 5 ASA
 5 ASA/steroid enema
Moderate disease  Oral prednisolone 40mg OM
Severe disease  IV hydrocortisone x 5/7
Maintain remission
Ulcerative colitis or Sulphasalazine (usu  Proven benefit in colitis = not for
Crohn’s colitis 2g) small bowel CD
 Decreases relapse risk from 60%
to 15%
Crohn’s ileal disease  Steroids
 Immunosuppressants

(b) Nutritional therapy


- Small bowel strictures (CD) = avoid nuts, raw fruits and vegetables
- Constipation = increase dietary fibre and fluid intake
- Malnutrition (esp in CD ileal disease) = refer dietician, diet supplements

(c) Surgery
- Indications = impaired quality of life
failure of conservative treatment

fulminant colitis/toxic megacolon (UC)


CRC/severe dysplasia
- UC = aiming for CURE (curative)
 Pan-proctocolectomy
Pan-proctocolectomy with ileostomy
 Proctocolectomy with ileal-anal anastomosis  may cause pouchitis (recurrent in
ileal pouch)
- CD = generally reserved for complications (fistulae, I/O, perforation, abscess, BGIT)
 At least 50% recurrence within 5 years
 Complications of ileal resection = <100cm (watery diarrhea due to impaired bile salt
absorption; cholestyramine)
>100cm (steatorrhoea due to bile salt deficiency;
Tx: fat restriction, medium-chain triglycerides)
 Perianal disease = I & D for abscess, stricture, plasty, fistulectomy
 Large bowel disease = pan-proctocolectomy with ileostomy (pouch not
recommended due to risk of recurrence)

Prognosis
 Life expectancy same as general population
Repiratory Medicine
Medicine (Respi) = History Taking: Respiratory System (General)

Name/age/race/gender/occupation
Date of admission

Presenting complaint
1. Respiratory symptoms
(a) Fever = when did it happen
acute/gradual onset
T max? associated with chills and rigours?
symptoms of raised ICP = vomiting, headache, photophobia, neck stiffness
pattern (constant, swinging, spiking)
relieved with anti-pyretics?
progressively better or getting worse?
Management before coming into hospital
(b) Cough = productive/dry
colour of sputum? amount? Smell?
haemoptysis (exclude haematemesis and trauma)
character (barking/brassy/hollow)?
(barking/brassy/hollow)?

- Barking = epiglottitis
- Brassy = tracheal compression by tumor
- Hollow = recurrent laryngeal nerve palsy (vocal cords are unable to close
completelybovine cough)

worse in the night/early morning or same throughout the day


does cough wake you up from sleep? Wake others up?

- Recent onset = acute bronchitis, pneumonia


- Chronic nocturnal cough and a/w wheezing = asthma, heart failure
- Irritating dry cough = GERD, ACE-inhibitors
- Large amount of purulent sputum = bronchiectasis, lobar pneumonia
- Foul-smelling dark coloured sputum = lung abscess with anaerobic organisms
- Pink and frothy = pulmonary oedema

(c) URTI symptoms = rhinorrhoea, blocked nose, sore throat


(d) Hoarseness (laryngitis, vocal cord tumour, recurrent laryngeal nerve palsy)
(e) Noisy breathing = inspiration (stridor)
expiration (wheeze)
(f) Chest pain = onset, frequency, duration
sudden/gradual onset
what were you doing at onset?
progressively worsening or getting better
site and radiation
character of pain
severity of pain
precipitating, aggravating and relieving factors
(g) Dyspnoea = onset, frequency, duration
progressively worsening or getting better
severity (must rest for how long)
precipitating, aggravating and relieving factors
effort tolerance (walking on level ground, climbing up stairs)

- a/w wheeze = asthma, heart failure, COPD


- chronic progression = pulmonary fibrosis, COPD
- acute onset = pneumonia, pneumonitis
- diurnal variation = asthma
- very rapid onset and a/w sharp chest
ch est pain = pneumothorax

(h) night sweats = ask about LOA, LOW, fatigue


fati gue

2. Contact and travel history

3. Aetiology

4. Complications

5. Systemic review

6. Current management in hospital

7. Has this happened before? What happened? Investigations done? Management?

Past medical history


1. DM, HPT, HCL, IHD, CVA, cancer, asthma, TB
2. If suspect asthmaallergic rhinitis, allergic conjunctivitis, eczema, food allergy, drug allergy
3. Previous hospitalizations
4. Previous surgeries

Drug history
1. Any known drug allergy
- If yeswhat kind of drug? Drug reaction (angioedema, anaphylaxis, urticaria)
2. Long-term medications
- For what medical conditions
- Type, length of use
- Dosage, frequency of dosing
- Side-effects
- Compliance with use

- Respiratory drugs = steroids, bronchodilators


- OCP = pulmonary embolism
- Amiodarone = pulmonary fibrosis
- Cytotoxics (methotrexate, cyclophosphamide) = interstitial lung disease
- NSAIDs/B-blockers = bronchospasm
- ACE inhibitors = dry cough

3. TCM use
Social history
1. Smoking (20 cigarettes/day for 1 year = 1 pack year)
2. Alcohol
3. Occupational history = exposure to dust/animals
duration of exposure
use of protective devices

do other workers have similar symptoms?


Improvement over the weekends or off-days?
4. Family set-up = main caregiver, health of family members, fiancés
5. Lift-landing
6. Functional status

Family history
1. Asthma, AR, allergic conjunctivitis, food allergy
2. Bronchial carcinoma
3. TB
Medicine (CVS) = Angina Pectoris
Pathogenesis
- Imbalance between myocardial oxygen supply and demand

Oxygen supply Oxygen Demand


Duration of diastole Heart rate
Coronary perfusion pressure Blood pressure
(aorta DBP – coronary sinus DBP)
Coronary vasomotor tone Myocardial contractility
Oxygenation Left ventricular hypertrophy

- Causes
 Reduced coronary blood flow = atheroma, thrombosis, embolus, vasospasm, arteritis
 Decreased oxygenation = anaemia, CO poisoning, V/Q mismatch
 Increased myocardial demand = ventricular hypertrophy, HOCM, thyrotoxicosis
- Accumulation of metabolites from ischaemic muscles -> stimulate cardiac sympathetic nerves -> pain
(patients with cardiac transplants who develop CAD do not feel angina as heart is denervated)

Types of angina
1. Typical angina = Central crushing chest pain
Triggered by stress (emotional, exertion)
Relieved by rest
2. Decubitus angina (severe coronary disease) = on lying down
3. Nocturnal angina (critical coronary disease) = vivid dreams at night
4. Prinzemetal/variant angina = rest angina triggered by coronary vasospasm
Higher frequency in women
5. Unstable angina = angina of recent onset, at rest or change in character/worsening symptoms (frequency)

Pathology
- Stable angina = ischemia due to fixed atheromatous stenosis of 1 or more coronary arteries
- Prinzemetal angina = ischaemia due to coronary vasospasm
- Unstable angina = ischaemia due to plaque rupture with superimposed thrombosis (dynamic
obstruction)

Risk factors

Modifiable Non-modifiable
Smoking Gender (men)
- Risk of Ami same as non-smokers
after 2 years
- Risk of angina same as non smokers
after 10 years
Alcohol Ethnicity (Indians)
Obesity Age (older)
Physical inactivity Family history of IHD/AMI
Hypertension Personal history of IHD/AMI
Hyperlipidaemia Homocysteinaemia
Diabetes mellitus
244
Clinical Presentation
History
1. Chest pain
- Location = central substernal chest pain
- Character = crushing
- Radiation = left jaw and arm
- Duration = 5-10mins (AMI -> 30mins)
- Triggers = exertion, cold exposure, emotional stress, palpitations
- Relieved = rest, GTN
- ? recent changes in character of pain
2. Exertional dyspnoea (due to elevated end-diastolic pressure 2ndary to ischaemia
- Not a/w orthopnoea and PND
- ? decrease in effort tolerance
3. Intermittent claudication
4. No symptoms of nausea, vomiting, diaphoresis, giddiness/syncope, ankle oedema, fatigue
5. Triggers = anaemia -> recent BGIT/menorrhagia/gross haematuria
Recent illness/sepsis
Hyperthyroidism -> palpitations, fidgety, insomnia, increased appetite, LOW, diarrhoea
6. Risk factors
- Hypertension
- Hyperlipidaemia
- Obesity
- Smoking
- Alcohol
- Sedentary lifestyle
- Family history = 1st  degree relatives (women <65 years old; men<55 years old)
- Personal history

Physical Examination
1. General inspection = obese
Signs of thyrotoxicosis (goitre, thyroid eye disease)
2. Peripheries = anaemia, tar stains, xanthoma, xanthelesma, carotid bruit
3. CVS = cardiomegaly, valvular heart disease, cardiomyopathy
4. Lower limbs = signs of PVD

Investigations
Immediate
1. ECG
- Normal
- Previous AMI (pathological Q waves, LBBB)
- Ischemia (T wave invesion, ST segment depression)
- Left ventricular hypertrophy
2. Cardiac enzymes = not raised in stable angina
3. CXR = cardiomegaly
4. Underlying conditions
- FBC = Hb (anaemia); WBC (leucocytosis)
- TFT = hyperthyroidism
Later
1. Stress test
- Exercise ECG = planar/down-sloping ST segment depression indicative of ischemia
- Dobutamine
245
- Nuclear medicine
- MRI
2. 2D-echocardiogram
- Left ventricular ejection fraction
- Valvular heart disease
3. MIBI perfusion scan
4. MUGA functional scan (multiple gated acquisition scan)
5. Coronary angiogram (delineate exact coronary anatomy in patients going for revascularisation)

Management
Acute
1. Stabilise patient if necessary
- Ensure patent airway
- Ensure spontaneous breathing -> Give supplemental O2
Place on SpO2 monitoring (keep SpO2 >95%)
- Ensure good circulation -> Obtain vitals
Obtain ECG
Place on continuous ECG monitoring if necessary
2. S/L GTN
- MOA = relieves coronary vasospasm & pulmonary congestion; vasodilation
- Absolute contraindications = hypotension; tachycardia (SBP<90mmHg)
3. B-Blockers
- 1st  line therapy (not GTN)
4. Aspirin
- MOA = anti-platelet
- Contraindications = anaemia; BGIT; Asthma

#other anti-platelets (e.g. clopidogrel, ticolpidine, Gp2b/3a inhibitors) given only when patient is going
for interventional procedures
5. ACE inhibitors
6. CCB (if pain is not relieved by above measures)

Long term
1. Patient education
2. Control risk factors
- Lifestyle modifications = quit smoking, drink less alcohol, exercise regularly, lose weight, healthy diet
- Hypertension
- Hyperlipidemia
- Diabetes
3. Medical Treatment
- Symptomatic relief = S/L GTN
- Prophylaxis
 Anti-platelet therapy = aspirin 75-150mg OM/Clopidogrel 75mg OM
 Anti-anginal therapy
 B blockers = atenolol 50-100mg *drug of choice in previous AMI)
 LA nitrates = ISMN (vasodilatation, relaxes coronary arteries)
 CCB = amoldipine (vasodilatation, relaxes coronary arteries, decreases contractility, slows
HR
 ACE inhibitors
4. Surgical treatment
- Percutaneous trans-luminal coronary angioplasty (PTCA)
 Ideal for a single and discrete lesion

246
 Use of balloon dilatation to relieve arterial obstruction (KIV stent placement ot prevent
re0obstruction)
 # stent coated with sirolimus -> prevents proliferation of endothelial fibroblasts -> reduces risk of
stenosis
 Effective symptomatic treatment for chronic stable angina
 No evidene that it improves survival
 Acute CX = occlusion of target vessel/side branch by thrombus or loose intimal flap -> ischemia
 Long term CX = re-stenosis
- Coronary artery bypass graft (CABG)
 Ideal for patients not suitable for PTCA or severe triple vessel disease
 Use of alternative arteries to bypass proximal stenosis
#left internal mammary artery (LAD) aka internal thoracic artery
Right internal mammary artery (RCA) aka internal thoracic artery
Reversed segments of saphenous veins
- Operative mortality = 1.5%

247
Medicine (CVS) = Ischaemic Heart Disease (History)

Name/Age/Race/Gender/Occupation
Date of admission

Presenting complaint
Symptoms
1. Chest pain
 Mode of onset
 Duration
 Frequency
 Sudden/gradual onset
 Constant/intermittent
 Progressively worse/better
 Site and radiation
 Character
 Pain score and severity
 Triggers = CVS  exertion (quantify), cold exposure, emotion, palpitation, rest
GIT  food
 Aggravating factors = RT  deep inspiration, coughing, movement of shoulders
GIT  alcohol, lying down
2. Dyspnoea
 Mode of onset
 Duration
 Frequency
 Sudden/gradual onset
 Progressively worse/better
 Severity
 Triggers = exertion (quantify and ?decrease in ET), emotion, rest
 Relieving factors = rest
 a/w orthopnoea and PND
3. Nausea/vomiting
Diaphoresis
Palpitations, giddiness, syncope
Ankle oedema
Intermittent claudication

Aetiology
1. Triggers
(a) Anaemia = chest pain, SOB, giddiness, palpitations, fatigue, pallor, BGIT, menorrhagia, gross haematuria
(b) Sepsis = fever
(c) Hyperthyroidism = goitre, fidgety, insomnia, increase in appetite, LOW, diarrhoea
2. History of recent trauma = pneumothorax, rib #
3. Fever, URTI, productive cough = viral myocarditis/pericariditis, pneumonia with pleurisy
4. Nausea, vomiting, heartburn, acid brash, water brash, epigastric pain, dysphagia = GERD

Complications

Systemic review

248
Management prior and during hospitalization

Has this happened before?


-describe prior episodes
-changes in character of pain

-investigations done = ECG, stress ECG, coronary angiogram


-PTCA/CABG done

Past medical history


-IHD/AMI, DM, HTN, HCL, CVA
-prior hospitalizations and surgeries
Drug history
-drug allergies
-current medications

Social history
-smoking
-alcohol
-diet
-physical activity
-family set-up
-main caregiver
-finances
-lift-landing
-functional status

Family history
-IHD/AMI, DM, HTN, HCL, CVA

249
Medicine (CVS) = Acute Coronary Syndrome (ACS)

 Acute coronary syndrome


 consists of
(a) unstable angina
(b) NSTEMI
(c) STEMI

Pathology
 Unstable angina = ischemia due to plaque rupture with superimposed thrombosisDynamic obstruction, no
myocardial damage
 Myocardial infarction = myocardial necrosis caused by acute occlusion of a coronary artery by plaque rupture
and superimposed thrombosis
 NSTEMI = subendocardial infarct
 STEMI = transmural infarct

Vascular territory of STEMI


Anterior = V1 – V4 Right coronary artery (RCA) supplies right ventricle, inferior and posterior heart
Septal = V3 – V4 Left anterior descending artery (LAD) supplies anterior + septum
Lateral = V5 – V6 Left circumflex artery (LCA) supplies left atrium and left ventricle
Inferior = II, III, AVF

WHO criteria of AMI


 Chest pain > 10 mins
 ECG changes = new BBB
ST elevation of > 2mm in 2 or more contiguous leads
Posterior AMI (ST depression in lead V1+V2)
 Rise in cardiac enzymes

Clinical Presentation
History
1. Chest pain
-location = central substernal chest pain
-character = crushing
-radiation = left jaw and arm
-duration = prolonged (>30 mins)
-triggers = exertion, cold exposure, emotional stress, palpitations
# DM and elderly patients can present atypically  epigastric pain
Painless AMI
2. Dyspnoea (due to heart failure or elevated end-diastolic pressure)
3. Nausea, vomiting
Diaphoresis
Giddiness, syncope
Ankle oedema
4. Triggers = anaemia  recent BGIT/menorrhagia/gross haematuria
Recent illness/sepsis
Hyperthyroidism  palpitations, fidgety, insomnia, increased appetite, LOW, diarrhoea
5. Risk factors
- Hypertension
- Hyperlipidaemia
250
- Obesity
- Smoking
- Alcohol
- Sedentary lifestyle
- Family history = 1st  degree relatives (women < 65 yrs old; men < 55 yrs old)
- Personal history

Physical examination
1. General inspection = dyspnoeic and tachypnoeic
sweating
2. Cardiogenic shock = altered mental state
Hypotensive
Thin and thready pulse
Pale, cool and clammy extremities
Reduced capillary refill time
Reduced urine output
3. Hands = tar staining, peripheral cyanosis
4. Face = xanthelesma, central cyanosis
5. Neck = raised JVP
6. Praecordium = cardiomegaly
Additional heart sounds (S3, gallop rhythm)
Systolic murmurs (new onset VSD, MR)
7. Lungs = bibasal inspiratory crepitations
8. Abdomen = tender hepatomegaly
9. Lower limbs = bilateral ankle pitting oedema
PR = BGIT

Management
Acute
1. Stabilize patient
- Ensure patent airway
- Ensure spontaneous breathing  Give supplemental O 2; place on SpO 2 monitoring (keep SpO 2 > 95%)
- Ensure good circulation  Obtain vitals (HR, BP, RR); Obtain ECG; place on continuous ECG monitoring if
necessary; create venous access and take bloods for investigations
- Resuscitate if patient is in cardiogenic shock (papillary muscle dysfunction/rupture, septal rupture, cardiac
tamponade)
# call cardiologist and CT surgeon
# start inotropic support = IV dobutamine/dopamine 5-20 g/kg/min
# catheterize patient to monitor urine output
2. IV morphine for pain relief
- Give with IV maxolon (anti-emetic)
3. Nitrates
- S/L GTN = relieve coronary vasospasm
- IV GTN = for ongoing chest pain, HTN and pulmonary congestion
Absolute CI: Hypotension (SBP <90mmHg), tachycardia
Relative CI: Inferior AMI with possible RV involvement
4. Aspirin 300mg stat followed by 100mg OM (can also give clopidogrel, ticlopidine, LMWH)
- Anti-platelet effect starts 1hr after ingestion
- Decreases mortality and re-infarction rate
- CI: asthma, anaemia, BGIT
5. -blocker  atenolol 100mg OM
251
Carvedilol (-blocker of choice for large infarcts)
- Reduces HR, BP and contractility
- To be given only if patient is euvolemic
- CI: asthma, COPY, complete heart block, severe heart failure, urinary retention, bradycardia
6. ACE inhibitors  captopril 12.5mg BD
- Given in suspected STEMIs
- CI: hypotension, CRF, bilateral RAS
7. CCB  amlodipine
- If pain is not relieved with above measures

Specific measures
1. STEMI
- Consider myocardial salvage therapy in those presenting <12 hrs from onset
(a) Thrombolysis = tPA or streptokinase
(b) Percutaneous transluminal coronary angioplasty
# preferred reperfusion strategy if performed promptly (door-to-ballon time < 90 mins)
# indications = anterior MI, inferior MI with RV involvement, cardiogenic shock in patients < 75yo

Thrombolysis PTCA
 Advantages  Rapid administration  Better clinical efficacy
 Widely available # superior vessel patency
 Convenient # reduced re-occlusion rates
 Does not require expertise  Less haemorrhagic risk
 Early definition of coronary
anatomy
# allows tailored therapy
# more efficient risk stratification

Disadvantages  Patency ceiling = vessel patency  Delay limits efficacy


restored in only 60 – 85%  Less widely available
 Less clinical efficacy  Requires expertise
# optimal reperfusion not
achieved in > 50% of patients
# higher rates of re-occlusion
 Haemorrhagic risk

 Algorithm for thrombolytic therapy


Selection Criteria

 Typical chest pain of AMI


 ST elevation of at least 1mm in at least 2 inferior ECG leads (II, III,
aVF) OR
ST elevation of at least 2mm in at least 2 contiguous anterior leads
 Less than 12 hours from onset of chest pain
 Less than 75 yo
Fulfilled criteria

252
Contraindications
Do not administer thrombolytics if the answer to any of the
 following is yes
 Suspected AD
 Previous CVA
 Known intracranial neoplasm
 Recent head trauma
 Other intracranial pathology
 Severe hypertension (BP>180/110)
 Hypotension (SBP<90)
 Acute peptic ulcer
 Acute internal bleeding
 Recent internal bleeding (<1 month ago)
 Recent major surgery (<1 month ago)
 Current use of anticoagulants
 Known bleeding diasthesis
 Prolonged CPR (>5 mins)
 Previous administration of thrombolytics
 Pregnancy
 Diabetic retinopathy
 LBBB on ECG

No contraindications

Streptokinase Recombinant tPA


 Most commonly used  < 50 yo
 Cost-effective  Anterior AMI
 Therapy of choice when risk of
intracranial haemorrhage is high
e.g. elderly patients

Risks of thrombolytics therapy


 Intracranial bleeding (1%) = age > 65, weight < 70kg, hypertension, use of tPA
 Streptokinase allergy (5%) = patients treated for the first time (esp those with recent strep
infection)
 Anaphylaxis (0.2%)
 Hypotension during IV streptokinase infusion (15%) = corrected by decreasing rate of
infusion and volume expansion

2. NSTEMI
(a) LMWH
-superior to UF heparin = no need to monitor PTT, greater bioavailability, less risk of bleeding
-can also use Gb2b/3a inhibitors = proven benefit in high-risk NSTEMI +UAP as well as PTCA for STEMI
(b) PTCA
-indications = high-risk patients (persistent ST-depression and/or raised trop-T)

253
General measures
1. Monitored in CCU for at least 2 days
-short term complications of AMI = arrhythmias, CCF, cardiogenic shock, pericarditis
-monitor vitals q4rly
-I/O chart
-fluid restriction < 1L/day
-low-salt diet
-stool softener

Investigations
Immediate
1. ECG
-a single ECG cannot rule out AMI
Serial ECGs can rule out STEMI but not NSTEMI
-unstable angina = normal, ST depression, T wave inversion
NSTEMI = ST depression, T wave inversion
STEMI = ST elevation of >2mm in 2 or more contiguous leads
New-onset BBB
Posterior AMI (ST depression in leads V1+V2)
-do right-sided ECG in inferior AMI to exclude concomitant RV infarct (GTN is contraindicated)

2. Cardiac enzymes (serial CE q8hrly)


- 1st  set of CE can miss up to 40-60% of AMI
- 2nd set of CE can pick up 98% of AMI

(a) Myoglobin = detected within 1-2 hrs, peak at 6-9hrs, normalized by 24-36hrs
-earliest marker to rise in AMI
Useful in ruling out AMI early (raised in nearly all AMIs at 6hrs)
Disadvantages = not specific for cardiac muscle (skeletal muscle injury, NM disorders, renal failure, IM
injections, strenuous exercise, post-coronary bypass surgery
(b) Creatine kinase (CK-MB) = detected within 4-6hrs, peak at 18-24hrs, normalized by 48-72 hrs
-serological gold standard of AMI
-disadvantages = not specific for cardiac muscle, false positive values in CRF patients (renal failure),
narrow diagnostic window, failure of total CK to rise to abnormal values in all AMI
-relative % index = CKMB/tota l CK x 100% (≥ 5% suggestive of AMI)
(c) Troponin T = detected within 4-6hrs, peak within 12-120 hrs, normalized by 10-14 days
-useful for late presenting AMI
-Prognostic indicator in unstable angina
-Specific for cardiac muscle
-false positive values in CRF and dialysis patients
(d) Troponin I = detected within 4-6 hrs, peak at 12-36 hrs, normalized by 7-9 days
-the most cardiac-specific marker
-no false positive values in renal failure patients
-prognostic indicator in unstable angina
-not very widely available
3. CXR
-cardiomegaly
254
-acute pulmonary oedema
-upper lobe diversion
-congestive cardiac failure
-Kerley B lines
4. FBC = Hb (anaemia can ppt AMI)
WBC (infection/sepsis can ppt AMI)
U/E/Cr
PT/PTT = esp if patient needs to go for interventional procedures later
GXM
5. 2D-echocardiogram = LV ejection fraction
Complications of AMI (VSD, MR)

Later
1. Stress test = identify presence of residual ischemia
- ECG
- Dobutamine
- Nuclear medicine
- MRI
2. MIBI perfusion scan
3. MUGA functional scan (multiple gated acquisition scan)
4. Coronary angiogram (delineate exact coronary anatomy in patients going for revascularization)

Long-term management
1. Patient education
2. Control risk factors
- Lifestyle modifications = quit smoking, drink less alcohol, exercise regularly, lose weight, healthy diet
-hypertension
-hyperlipidemia
-diabetes
3. Medical treatment
-symptomatic relief = S/L GTN
-prophylaxis
(a) anti-platelet therapy = aspirin 75-150mg OM
Clopidogrel 75mg OM
Ticlopidine
Gp 2b/3a inhibitors
(c) Anti-anginal therapy
# β-blockers = atenolol 50-100mg (drug of choice in previous AMIs)
# LA nitrates = ISMN (vasodilatation, relaxes coronary arteries)
# CCB = amlodipine (vasodilatation, relaxes coronary arteries, decreases contractility, slows HR)
4. Surgical treatment (choice depends on technical difficulty, patient’s condition)
(a) Percutaneous trans-luminal coronary angioplasty (PTCA)
-ideal for a single and discrete lesion
-use of ballon dilatation to relieve arterial obstruction (KIV stent placement to prevent re-obstruction) #
stent coated with sirolimus  prevents proliferation of endothelial fibroblasts  reduces risk of stenosis
-effective symptomatic treatment for chronic stable angina
-no evidence that it improves survival
-acute Cx = occlusion of target vessel/side branch by thrombus or loose intimal flap ischaemia
Long-term Cx = re-stenosis

255
(b) Coronary artery bypass graft (CABG)
-ideal for patients not suitable for PTCA or severe triple vessel disease
-use of alternative arteries to bypass proximal stenosis
# left internal mammary artery (LAD)
Right internal mammary artery (RCA)
Reversed segments of saphenous veins
-operative mortality = 1.5%

Complications
Early
 Arrhythmia
-most common complication due to formation of re-entry circuits at junction of necrotic and viable
myocardium
-sudden death, VF, AF, heart block, bradycardias
 Contractile dysfunction
-CCF
-LVH with pulmonary oedema
-cardiogenic shock
-papillary muscle dysfunction  valvular regurgitation
 Extension of infarct
 Rupture
-rupture of papillary muscle (D3)  severe MR
-rupture of septum  VSD
-free wall rupture (D10)  haemopericardium  cardiac tamponade
 Pericarditis (D3)
-onset of different pain = position, worse on inspiration
-pericardial rub and pericardial effusion may be present
-Dressier’s syndrome (post -MI syndrome) = persistent fever, pericarditis, pleurisy
# Mx = wait and see
High-dose aspirin, NSAIDs, steroids
 Mural thrombus  embolus

Late
 Ventricular aneurysm
-due to bulging of non-contractile fibrous myocardium during systole
 Recurrent AMI

Prognosis
 Prognostic indicators = age of patient, extent of infarct, residual LV function
 50% mortality within 24 hrs of onset (25% die before arriving at the hospital)
 40% mortality within the 1 st  month
 1st  year survival rate = 80%

256
Medicine (CVS) = Congestive Cardiac Failure (CCF)

Definition
 Structural or functional heart disorder that prevents adequate cardiac output for tissue perfusion OR only at an
elevated filling pressure
 Acute heart failure = present de novo acute decompensation of chronic heart failure

Epidemiology
 Accounts for 4.5% of all hospital admissions and 2.5% of overall mortality in the elderly
 Common condition = lifetime incidence is 20%
 No gender predilection
 Main risk factors
 coronary artery disease (DM, HCL, obesity, smoking)
 hypertension
 valvular heart disease
 cardiomyopathy
other risk factors = previous AMI, arrhythmias, family history
 poor prognosis = many die suddenly due to malignant ventricular arrhythmias or AMI

Pathophysiology
 arises from either systolic or diastolic dysfunction
(a) Systolic
- Reduced systolic dysfunction leads to 4 compensatory mechanisms
1. Increase pre-load by activating RAAS. # Starling’s law = cardiac output depends on preload (EDV),
afterload (arterial resistance) & myocardial contractility
2. Increase pre-load by ADH release
3. Sympathetic activation by releasing catecholamines
4. Local changes = ventricular hypertrophy (pressure load), ventricular hypertrophy and dilatation
(volume load)
(b) Diastolic
- Ischaemia  muscle fibrosis  decreased relaxation/elastic recoil or ventricle  elevated LV end-diastolic
pressure  decreased stroke volume
- Classically caused by hypertension and HOCM
- Normal LV ejection fraction
 causes of pulmonary and peripheral oedema
(a) high arterial pressures
(b) impaired renal perfusion  secondary aldosteronism  salt and water retention
- CCF causes increased venous pressure which transmits to renal venous system
- Decreased pressure gradient between renal arterial and venous system results in decreased renal
perfusion

 Aetiology
 Pump failure
(a) Heart muscle = coronary artery disease (ischaemiafibrosis), cardiomyopathy, myocarditis
(b) Restricted filling = constrictive pericarditis, cardiac tamponade, restrictive cardiomyopathy
(c) Inadequate heart rate = negative inotropic drugs (anti- arrhythmics, β-blockers), arrhythmias (fast AF,
heart block), post-AMI
 Excessive preload
(a) Fluid overload
(b) Regurgitant valvular heart diease = MR, AR
 Excessive afterload
257
(c) Right-sided
 LV failure (most common cause)
 Cor pulmonale
 Pulmonary stenosis
 Pulmonary embolism
(d) Left-sided
 Hypertension
 Aortic/mitral stenosis
 High-output states
(a) Severe anaemia
(b) Thyrotoxicosis
(c) Large AV shunts
(d) Pregnancy

Left-sided HF  Decreased LV output


 Increased LA or pulmonary venous pressure
 Acute ↑ = APO
 Gradual ↑ = reflex pulmonary vasoconstriction  pulmonary HPT
Right-sided HF  Decreased RV output
Biventricular  Causes = dilated CMP, IHD
HF
High-output HF
Systolic dysfn  Impaired myocardial contraction
 May be associated with diastolic dysfunction
 More likely in younger patients, history of MI, displaced apex beat, S3
gallop, cardiomegaly on CXR
Diastolic dystn  Defective diastolic filling due to decreased LV complianceimpaired LV
filling
Elevated left atrial and pulmonary venous pressures
 Causes = LVH due to HPT or IHD or HOCM
 Findings = LVH, dilated left atrium, normal ejection fraction
 More likely in older patients, history of HPT, thrusting apex beat, S4 gallop,
LVH on ECG

Clinical presentation
Name/age/ethnicity/gender/occupation
Drug allergy
Past medical history
Date of admission

History of presenting complaint


Symptoms
 Left-heart failure
(a) Dyspnoea = duration, triggers (exertional or at rest), effort tolerance, severity (NYHA classification),
aggravating and relieving factors, associated with orthopnoea and PND
(b) Chest pain, nausea/vomiting, diaphoresis, giddiness  recent AMI
(c) Palpitations (fast AF can trigger CCF) = giddiness and syncope
(d) Fatigue
 Right-heart failure = ankle oedema, abdominal distension, facial oedema, RHC pain (tender hepatomegaly 
cardiac cirrhosis  nutmeg liver)
 Hypoperfusion = giddiness, confusion, oliguria

258
Aetiology
 Recent AMI
 Sepsis = fever, RTI (productive cough), UTI (FUN, dysuria, haematuria)
 Anaemia = PR bleeding
 Non-compliance with fluid and salt restrictions
 Non-compliance with medications

Compliance

Systemic review

Management of current episode

Has this happened before?


 When was CCF first diagnosed?
- Presenting complaint
- Investigations done = ECG, treadmill ECG, 2DE, nuclear scans (MIBI/MUGA)
- Current management = follow-up with whom? Compliance? Medications (type, dosage, recent changes,
compliance), fluid and dietary restrictions (compliance)
- Level of control = number of relapses? Treatment

Past medical history


 DM, HPT, HCL, IHD, AMI, CVA
 Valvular heart disease
 Previous admissions and surgeries

Drug history

Social history
 Smoking
 Alcohol
 Family set-up
 Main caregiver
 Lift-landing
 Type of housing
 Financial status
 Functional level

Family history

Physical examination
 Vitals = HR, RR, BP, T, SpO2
 General condition = mental state, anasarca, respiratory distress (tachypnoea, dyspnoea, use of accessory
muscles of respiration, pursed lip breathing, intercostals/subcostal retractions, cyanosis), midline sternotomy
scar with saphenous vein harvest site (previous CABG)
 Peripheries = pulse (tachycardia, AF, weak and thready), cold and clammy skin, prolonged capillary refill time,
conjunctival pallor
 Signs of fluid overload = facial oedema, raised JVP, pleural effusion/pulmonary oedema, tender hepatomegaly,
ascites, sacral oedema, bilateral lower limb pitting oedema
 Praecordium = displaced and heaving apex beat, S3/4 heart sound, gallop rhythm, heart murmurs (valvular
heart disease)

259
Differentials
1. Pulmonary embolism
2. Fluid overload = renal (nephrotic syndrome, ESRF), GIT (liver cirrhosis, protein-losing enteropathy, IBD)
3. Cor pulmonale
4. COPD

Investigations
Heart failure is principally a clinical diagnosis!
Bloods
 Cardiac enzymes = AMI
 FBC = Hb (anaemia), WBC (sepsis)
 U/E/Cr = hypokalaemia from RAAS activation and K +-losing diuretics, hyponatraemia (fluid overload), renal
impairment from hypoperfusion
 LFT = hepatic congestion, cardiac cirrhosis
 Serum NT-pro-BNP
- Peptide hormone secreted by ventricular myocytes  play key role in volume homeostasis
- Plasma concentration reflects ventricular pressure  raised in heart failure
- Actions = increases GFR, decreases renal sodium reabsorption
- High negative predictive value = useful in excluding diagnosis of heart failure in patients with dyspnoea/
fluid retention
 ABG (if SpO2 < 92%)
 Blood c/s if in sepsis

Imaging
 ECG
- MI(old infarcts  pathological Q waves; new infarcts  ST hyperacute changes)
- Arrhythmias (AF, heart block)
- LVH
- Goldberg’s triad for dilated CMP (poor R progression, small limb voltages, large chest voltages)
- Electrical alternans (cardiac tamponade, pericardial effusion)
 CXR
- Cardiomegaly, upper lobe diversion, peri- hilar bat’s wing shadow (alveolar oedema), Kerley B lines
(interstitial oedema), pleural effusion, pneumonia
 2D-echo
- Assess cardiac morphology
- Global and regional function
- Identify causes of heart failure (myocardial, vascular or pericardial origin)
 Identify underlying ischaemia and myocardial viability  revascularization
(a) 2D-echo = treadmill, dobutamine
(b) Radionuclide studies = MIBI (perfusion), MUGA (multiple gated acquisition scan)  functional
(c) MRI = useful for quantifying myocardial necrosis, perfusion and function; usually indicated in cardiac
masses, complex congenital heart disease or pericardial disease
(d) CT = calcifications (coronary artery, pericardium)
(e) Coronary angiogram and cardiac catheterization = indicated in patients with angina, history or MI or at
high risk for coronary artery disease.

 Acute management
Acute decompensation of chronic heart failure
 Stabilize patient’s vitals
(a) Secure airway if unconscious
(b) Ensure that patient is breathing spontaneously
- Place on continuous pulse oximetry monitoring
260
 Coronary revascularization arteries
 Congestive heart failure  Haemorrhages or exudates
Vascular disease  Papilloedema
 Dissecting aneurysm Atherosclerosis
 Symptomatic arterial disease / PVD  Ultrasound or radiological evidence of
atherosclerotic plaque (carotid, iliac, femoral
and peripheral arteries, aorta)

Risk assessment

Besides the level of BP, it is also important to assess the overall cardiovascular risk of a patient prior to definitive
therapy in order to optimize risk-benefit ratio. Adding the numbers of traditional, documented risk factors in a
person is one such way. The use of well tested and accepted risk tables, charts or formulae to estimate a patient’s
absolute risk is encouraged. In individuals such as those with known or established coronary heart disease (CHD),
atherosclerotic disease, diabetes mellitus, familial hypercholesterolemia or malignant hypertension, the overall
cardiovascular risk assessment may not be necessary as the risk is already high and treatment should be started as
soon as the diagnosis of hypertension is confirmed.

Risk stratification and treatment plan


BP Category Risk Group A Risk Group B Risk Group C
(No risk factors) (1-2 risk factors) (>3 risk factors or
Diabetes Mellitus or
TOD/ACC)
Systolic BP 130- LM LM LM + Rx
139mmHg/ Diastolic
BP 80-89mmHg
Systolic BP 140- LM + Rx* LM + Rx LM + Rx
159mmHg/ Diastolic
BP 90-99mmHg
Systolic BP >160 LM + Rx LM + Rx LM + Rx
mmHg/ Diastolic BP
>100 mmHg

*if BP control inadequate with LM alone


TOD = Target organ disease ACC = Associated Clinical Condition
LM = Lifestyle modification Rx = Drug therapy

Low risk Moderate risk High risk

265
Medicine (CVS) = Hypertension
Definitions
 Hypertension = 3 or more elevated BP readings taken on 3 or mo re different settings separated by at least 2
hrs

JNC (Joint national committee) Classification

Category Systolic BP (mmHg) Diastolic BP (mmHg)


Normal BP <130 <80
High Normal 130-139 80-89
Stage 1 HPT 140-159 90-99
Stage 2 HPT >160 >100
Isolated systolic HPT >140 <90

 Hypertensive crisis
o Hypertensive emergency
 Life-threatening and severe hypertension (diastolic BP ~ 120-130 mmHg) associated with
acute progressive end-organ damage
 Characterized by accelerated microvascular damage with fibrinoid necrosis in vessel walls of
small arteries and arterioles resulting in intravascular thrombosis
 Clinical features
# CVS = Hypertensive left ventricular failure (APO)
Acute aortic dissection
Acute myocardial infarction
# CNS = Stroke } needs to be differentiated as BP lowering is contraindicated to stroke
Hypertensive encephalopathy
# Renal = acute renal failure
#eyes = papilloedema
# eclampsia
 Requires prompt BP reduction (ICU setting)
o Hypertensive urgency
 Severe hypertension without acute end-organ damage
 Clinical features
 Hypertensive retinopathy
 Chronic renal failure
 Pre-ecampsia
 Accelerated hypertension = grade 3 hypertensive retinopathy
 Malignant hypertension = grade 4 hypertensive retinopathy
 BP should be reduced within 24 hrs (outpatient basis)
 Patients with accelerated/malignant hypertension should be admitted

266
Epidemiology
 Local prevalence rate = 24.9% (Singapore NHS 2004)
 Males > females
 Chinese > Indians > malays

 Aetiology
Primary Hypertension
 Accounts for 95% of the cases
 No underlying cause found
 Possible aetiology
o Increased sympathetic neural activity with enchances beta-adrenergic responsiveness
o Increased angiotensin II activity and excess mineralcorticoids
o Genetic factors = strong family history, ethnicity
o Environmental influences = obesity, smoking, excessive alcohol consumption, lack of physical
exercise, diet

Secondary hypertension
 Accounts for 5% of the cases
 Must investigate for the following causes in a young hypertensive patient (<40 years old)

Renal = Renal impairment (glomerulonephritis, diabetic nephropathy, analgesic nephropathy, chronic


pyelonephritis, APKD, obstructive uropathy, reflux uropathy)
Renal Artery stenosis (atheroma, fibromuscular dysplasia)

Endocrine = Cushing’s syndrome


Conn’s syndrome
Phenochromocytoma
Acromegaly
Thyroid disorders (primary hypothyroidism, thyrotoxicosis)
Toxaemia of pregnancy (pre-eclampsia, eclampsia)
Drugs (OCP, steroids, cocaine, amphetamines)

Neurogenic = Raised ICP


Obstructive sleep apnoea

Aortic = Coactation of aorta


Atherosclerosis

Labile = Pschogenic
Stress-related

267
Complications
 Cardiovascular
‐ Heart failure
‐ Ischaemic heart disease/coronary artery disease
‐ AMI
‐ Cardiac arrhythmias
‐ Peripheral vascular disease
 CNS
‐ CVA
‐ TIA
‐ SAH
‐ Hypertensive encephalopathy
 Renal
‐ Chronic renal failure -> hypertensive nephrosclerosis -> ESRF
# can directly cause renal failure eand accelerate disease progression
 Eyes
‐ Blindness
 Hypertensive crisis

History
Name/age/ethnicity/gender/occupation
Drug allergy
Past medical history

Presenting complaint 
Symptoms
 Cardiovascular = chest pain (radiating to the back), SOB, palpitations, ankle oedema, intermittent claudication,
fatigue, giddiness, nausea/vomiting, diaphoresis
 CNS = headache, nausea/vomiting, giddiness, blurring of vision, focal neurological deficits, seizures
 Renal = haematuria, oliguria/polyuria
 Eye = decreased visual acuity

Aetiology
 Renal
‐ Haematuria, proteinuria, polyuria/nocturia, flank pain, ankle oedema
‐ History of renal impairment
‐ GN = vascular (childhood rash on legs)
Infective (Hep B, Hep C, HIV)
Toxin (recent drug intake)
Autoimmune (rash, joint pain and swelling, SLE)
Metabolic (DM)
‐ History of DM
‐ History of long-term analgesia
‐ History of urinary stones causing obstruction
‐ History of APKD
‐ History of reflux disease (recurrent UTI)
‐ History of kidney infection
‐ History of renal artery stenosis
 Endocrine
‐ Conn’s = muscle weakness (hypokalaemic periodic paralysis)
‐ Cushing’s = weight gain around abdomen and face
‐ Pheochromocytoma = episodic headaches, palpitations, diaphoresis, postural giddiness
‐ Hypothyroidism = neck swelling, constipation, weight gain, fatigue, oligomenorrhoea, cold intolerant
‐ Thyrotoxicosis = neck swelling, diarrhea, polyphagia, LOW, palpitations, irritable, insomnia,
amenorrhoea, diaphoresis, heat intolerant
‐ Toxaemia of pregnancy = Last menstrual period
Symptoms of pregnancy
‐ Recent ingestion of OCP and steroids
 Neurogenic
‐ OSA = snoring, daytime somnolence
 Aortic
‐ History of coarctation of aorta
 Labile
‐ Stressed recently

Systemic review

Management prior and during admission

Has this happened before


 Duration of hypertension
 Initial presentation, investigations and management
 Follow-up with whom
‐ Frequency of follow-ups
‐ Compliance with follow-ups
‐ Annual investigations
‐ Level of control
‐ Home-monitoring system in place
 Conservative = weight loss, exercise, diet, compliance
 Medical therapy = types of drugs, dosages, side effects, recent changes, compliance
 Complications
‐ CVS = History of angina or AMI
Chest pain, Sob, palpitations
‐ Peripheral vascular disease = intermittent claudication, poor wound healing, pain, parasthesiae
‐ CNS = history of TIA/CVA/SAH
Headache, nausea/vomiting, BOV, focal neurological deficits
‐ Renal = history of CRF/ESRF
‐ Eyes = poor visual acuity, frequent DRP screening?
‐ Hypertensive emergency = history of such episodes
Presenting complaint, investigations and management

Past medical history


DM, HCL, IHD/CAD, AMI, CVA
Pre-eclampsia or eclampsia

Drug history

269
Social history
Smoking
Alcohol consumption
Occupation -> stressful
Diet
Physical activity

Family history

Physical examination
Blood Pressure
 Procedure
- Refrain from smoking or ingesting caffeine 30 mins preceding BP measurement
- Ensure that patient is well-rested and not anxious -> white coat hypertension
- Use appropriate cuff = bladder within cuff should encircle at least 80% of arm
- Place sphygmomanometer at heart leavel
- Measure BP in both arms at first visit -> Coarcation of aorta
Aortic dissection
Patent ductus arteriosus
Thoracic outlet syndrome
- Take 2 or more readings separated by 2 mins and obtain average measurement (obtain more
readings if differ by >5mmHg)
 Take BP in both standing and supine positions for elderly and DM
- Increase DBP on standing = Primary HTN
- Decrease DBP on standing = secondary HTN
Postural hypotension 2ndary to anti-hypertensive medications

General inspection
 Sallow appearance, AV fitula/tenchkoff catheter, bruises and scratch marks = ESRF
 Round-like facies, central obesity, violaceous abdominal striae = cushing’s syndrome
 Prognathism, frontal bossing, large hands and feet = acromegaly
 Café au lait spots = NF-1 (renal artery stenossis, pheochromocytoma, coarctation of aorta)

Peripheries
 Pulse
 Radio-radio delay and radio-femoral delay = coarctation of aorta

CVS examination
 Raised JVP
 Displaced apex beat – LV hypertrophy
 Mitral regurgitation
 S4 heart sound
 Bibasal inspiratory crepitations
 Peripheral oedema
 Carotid bruit

Abdomen
 Bilateral ballotable kidneys = APKD

270
 Adrenal mass = cushings
 Renal bruit = renal artery stenosis

Lower limb
 Neurological examination = deep tendon reflexes, focal neurological deficits
 Evidence of peripheral vascular disease = trophic skin changes, temperature gradient, capillary refill time,
pulses

Fundoscopy
Grade 1 = silver wiring of arteries (sclerosed vessel wall reduces transparency -> central light streak appears
broader)
Grade 2 = arteriovenous nipping
Grade 3 = flame-shaped haemorrhages
Soft exudates (cotton wool spots due to ischaemia)
Hard exudates (lipid residues from leaky vessels)
Grade 4 = papilloedema

Hypertensive crisis
History

Physical examination
 Mental state
 Takes BP on both arms
 CVS = heart failure, AR
 CNS = focal neurological deficits, confusion, coma, seizures
 Eyes = fundoscopy

Investigations
Bloods
 FBC
 U/E/Cr
 Cardiac enzymes

Urine
 Urine dipstick

Imaging
 ECG
 CXR
 CT head = hypertensive encephalopathy, stroke, SAH
 2D-scho/CT thorax = ? New onset AR (aortic dissection)

Causes
 Poor control of pre-existing hypertension = not detected
Inadequate treatment
Non-compliance with medications
 Secondary causes of hypertension

Management 
 Stabilize patient’s vitals
o Secure airway if patient unconscious
271
o Ensure patient is breathing spontaneously -> give supplymental O2, monitor pulse oximetry
o Obtain patient’s vitals (HR, RR, BP, SpO2)
oObtain Ecg and place on continuous ECG monitoring
o Set IV canula -> take blood for investigation
 2 most urgent indications for immediate BP reduction = Hypertensive encephalopathy
Aortic dissection
 Hypertensive emergency

Target
 Lower MAP by 20-25% or DBP to no less than 100-110 mmHg within a few hours
 Aim for 160/100 mmHg over the next 2-6 hrs

Sodium  First line treatment


nitroprusside  Contraindicated in pre-delivery eclampsia (use hydralazine instead)
 S/E = cyanide/thiocynate toxicity -> lactic acidosis, AMS
 Monitor patient closely if used

Labetalol  Indications = failure of nitroprusside


IHD (decreased HR & O2 demand)
Aortic dissection (decreased systolic ejection force & sheer
stress)
 Cortraindications = asthma, COPD, CCF, bradycardia, heart block

Esmolol  Indications
o Use with nitroprusside for thoracic aortic dissection
o Used with phentolamine (alpha – blocker) for pheochromocytoma
crisis

Nitroglycerine  Indications = Hypertension complicating unstable angina

Disposition
 Admid ICU/CCU

 Hypertensive urgency

Target
 Lower DBP to 100mmHg over 24-48hrs

Oral Felodipine
PO Captopril
Disposition
 If BP improves in monitored area -> discharge with review in next 1-2 days
 If BP does not improve -> admit
 Malignant/accelerated hypertension -> admit

272
Medicine (CVS) = Anti Hypertensive medication
1) Classes of anti-hypertensives
a) Ace inhibitors/angiotensin 2 receptor blockers (ARB)
b) B-Blockers
c) Calcium channel blockers
d) Diuretics

2) All drugs drop BP equally (SBP decrease 10-15mmHg)


a) Choice of drug not dependent on MOA but on clinical factors
b) Able to treat ~40% hypertensives successfully

3)  Ace Inhibitors
a) Rennin – angiotensin – aldosterone axis

Angiotensinogen

Renin

Angiotensin I

ACE (angiotensin – converting enzyme

Angiotensin II

Aldosterone Vasoconstriction

Ace also inactivates bradykinin (causes vasodilation/dry cough) -> vasoconstriction


273
b) Prototypes c) Advantages
i) Enalopril i) No dry cough of ACE inhibitors
- Pro-drug that is converted by de-esterification to ii) Renoprotective
elanoprilat (only IV use) iii) More selective than ACE inhibitors for angiotensin effects
ii) Captopril iv) More complete inhibition of angiotensin effects of ACE
- Competitive ACE inhibitor inhibitors
- Short T1/2 : TDS dosage
c) Side effects 5) B Blockers
i) Dry cough a) Mechanism of action = blocks B-adrenergic actions
ii) First – dose hypotension i) Negative inotropic and chronotropic effects
iii) Hyperkalemia & metabolic acidosis - Decrease Hr (decrease Av and SA conduction velocity)
iv) Macular rash - Decrease contractility – decrease cardiac output
v) Neutropenia - Vasodilatation
vi) Fetotoxicity - Increase diastolic time – increase coronary perfusion
vii) Nephropathy ii) Inhibits B mediated rennin release
- Reduces glomerular filtration pressure b) Contraindications
- Precipitates ARF in patients with impaired renal function i) Asthma/COPD
or RAS ii) Severe bradycardia
- Contraindicated if Cr > 300mmol/L iii) Complete heart block
- Check u/e/cr before and 1-2 weeks after starting Rx iv) Peripheral vascular disease
d) Advantages v) Diabetes
i) Very effective especially when given with diuretics c) Drug-drug interactions
ii) Renoprotective = prevents/reduces proteinuria i) Avoid verapamil and diltiazam = excessive negative inotropic
Stabilies renal function effect
iii) Extremely useful in Tx of heart failure ii) Effects decreased in presence of NSAIDS = reduced
iv) Greater fall in Bp in high rennin states production of prostaglandins (vasodilators)
iii) Metabolized by liver = increased concentrations if given with
4)  Angiotensin 2 Receptor blockers cimetidine
a) Mechanism of action = competitive inhibition of angiotensin 2 d) Side effects
receptors – G protein linked i) CVS = bradycardia, hypotension, syncope
i) Lorsartan ii) CNS = giddiness, irritable, hearing and visual disturbances,
ii) Irbesartan confusion - usually with chronic treatment
b) Side effects iii) GIT = nausea, vomiting, abdominal pain, constipation,
i) Hyperkalemia and metabolic acidosis diarrhea
ii) Nephropathy iv) DM = hypoglycemia, mask positive signs of hypoglycaemia
iii) Fetoxicity v) Asthma = bronchoconstriction
vi) Withdrawal syndrome after discontinuation of prolonged use iv) K+ sparing late distal tubules = aldosterone antagonists –
- Tachycardia spironolactone
- Angina
- Hypertension b) Carbonic anhydrase inhibitors
- AMI i) Acetazolamide (diamox)
ii) Proximal tubule = iso-osmotic reabsorption of water
6) Calcium channel blockers Secretion of H+ and organic anions
a) Classification Reabsorption of Na+
i) Dihydropyridine iii) MOA = inhibits carbonic anhydrase which catalyses
- Amlodipine (norvasc) dehydration of carbonic acid (H2Co3)
- Nifedipine (adalat) - Affects Na+/H+ exchanger
ii) Non-Dihydropyridine - Inhibits Na/Hco3 reabsorption
- Verapamil iv) Weak diuretic – limited clinical utility due to compensation
- Diltiazam by Na/Cl cotransporter
b) Mechanism of action v) Side-effects = hypokalemia
i) Arteriodilatation = reduces afterlaod Metabolic acidosis
ii) Decreases contractility CNS toxicity
iii) Negative inotropic/chronotropic effects (except amlodipine vi) Indications = glaucoma
& nifedipine) Epileptic seizures
c) Contraindications Acute mountain sickness
i) Heart failure
ii) Prolonged QT syndrome c) Loop diuretics
iii) Heart block i) Example = frusemide (lasix)
d) Side effects ii) Ascending limb of LOH = active NaCl reabsorption/water
i) Fluid retention impermeable
ii) Constipation iii) Most potent diuretic
iii) Vasodilatation = dizziness, flushing, headache iv) Mechanism of action = inhibits Na/K/2Cl cotransporter
Induces prostaglandin synthesis
7) Diuretics v) Side effects
a) Sites of action - Hyponatremia
i) Proximal tubule = carbonic anhydrase inhibitors - Hypokalemia
ii) Thick ascending limb = loop diuretics (frusemide) – inhibits - Hypochloremia
Na/K/2CL - Hypomagnesmia
iii) Distal tubule = thiazides – inhibits Na/Cl cotransporter - Calciuria – no hypocalcemia as Ca is absorbed in PCT

275
- Metabolic alkalosis ii) Short T1/2 = 10 mins
- Auditory and vestibular toxicity Bioactive metabolite (canrerone) has longer T1/2 = 15 hrs
- Venodilatation and hypovolaemia iii) Side effects
- Hyperglycemia - Hyperkalaemia
- Hyperuricaemia - Metabolic acidosis
- Hyperlipidemia - Anti-androgenic effects = gynacomastia/testicular
vi) Indications atrophy/menstrual disorders/hirstrusim
- CCF iv) Mild diuretic used together with other diuretics
- Oedema f) Osmolar diuretics
- Acute hyperkalemia and hypercalcemia i) Examples = mannitol, urea
d) Thiazides ii) Characteristics = small inert molecules that are filterable &
i) Example = hydrochlorothiazide non diffusible
ii) Distal tubule = active NaCl reabsorption/impermeable to Poor oral absorption
water Not metabolized
iii) Mechanism of action = inhibits NaCl cotransporter Excreted unchanged
Reduces peripheral resistance with Given IV
chronic use iii) Mechanism of action
iv) Side effects = hypo Na, hypo K, hypo Cl, hypo Mg - Act in segments that are freely permeable to H20 = PCT,
Reduced urinary Ca2+ descending limb
Metabolic alkalosis - Increases urine volume
Photodermatitis - Increases urine flow rate
Venodilatation and hypovolemia - Decreases contact time between fluid and tubular
Hyperglycemia, hyperuricaemia, epithelium
hyperlipidaemia - Decreases Na+ reabsorption
e)  Aldosterone antagonists (K+ sparing) iv) Side effects
i) Examples - Dehydration and hyper Na+ (inadequate hydration;
- Spironolactone excessive use)
- Amilonide - ECF volume expansion & Hypo Na+ (may cause Heart
Mechanism of action = decreases synthesis of aldosterone Failure)
sensitive proteins – involved in Na+/K+ ATPase & apical Na+ v) Indications
channels - Increased ICP
*aldosterone = Stimulates Na+/H+ exchange - Cerebral oedema
Stimulates K+ excretion
- Increased IOP
Acts on cytosolic and membrane
receptors
276
Medicine (CVS) = Guidelines for selecting drug treatment of hypertension

Concomitqant conditions Recommended drugs Contraindicated drugs


Heart failure Diuretics Calcium channel blockers
ACE inhibitors
Angiotensin II receptor blockers
Angina Beta – blockers
Calcium channel blockers
Post Myocardial infarction Beta-blockers
ACE inhibitors
Angiotensin II receptor blockers
Isolated systolic Hypertension Diuretic
Calcium channel blockers
ACE inhibitors
Angiotensin II receptor blockers
Diabetes Mellitus with Proteinuria ACE inhibitors
(micro or Macroalbuminuria) Angiotensin II receptor Blockers
Diabetes Mellitus ACE inhibitors Beta Blockers
Angiotensin II receptor blockers Diuretics
Calcium channel blockers
Post Stroke Diuretics
ACE inhibitors
Asthma & Chronic Obstructive Beta Blockers
Pulmonary Disease
Heart Block Beta blockers
Calcium channel blockers
Gout Diuretics
Bilateral Renal Artery Stenosis ACE inhibitors
Angiotensin II receptor blockers
Peripheral Vascular Disease B Blockers
Pregnancy ACE inhibitors
Angiotensin II receptor blockers
Medicine (CVS) = Lipids
1. Reference Ranges
Patients w/o preexisting cardiac risk Patients with preexisting cardiac risk
LDL <3.4 mmol/L LDL < 2.6 mmol/L
HDL > 1.0 mmol/L HDL > 1.0 mmol/L
Total <5.2 mmol/L Total < 4.1 mmol/L

2. Lipid Disorders plays a major role in pathogenesis of CHD


- Hypercholesterolaemia Highest In
 Clinically relevant risk of CHD begins with min TC = Malays > Chinese > Indians
3.9 mmol/L
Males > Females
 Escalates sharply when TC > 5.2 mmol/L
 Most impt is LDL – C
- HDL – C
 Powerful protective effect
 Low HDL – C independent risk factor for CHD
 Decrease HDL – C = Obesity/Smoking/sedentary lifestyle
 Increased HDL – C = exercise/alcohol intake
- Triglyceride
 Association with CHD not as well proven
3. Classification

Hypercholesterolaemia Increased LDL Increased Cholesterol


(Familial, Polygenic)
Mixed Dyslipedemia Increased LDL, VLDL Increased cholesterol & TG
(Familial, polygenic
Hypertriglyceridaemia Increased VLDL Increased TG
(TG > 4.5 mmol/L)
Severe Hypertriglyceridaemia Chylomicrons Increased TG
(TG > 10mmol/L)
*main CX = acute pancreatitis

*types of pipopriteins = Chylomicrons (transport dietary lipids to liver)


VLDL (transport TG from liver to tissues)
LDL (transport cholesterol from liver to tissues)
4. Secondary Dyslipidaemia

Diabetes mellitus Increased TG


Decreased HDL- C
Chronic renal failure Increased TG
Nephrotic syndrome Increased Total Cholesterol
Hypothyroidism Increased Total Cholesterol
Alcohol abuse Increased TG
Pregnancy Increased TG
Cholestasis Increased Total cholesterol
Drugs (diuretics, B-blockers, OCP, Steroids Increased TG
May increased Total Cholesterol
Decreased HDL - C
278
5. Lipid Measurements
- TC and HDL – C can be measured at any time of the day
- TG must be obtained after 10-12hrs of fasting
- Direct measurements = TC, HDL –C, TG
- Indirect measurements = LDL – C

Friedwald Formula
LDL – C = TC – [HDL – C + (TG/2.2)]
*formula cannot be used if TG > 4.5 mmol/L

6. Risk stratification
- Catagories
 Low-risk = 10 year CHD risk < 10%
 Moderate – risk = 10 year CHD risk 10-20%
 High risk = 10 year CHD risk > 20%

Algorithm

Step 1 = identify individuals in high risk category


 Established CHD
 CHD-like equivalents
(a) DM
(b) CVS
(c) PVD
(d) AAA

Step 2 = input number of risk factors


0 – 1 = Low risk
> 2 = calculate 10 year CHD risk score

Step 3 = > 2 risk factors


Estimate 10 year CHD risk score and re-stratify patients into low/moderate/high risk

7. Management
- Lifestyle changes
 Stop smoking
 Reduce weight
 Exercise regularly
 Dietary restrictions = reduce alcohol consumption if TG raised
(may encourage alcohol if HDL – C low)
- Medical therapy
 Recommend drug therapy

Hypercholesterolaemia Statin, ezetimibe


Mix dyslipidaemia Statin + fibrate/nicotinic acid
Hypertriglyceridaemia Fibrate =/nicotinic acid
Severe hypertriglyceridaemia Fibrate/nicotinic acid + omega 3 fish oils
Isolated low HDL – C Fibrate/nicotinic acid

279
 Pregnant = treatment only indicated in patients with severe hyper TG
Intensive dietary therapy = omega 3 fish oils

Statins (HMG Co-A Reductase inhibitors)


- Mechanism of action = lowers TC and LDL – C
- Most potent lipid lowering drug (decrease TC by 31-40 %)
 5mg rosvastatin = 10 mg atorvastatin = 20mg simvastatin = 40mg lovastatin = 80mg finvastatin

Side effects
- Transaminitis ( increase AST/ALT)
 Check LFT before and 2-3/12 after starting statin therapy -> annually
 Stop when AST/ALT > 3x upper limit of normal
 Restart at lower dose when liver function returns to baseline
- Myopathy and rhabdomyolysis
 Both likely to occur with high dosages of statins
#prescribe with caution in elderly, impaired renal function & when statin is combined with
fibrates/nicotinic acid
 Maybe due to depletion of mevalonate
 Stop when serum CK > 5x-10x upper limit of normal associated with muscle pain
 Severe rhabdomyolysis may precipitate ARF -> fatal
- Others = headache, nausea, vomiting, diarrhea, rash
#preferably given in evenings -> coincide with cholesterol biosynthesis
# Contraindications = pregnancy/lactating females
Children < 12 years old

Ezetimibe
- Mechanism of action = selectively inhibits intestinal absorption of cholesterol and related plant steroids

Bile acid binding resins


- E.g. cholestyramine
- Mechanism of action = binds bile acids and increases excretion
Increases cholesterol conversion to bild acids
Offset by increased intrahepatic cholesterol synthesis & up-regulation of LDL
receptors
# effect enhanced if given with statins
- Effective in lowering LDL – C & TC by 15-20%
#If combined with statins = 50%
- Only drugs that eliminate cholesterol from the body
- Infrequently used due to side-effects
 GIT = nausea, vomiting, constipation, steatorrhoea , sand like taste

Fibrates
- E.g. fenofibrate, gemfibrozil
- Mechanism of action = lowers VLDL and TG
Increases HDL – C
- Side effects

280
 Transaminitis
 Myopathy
 Gasllstone disease
- Usually started when TG > 4.5 mmol/L
- Gemfibrozil should never be combined with a statin***
- Principles of combination therapy
 Start the 2nd drug at a lower dosage & increase gradually until goal level achieved

 Avoid high doses of statins
 Monitor LFT & serum CK before and 6-8 weeks after initiation of therapy
 Advise patient to report to doctors if got muscle pain/tenderness/weakness

Nicotinic acid/Niacin
- Mechanism of action = lowers TC & TG
Increases HDL – C
- Side effects = intense cutaneous flush & pruritus over face & upper body
Nausea, vomiting, diarrhea, dyspepsia
Transaminitis
Hyperuricaemia Avoid in these
Hyperglycaemia patients

281
Medicine (CVS) = Myocarditis

 Aetiology
 Infections
o Viruses = coxsackieviruses A & B, adenoviruses, influenza, HIV, CMV
o Bacteria = diphtheria, meningococcus, Lyme disease, clamydia, rickettsia
o Fungi = candida
o Protozoa = Trypanosomiasis, toxoplasmosis
o Helminthes = trichinosis
 Immune-mediated reactions
o Post viral
o Post streptococcal
o SLE
o Drug hypersensitivity reaction = methyldopa, sulfonamides, doxorubicin
o Cardiac allograft rejection
 Unknown
o Sarcoidosis – non caseating granulomas
o Amyloidosis – amyloid protein depositions

Pathology
Gross morphology
 Acute onset -> heart usually of normal size (esp if patients die soon after onset)
 Chronic onset -> dilated chambers
 Flabby and pale myocardium with small areas of haemorrhage -> mottled appearance
 Endocardium and valves unaffected

Clinical features
History
 Range from asymptomatic to severe CHF
 Fatigue, dyspnea, chest pain, palpitations

Physical examination
 Signs of heart failure
 Tachycardia
 Soft S1
 S4 gallop

Investigations
 ECG = ST segment elevation/depression, T wave inversion, atrial arrhythmias, transient AV block
 Serology
 Endomyocardial biopsy

Management 
 Treat underlying cause
 Supportive measures

282
Outcomes/complications
 Outcomes = usually recover without sequelae
Some may develop intractable chronic CCF
 Complications = arrhythmias, dilated CMP, sudden death

283
Medicine (CVS) = Cardiomyopathy

Dilated cardiomyopathy
 Epidemiology = may present at any age (usually between 20-60 years old)
Most common form of CMP (90%)
 Inheritance = usually sporadic but some are familial
 Aetiology
- Post-viral myocarditis
- Alcoholism
- Toxins = cobalt, doxorubicin
- Peripartum CMP
- Genetic mutations involving cytoskeletal proteins
 Pathology
- Progressive cardiac hypertrophy and dilatation of all chambers
- Results in contractile dysfunction -> ineffective contraction (EF < 25%)
- Substantial dilatation and poor contractile function -> mural thrombus formation -> emboli
 Clinical presentation
- CCF
- Ventricular arrhythmias
- Thromboembolism
 Management
- CCF = diuretics, ACE inhibitors, nitrates, digoxin if hypotensive, anticoagulation
- Heart transplant

Hypertrophic obstructive cardiomyopathy (HOCM)


 Epidemiology = may present at any age
 Inheritance = 50% autosomal dominant; 50% sporadic
 Pathology
- Asymmetrical septal hypertrophy characterized by
a) Myocardial hypertrophy = thick, muscular
b) Increased myocardial contractility = powerful hyperkinetic contractions which rapidly expel
blood from left ventricle BUT ineffective as amount of blood in ventricle is greatly reduced
c) Decreased elastic recoil (stiff ) = impaired diastolic filling
 Clinical presentation
- Asymptomatic
- Symptomatic (usually in young adults)
a) Angina
b) Exertional dyspnoea
c) Exertional syncope cardiac arrhythmia (e.g. ventricular arrhythmia, WPW syndrome) ->
sudden death
d) Infective endocarditis
e) CCF
 Management
- Arrhythmias = B-blockers, CCB, amiodarone
- CCF = diuretics, ACE inhibitors, nitrates, digoxin if hypotensive

284
Anticoagulation
Implantable cardio-defebrillator, biventricular cardiac resynchronizing therapy
- Septal myomectomy

Restrictive cardiomyopathy
 Epidemiology = least common
 Pathology
- Primary decrease in ventricular compliance -> impaired diastolic filling
 Aetiology
a) Idiopathic
b) Infiltrative = amyloidosis, sarcoidosis, scleroderma
c) Radiation-induced fibrosis
d) Endomyocardial fibroelastosis
e) Endomyocardial fibrosis

285
Medicine (CVS) = Takayasu Arteritis

Definition
 Granulomatous vasculitis of medium and large arteries
o May result in fibrous thickening of aortic arch -> obliterate origins of distal branches
o Absence of pulses in upper extremities
 Unknown aetiology (? Immune mechanisms)

Clinical features

Demographics Asian females


40-45 years old
Symptoms Early = non specific (fatigue, weight loss, fever)
Vascular symptoms = markedly lower BP and weaker pulses in upper extremities
compared to lower extremities with coldness or numbness of
fingers
Ocular disturbances = visual defects, retinal hemorrhages, total blindness
Neurological deficits
Complications Involvement of root of aorta -> aortic regurgitation
Narrowing of coronary ostia -> AMI
Involvement of distal aorta -> intermittent claudication
Involvement of pulmonary arteries -> pulmonary hypertension
Renal artery narrowing -> RAS
Course Variable = may be slow or rapidly progressing

Management
 Symptomatic treatment
 Treat complications

Large vessels
Small vessel
 Takayasu’s arteritis
 Henoch–Schönlein purpura
 Giant cell arteritis
 Wegener’s granulomatosis
Medium vessels  Infective endocarditis
 Polyarteritis nodosa  Cryoglobulinaemia
 Kawasaki’s disease

286
Medicine (CVS) = Valvular heart disease

Murmur Location of max Diagnosis Signs


intensity
Mid diastolic Apex (heard with Mitral General:
-low-pitched bell) stenosis - Mitral facies (malar flush → purple cheeks)
-rumbling - Bruising (from warfarin anticoagulation)
Pulse:
- Small volume, slow-rising (anacrotic)
- Atrial fibrillation
Auscultation:
- MDM accentuated when patient turned to left lateral position/ exercise ( ↑flow)
- Loud S1 (occurs when leaflets are mobile, s lammed shut during ventricular
systole)
- Opening snap (opening of stenosed mitral valve, indicates pliable leaflets)
- Loud P2 (if pulmonary HPT present)
- Graham-Steell murmur (EDM; MS  pulmonary HPT  PR)
→ →

- Functional TR
Lungs – bibasal crepitations (pulmonary congestion)
BP – narrowed pulse pressure
Signs of RV failure – raised JVP, hepatomegaly, ascites, peripheral edema
Ejection Aortic area Aortic Pulse:
systolic Radiation to carotids stenosis - Small volume, slow-rising (anacrotic)
- Crescendo- (ddx: HOCM) Apex beat:
decrescendo - heaving, not displaced
- Palpable thrill over aortic area
Auscultation:
- ESM accentuated with patient sitting up in full experiation
- Usu a/w loud S2
Lungs – bibasal crepitations
BP – narrowed pulse pressure
Aortic area Aortic Pulse – normal
No radiation to sclerosis Auscultation: normal S2
carotids Innocent
Pulmonary area Pulmonary Auscultation:
stenosis - ESM accentuated with patient sitting up in full inspiration (pulmonary stenosis)
ASD - Fixed wide splitting of S2 (ASD)
Innocent
Apex Innocent
Aortic

287
Cannot make distinction between follicular adenoma and normal TSH + hormone-binding problems = pregnancy,
carcinoma (need to see encapsulating capsule, therefore open abnormal T4 increased thyroid-binding proteins,
biopsy needed) amiodarone
(inappropriately high may affect T3 and T4 levels but fT4/fT3 levels
8. Others T4) remain normal  TSH normal
Isolated TSH a) subclinical hyperthyroidism
ECG Hypothyroidism (bradycardia, small complexes suppression b) recovery from overt hyperthyroidism
in all leads) c) 1st -trimester pregnancy
Hyperthyroidism (sinus tachycardia, atrial d) drugs (dopamine, glucocorticoids,
fibrillation) somatostatin)
Isolated TSH a) subclinical hypothyroidism
CT If patient suspected of having retrosternal goitre elevation b) recovery from overt hypothyroidism
Staging of thyroid cancer c) drugs (amiodarone, lithium)
Do not use iodinated contrast agents → risk of
inducing hyperthyroidism

MRI staging of thyroid cancer

PET useful for patients on f/u with raised TG but no


focus of detectable clinical disease

Interpretation of tests
 TSH +  T4 TSH-secreting tumour
thyroid hormone resistance

 TSH +  T4 primary hypothyroidism

 TSH + normal T4 subclinical hypothyroidism

TSH +  T3/T4 primary hyperthyroidism


 TSH + normal T4 subclinical hyperthyroidism
T3 thyrotoxicosis
 TSH +  T3/T4 pituitary disease (pituitary tumours, post-
pituitary surgery, post-NPC radiation)
sick euthyroidism (in systemic illness;
typically for ‘everything to be low’)
301
Surgery (Thyroid) = Short Cases

Diffuse goitre
Mdm XXX is a middle-aged chinese lady who appears to be alert and comfortable at rest. She does not
appear restless. On general inspection, I note a diffusely enlarged goitre measuring __cm x __ cm in size
which moves with swallowing but not with tongue protrusion. It is non-tender, smooth, firm and not fixed
to the overlying skin or underlying muscles. There is no evidence of retrosternal extension, increased
vascularity or cervical lymphadenopathy. There is also no tracheal deviation or involvement of the carotid
arteries.

Mdm XXX is clinically euthyroid and there are no signs of thyroid eye disease.

So in summary, Mdm XXX has a diffuse goitre and is clinically euthyroid.

Multinodular goitre
Mdm XXX is a middle-aged chinese lady who appears to be alert and comfortable at rest. She does not
appear restless. On general inspection, I note an asymmetrically enlarged goitre which moves with
swallowing but not with tongue protrusion. On palpation, the thyroid gland is nodular with the right lobe
larger than the left lobe. It is non-tender, firm and not fixed to the overlying skin or underlying muscles.
There is no evidence of retrosternal extension, increased vascularity or cervical lymphadenopathy. There
is also no tracheal deviation or involvement of the carotid arteries.

Mdm XXX is clinically euthyroid and there are no signs of thyroid eye disease.

So in summary, Mdm XXX has a multinodular goitre and is clinically euthyroid.

Graves’ disease
Mdm XXX is a middle-aged chinese lady who is thin and restless on examination. On general inspection, I
note a diffusely enlarged goitre measuring __cm x __cm in size which moves with swallowing but not with
tongue protrusion. It is non-tender, smooth, firm and not fixed to the overlying skin and underlying
muscles. This is associated with signs of increased vascularity such as prominent dilated veins, increased
warmth, palpable thrill and bruit. There is no tracheal deviation or displacement of the carotid arteries. No
cervical lymphadenopathy was detected.

Mdm XXX is clinically hyperthyroid. I say this because she has fine tremors associated with sweaty and
warm palms. In addition, she is also in sinus tachycardia. However, there is no thyroid acropathy,
onycholysis, proximal myopathy, hyperreflexia or pretibial myxoedema. There are also no features
suggestive of thyroid eye disease.

So in summary, Mdm XXX most likely has Graves’ disease and is clinically hyperthyroid.

Thyroid nodule
Mdm XXX is a middle-aged chinese lady who appears to be alert and comfortable at rest. She does not
appear restless. On general inspection, I note a hemispherical swelling in the anterior triangle of the neck
which moves with swallowing but not with tongue protrusion. This is likely to arise f rom the thyroid
gland. On palpation, the nodule is non-tender, firm and not fixed to the overlying skin or underlying
muscles. There is no evidence of cervical lymphadenopathy.

Mdm XXX is clinically euthyroid and there are no signs of thyroid eye disease.

So in summary, Mdm XXX has a thyroid nodule and is clinically euthyroid.


Surgery (Thyroid) = Congenital anomalies

Lingual thyroid
 thyroid gland fails to descend patient presents with a lump at the foramen caecum
 asymptomatic OR interferes with speech and swallowing

Thyroglossal cyst
 usually occurs in young adults
 presents as a fluctuant swelling in the midline of the neck
 remnant tract left behind by the thyroid gland as it descends
from the foramen caecum to its position in front of the trachea
 tract usually reabsorbs → results in thyroglossal cyst or fistula
formation if it persists
 characteristics = moves on swallowing (attachment to larynx
by pretracheal fascia)
moves upwards when tongue protrudes out
(attachment to hyoid bone)
 Tx = Sistrunk’s operation (cyst, thyroglossal tract, body of hyoid
bone)
 Cx = infection  abscess formation
thyroglossal fistula
- presents as a discharging area in the midline
- follows rupture or inadequate excision of a thyroglossal
cyst  recurrent inflammation  fistula intermittently
discharges mucous
- Tx = fistulectomy
malignant change is rare
Medicine (Rheumatology) = Approach to the Rheumatological Case

GALS Screen
.
 Appearance Movement
Gait
 Arms
Legs
Spine

 History
1. Pain / Stiffness in muscle / joints / back
- Cardinal symptoms of rheumatic disease
2. Ability to wash and dress completely without difficulty
- ADL: Assessing functional problem of UL
3. Ability to get up and down stairs easily and ability to squat
- ADL: Assessing functional problem of LL
 Physical Examination
 Gait
1. Ask patient to stand
 Ease of transfer from chair / lying position to standing position
2. Get patient to walk and turn around
 Smoothness and symmetry of leg
 Pelvis and arm movement
 Normal stride length
 Ability to turn quickly
 Without pain
 Spine
1. Inspection (from back and from sides)
 Start from back
 Abnormal scoliosis curvature of spine
 Symmetry of paraspinal muscles and girdle muscles
 Symmetrical pelvic position, level iliac crests
 Inspect from sides
 Normal curvature in the neck and thoracic spine
 Normal lumbar lordosis
 Symmetry of paraspinal muscles
 From front
 Lateral cervical flexion ('Place your ear on your shoulder’)
 Hyperalgesic response of fibronyalgia (Squeeze over midpoint of supraspinatus
muscle)  If tender proceed to other sites, e.g. below medial epicondyle
2. Movement
 Put finger at spine along 2 lumbar vertebrae and ask patient to 'Bend forward and touch
toes'
 Finger to floor distance less than 15cm, lumbar expansion >6cm
 Arms
1. Inspection
 Arm:
 From front: Normal girdle muscle bulk and symmetry
 From back: Normal acromioclavicular sternoclavicular and glenohumeral joints; Full
elbow extension
Examine dorsal surface. Squeeze at carpal region
 No swelling, deformity
 Turn over to palmar surface.
 Supination movement (Arms and elbow)
 Palmar surface.
 Redness over palms in inflammatory arthritis, swelling, deformity
 Squeeze acoss MCP joint
 Early arthritis: pain and tenderness on squeezing before other abnormalities seen
2. Movement
 Make a fist. Spread out.
 Detection of power grip
 Pinch in pincer manner
 Impt for fine movements
 Spread elbows straight out
 Test shoulders. Put arms behind head and elbows back.
 Putting elbows back test full degree of external rotation
 Legs
1. Inspection
 Leg
 From front: No knee, forefooting or mid foot abnormalities
 Knee: Bulk of quadriceps muscle, normal concavities on each side of the patella (
concavities lost with effusion at knee joints)
 Feet
 Pay special attention to soles of feet. Deformity  change pressure points 
thickening of skin
2. Movement
 Bend leg up and twist
 Internal rotation first to go in hip disease
 Hand on knee joint feels for crepitus
 Look at knees and feel across lateral border
 Back of hand sensitive to temperature changes
 Squeeze across MTP joints
 Early arthritis: pain and tenderness on squeezing before other abnormalities seen

History
History of Presenting Complaint
 Pain
Aspect Differential
Distribution and Joint Acute monoarthritis
Involvement Septic arthritis, traumatic, gout/pseudogout, haemarthrosis
 N.B. arthralgia = presence of Chronic monoarthritis
joint pain wout swelling; Chronic infection e.g. TB, sero-ve spondyloarthritis,
arthritis = pain + swelling pigmented villonodular synovitis
Acute / chronic Acute polyarthritis
Getting better / worse Infection, onset of chronic polyarthritis
Chronic polyarthritis
RA, sero-ve spondyloarthritis, OA, gout, pseudogout, CTD e.g.
SLE, infection
Effect of exercise or rest  RA – symptoms worse aft rest
 OA – symptoms worse aft exercise
Sequence of onset of joint  RA/OA – symmetrical
involvement  Sero-ve spondyloarthritis – asymmetrical

 Approach to specific symptoms


Type Subset Characteristics
Mechanical Well localised
Aggravated by mvt, coughing, straining
Spinal cord lesion Pain in dermatomal distribution
Non-mechanical Osteoporosis Progressive and unremitting pain
Osteomalacia
Malignancy
 Metastasis
 Leukaemia
 Myeloma
 Limb Pain – Where, Onset, Aggravation
Type Subset Characteristics
MSK Polymyositis Aching pain in muscles ard region a/w weakness
Polymyalgia Older pts, pain with stiffness
rheumatica
Bone dz e.g. OM,
osteomalacia,
osteoporosis
Tenosynovitis
Vascular Arterial occlusion Acute. Severe pain of sudden onset
PVD Chronic. Calf pain on exercise relieved by rest
Nervous system Entrapment and
neuropathy
 Morning Stiffness
 Classically in RA and other inflammatory arthropathies
 Deformity
 Instability
 Described as “giving way” or “coming out”
 Due to dislocation or muscle weakness / ligamentous problems
 Change in sensation
 Functional capacity
 Systemic symptoms
 Fatigue
 Wt loss
 Ulcers
 Dry eyes & mouth
 Stiffness
 Fever
Past Medical History
 Treatment history + SEs
 H/o trauma or surgery
 H/o recent infection inc hepatitis, streptococcal pharyngitis, rubella, dysentery, gonorrhoea, TB
Social History
Family History

Physical Examination
 General Principles
 Impt in assessing pt’s functional disability and gaining clues about diagnosis
 Can get pt to transfer to side of bed / sit out in chair / expose and in doing so observe for
functional ability
 Look, feel, move, measure, compare with opposite side
- Look
 Compare left VS right
 Inspect front, back, sides
 Joint/bone: swelling, deformity, subluxation
 Muscle: wasting
- Feel
 Warmth
 Tenderness (say “please let me know if this is uncomfortable for you” ): Grade I – pt c/o
pain, Grade II – pt c/o pain and winces, Grade III – pt c/o pain, winces and withdraws,
Grade IV – pt does not allow palpation
 Synovitis (soft and boggy swelling)
- Move
 Passive movement
 Active movement
 Stability – tested by attempting to move joint gently in abnormal directions
 Crepitus
 Examination of individual joints
Examine this patient’s hands
 Introduction - Nails
 Sit the pt over the side of the bed and  Psoriatic changes – pitting,
place hands on the pillow with palms ridging, onycholysis,
down hyperkeratosis, discolouration
 General Inspection  Feel and Move
- Cushingoid - Wrist  MCPJ  PIPJ  DIPJ
- Weight  Synovitis, effusiuon, ROM,
- Iritis, scleritis etc Crepitus
- Obvious other joint disease - Dorsi/Palmarflex
 Look - Radial/Ulnar deviation
- Dorsal  palmar - Palmar tendon crepitus
- Wrist - CTS tests
 Skin  Joints/Bone  Muscle - Active mvt
- MCPJ - Wrist ext / flex
 Ulnar deviation (= deviation of - Thumb ext / abd / add / opp
phalanges at MCPJ towards - Fist (intrinsics)
medial (ulnar) side of hand), volar  Hand Function
(anterior) subluxation - Grip strength
- PIPJ, DIPJ - Key grip
 Swan neck (hyperextension at - Opposition strength
PIPJ and FFD at DIPJ), - Practical ability
boutonniere (FFD at PIPJ and  Others
extension at DIPJ), Z deformity - Elbow
(hyperextension of IPJ and FF and  Subcutaneous nodules
subluxation of MCPJ)  Psoriatic rash
 DIPJ – Heberden’s nodes  Other joints
 PIPJ – Bouchard’s nodes  Signs of systemic diseases

Examine this patient’s feet 


 Introduction - Subtalar joint
 Sit the pt over the side of the bed  Inversion and eversion
 General Inspection - Squeeze MTPJ
 Look  Tenderness  inflammation e.g.
- Skin: swelling, scars RA
- Bone/joints
 Deformity – hallux valgus,
clawing, crowding - Press upwards from sole of foot just
- Muscle proximal to the MTPJ of 3rd and 4th
- Nails toes
- Transverse and longitudinal arch - Feel and move each individual IPJ
 Psoriatic changes – pitting,  Pain in first MTPJ  acute gout
ridging, onycholysis,  IPJs typically affected in sero-ve
hyperkeratosis, discolouration aspondyloarthopathies
 Feel & Move - Palpate Achilles tendon for
- Ankle rheumatoid nodules
 Swelling around lateral and - Simmond’s test for Achilles tendon
medial malleoli rupture
- Talar joint - Tenderness at inferior aspect of heel
 Dorsi and plantar flex  Plantar fasciitis
.
o Gottron’s sign – Erythematous, violaceous smooth or scaly
Medicine (Rheumatology) = Dermatomyositis and Polymyositis patches over the dorsal I PJ, MCPJ, elbows, knees, medial
Definition malleoli
 A number of conditions in which muscles become damaged by a non- o Erythema spares the phalanges (cf SLE – phalanges involved,
suppurative lymphocytic inflammatory process knuckles spared)
 Polymyositis – inflammation of muscles, Dermatomyositis – skin + o Erythematous rash may be present on the neck and upper
muscles chest (often in the shape of a V), shoulders (shawl sign),
Etiology and Pathophysiology elbows, knees, malleoli
 Polymyositis – CD8 cell-mediated muscle necrosis, found in adults o Cuticlies may be irregular, thickened, distorted
 Dermatomyositis – B-cell and CD4 immune complex-mediated o Lateral and palmar areas of the fingers – rough and cracked
perifascicular vasculitis with irregular “dirty” horizontal lines, resembling those in a
mechanic’s hands
HISTORY   Neurological examination of the UL/ LL
 Adult form usually occurs after the 40 yo o Main findings are in the power testing
 Progressive symmetrical weakness of proximal muscles evolving over  Proximal muscle weaknes and tenderness of muscles
weeks/ mths (muscle wasting absent/ minimal)
o Difficulty in getting up from low chair/ squatting position  Weakness of neck flexors in 2/3 of cases
o Difficulty in climbing stairs  Intact/ absent deep tendon reflexes
o Lifting and running  Requests
o Inability to raise head (lift head off pillow) o If the patient > 40yo, tell the examiner that you would like to
o Inability to get up from bed look for an underlying neoplasm
 Dysphagia (due to weakness of the muscles of the pharynx)  Summary
 Dysphonia o Depends on the stem
 Muscle pain and tenderness o If asked to examine UL or LL , say that this is a ___ yo ___ who
 Raynaud’s phenomenon has a pattern of proximal myopathy on neurological
 Rash worsened by sunlight (photosensitivity) examination of the UL or LL, with weakness of shoulder
abduction/ adduction/ of power grade __. There is also muscle
PHYSICAL EXAMINATION (stem – usu examine this patients UL/ LL) tenderness etc.
 General inspection o In addition I also noticed cutaneous features suggestive of
o Cushingoid features? dermatomyositis as evidenced by the presence of _____.
 Skin o Functionally he/ she is able/unable to stand from the
o Heliotrope rash or purplish-blue rash around the eyes, back of squatting position/ get up from lying positon/ unable to
hands swallow (NG tube) etc etc.
o Dilated capillary loops at the base of fingernails
o Gottron’s papules – pink, violaceous, flat-topped papules
overlying the dorsal surfaces of the IPJ

309
CUTANEOUS MANIFESTATIONS

Heliotrope rash Periungual telangiectasia


SYSTEMIC MANIFESTATIONS AND COMPLICATIONS

Gottron’s sign on knuckles/ elbows

FIGURE 3. Shawl sign. FIGURE 4. Mechanic's hand.


Poikilodermatous macules Fissured, scaly, hyperkeratotic
appear in a "shawl" distribution and hyperpigmented hands are
over the shoulder, arms and suggestive of manual labor.
Gottron’s papules upper back.

310
INVESTIGATIONS  Resistant cases – methotrexate, azathioprine, cyclophosphamide,
 Confirm diagnosis cyclosporine, high-dose IVIg
o Serum CK – elevated. Levels mirrors disease activity  Malignancy surveillance (regular f/u)
 Also  in DMD, drugs (statins, chloroquine, o Detailed history and physical (breast, pelvic, rectal exam)
colchicines, px on chronic haemodialysis) o CXR, abdominal and pelvic u/s, FOBT, Pap smear,
o EMG – myopathic changes (spontaneous fibrillation, salvos of mammogram
repetitive potentials, short duration of polyphasic potentials of  Those with underlying neoplasm may remit after treatment of the tumour
low amplitude)
o Muscle biopsy – necross and phagocytoss of muscle fibres,
interstitial and perivascular infiltration of inflammatory cells. Classification of polymyositis-dermatomyositis (Bohan)
o Myositis specific auto-antibodies - the aminoacyl–transfer RNA Group I Primary idiopathic polymyositis
(tRNA) synthetases (anti-Jo-1), the nuclear Mi-2 protein, and Group II Primary idiopathic dermatomyositis
components of the signal recognition particle (SRP). Group III Dermatomyositis (or polymyositis) a/w neoplasia
o CXR Group IV Childhood dermatomyositis (or polymyositis) a/w vasculitis
o Serum aldolase, LDH, ALT, AST, FBC, ANA/ENA, U/E/Cr Group V Polymyositis (or dermatomyositis) with associated collagen
o Urinalysis, urine myoglobin vascular dz

DIAGNOSTIC CRITERIA FOR DERMATOMYOSITIS/ POLYMYOSITIS Classification of Dermatomyositis and Polymyositis


Criteria Description
1. Progressive symmetric prox Typical involvement of shoulders
muscle weakness and hips
2. Elevated muscle enzymes  CK, aldolase, LDH, AST, ALT
3. EMG changes Short polyphasic motor units, high
freq repetitive discharge,
insertional irritability
4. muscle biopsy Segmental fibre necrosi, basophilic
regeneration, perivascular
inflammation and atrophy
5. typical rash of dermatomyositis Required for dx of dermatomyositis
 definite poly/dermatomyositis if 4 criteria fulfilled Overlap syndrome
 probable if fulfil 3 criteria  Dermatomyositis overlaps with systemic sclerosis and mixed connective
 possible if fulfill 2 criteria tissue dz
o Signs
TREATMENT  Sclerotic thickening of dermis
 Steroids – most patients respond  Contractures
o Prednisolone is first line drug  Oesophageal hypomotility
 Microangiopathy
311
 Calcium deposits

Prognosis
 Dermato/polymyositis a/w malignancy
o  risk of malignancy if age > 50, DMY> PMY, normal CK,
refractory disease
o 2.4-6.5 fold  risk of underlying malignancy usu in internal
organs

DIFFERENTIAL DIAGNOSES of Dermatomyositis

Conditions associated with myositis


 Sarcoid myositis
 Focal nodular myositis
 Infectious polymyositis (Lyme disease, toxoplasma)
 Inclusion body myositis
 Eosinophilic myositis

Differential dx of proximal myopathy


 Endocrine – thyroid disorders, Cushing
 Drugs – TCM, steroids
 Infections
 Toxins – botox, NMBAs
 Autoimmune – polymyositis, dermatomyositis, SLE
 Electrolytes – hypo/ hyperkalaemic
312
- triggers = movement (OA), rest (inflammatory arthritis)
Medicine (Rheumatology) = History-taking - relieving factors = movement (inflammatory arthritis), rest (OA)

name/age/ethnicity/gender/occupation 4. Instability (sense of joint giving way)


date of admission - a/w locking of the knee?

Presenting complaint  5. Deformity


Symptoms - kyphosis
“Priscilla says stupid individuals don’t do NS” 
6. Disability
1. Pain - functional status = ability to dress, write, bath, transfer, feed
- mode of onset - require walking aids/splints
- frequency - ability to stand, walk , run and climb stairs
- duration - impact on social and recreational activities
- acute/sudden onset - mobility within home
- constant/intermittent - mobility outside home
- progressively worsening/improving
- site and radiation 7. Neurological symptoms
- character - numbness
- pain score and severity - parasthesiae
- triggers = movement (OA), rest (inflammatory arthritis) - weakness
- aggravating factors = movement (OA)
- relieving = analgesia, rest (OA), movement (inflammatory arthritis) Aetiology
Vascular = bleeding disorder, easy bruisability
2. Swelling Infective = fever, chills, rigors
- acute/gradual onset dysuria, urethral discharge, red eyes, recent URTI/GE (reactive
- history of trauma arthritis)
- frequency prolonged cough, haemoptysis, night sweats, LOA, LOW, malaise
- duration Trauma = history of trauma
- site Autoimmune = other joint involvement (see systemic review)
- associated with pain, redness and increased warmth Metabolic = h/o gouty attacks (gout)
- progressively worsening/improving Neoplasia = LOA, LOW, malaise, changes in urinary/bowel habit

3. Stiffness Complications (depends on underlying aetiology)


- duration of early morning stiffness = > 1hr (inflammatory arthritis), < 30
mins (OA) Systemic review
- site constitutional = LOA, LOW, fever, fatigue
313
anaemia = chest pain, SOB, palpitations, giddiness, fatigue smoker
RA = dry eyes, dry mouth, chest pain, SOB, numbness, parasthesiae, weakness alcoholic drinker
SLE = alopecia, headache, sudden weakness/numbness, photosensitivity, occupation
malar rash, oral ulcers, chest pain, SOB, changes in urine output, haematuria, family set-up and main caregiver
frothy urine, increased susceptibility to infection, petechiae, easy bruising financial status
Sero-ve spondyloarthritis = red and dry eyes, dry mouth, back pain, alternating type of housing and lift-landing
constipation and diarrhoea, bloody and mucoid stools,
recurrent abdominal pain Family history
Behcet’s disease = oral and genital ulcers SLE, RA, gout, TB
DM, HPT, HCL, IHD/AMI, CVA, cancer
Management prior and during admission

Has this happened before?


Describe initial presentation = symptoms, investigations, aetiology,
management
How has disease progressed over the years?
Regular follow-up = with whom? how often? compliance? yearly
investigations? what did specialist say at last visit?
Medications = what kind? compliance? side-effects? recent changes?
Non-pharmacological management = diet, PT/OT, intra-articular steroid
injections,
splinting, surgery
Complications of disease and management
Level of control = frequency of attacks? well in-between attacks? triggers?
management each time? number of admissions?
Current symptoms

Past medical history


DM, HPT, HCL, IHD/AMI, CVA, cancer, gout, TB, RA, SLE, AS, asthma
number of hospitalizations and surgeries

Drug history
any known drug allergies
current medications
steroids, aspirin, NSAIDs, warfarin

Social history
314
Medicine (Rheumatology) = Hand
Permission (& presence of Pain)
Position Sitting
Exposure Exposed elbows
Examination

The hand is a complex instrument with intricate function. There are 14 joints in the hand excluding the carpus.
The carpus itself is a complex articulation which comprises eight bones linked by ligaments which provide
three degrees of motion. The approach to examination of the hand is thus different and must be preceded by a
hand screen. The hand screen is still based on the basic principles of an orthopaedic examination, namely look,
feel and move.

Look

Skin
Swellings - ganglions and other lumps
Scars - surgical (esp carpal tunnel)

Bones & Joints


Deformities - rheumatoid hand
Alignment
Effusion - wrist joint effusion

Muscles & Tendons


Atrophy - thenar, hypothenar, dorsal interossei
Discontinuity

Nerves
Attitude - ulnar claw, median benediction
sign, wrist & finger drop

Vessels Venous
Arterial - atrophic skin changes
Lymphatic

Feel

Skin
Temperature - red, dry skin
Characterise swellings

Bones & Joints


(Ligaments) Bony outline
Tenderness - 1st  extensor compartment, anatomical snuffbox

Effusion

Muscles & Tendons


Subluxation
Tenderness - along flexor sheath

Nerve
Gross sensation

Vessel
Pulse - capillary refill
Pitting edema

Move

PROM & AROM


Flexion/ extension
Abduction/ adduction
Rotation

Special tests

Stability
Anterior/ posterior
Lateral

Impingement - Finkelstein’s

Fixed deformity

Neurovascular assessment

Pulses - radial & ulnar

Peripheral nerve
Motor - *see special tests for nerve
Sensory
Reflexes

Functional assessment

prehension
Hand Screen

1. Palms up

Zones
1. thenar- wasting, crease
scar
2. fingers- attitude
3. hypothenar- wasting
4. wrist- scars

2. Finger flexion

Zones
1. joint ROM
2. trigger
- S/E = radiation pneumonitis
pulmonary fibrosis

Small cell tumours


1. Chemo/RT

Palliation
1. Radiotherapy = bronchial obstruction
SVC obstruction
haemoptysis
dysphagia
bone pain
cerebral mets
2. Pleural drainage = cope loop, chest tube, pleurodesis
3. Pain relief = morphine
4. Discuss end of life issues = comfort measures / intensive resuscitation

Prognosis
 Overall 5-year survival = < 15%
 Non small cell without mets = 50% 2yr survival
 Non small cell with mets = 10% 2 yr survival
 Small cell (treated) = 1 year median survival
(untreated) = 3 months median survival
Medicine (Respi) = Infections  – Tuberculosis

Principles of Diagnosis
Know when to suspect tuberculosis.
Know how to make/confirm a diagnosis of tuberculosis on clinical, radiological and bacteriological grounds.
Principles of Management
Explain the basic principles of TB treatment, including the role of DOT, drug resistance.
Describe the common drugs required for treatment of TB including their significant adverse effects.
Describe how to monitor response to treatment.
Prevention
Discuss the public health aspects of Tuberculosis (including the basic principles of TB control), contact tracing,
treatment of latent TB.
Outline the pathogenesis of TB ie TB infection, TB disease, TB relapse.
Outline the protective effect of BCG and the use of the mantoux test.
Describe the basic epidemiology of TB in Singapore and globally.

Epidemiology
 Communicable disease
 Causes 6% of deaths worldwide, making it the most common cause of death from a single infectious
agent (WHO)
 Developing countries
 Incidence is increasing in developed countries as well, due to increasing prevalence of AIDS (most
impt risk factor for devpmt of TB) and migration
 Common in poverty stricken, overcrowded areas, malnutrition
 Common in those with chronic illnesses eg. DM, chronic lung disease, elderly or immunocompromised
(AIDS)
 Notifiable disease

 Aetiology
 Mycobacterium tuberculosis (M. bovis from unpasteurised cows’milk is rare)
 Transmission: direct person-to-person transmission via airborne droplets from an active case
(latent disease is not transmissible unless it reactivates in times of immsuppression)
 Pathogenesis:
- Mycobacterium enter macrophages  inhibit microbicidal activity  uncontrolled proliferation of
mycobacterium  bacteremia and seeding of multiple sites
- Recruitment of monocytes which differentiate into epithelioid histiocytes that characterise the
granulomatous response 
- Also results in delayed type tissue hypersensitivity (T lymphocytes)

Pathology
Primary tuberculosis
- Develops in previously unsensitised individuals
- Elderly persons may lose their sensitivity to MTB and hence develop primary TB more than once
- Source of organism is exogenous
- Bacilli deposit near the pleura proliferate in macrophages  form tubercles with caseous necrosis
(Ghon focus)
- Bacilli drain to the regional LN which also undergo caseous necrosis
(Ghon complex = parenchymal lesion + nodal involvement)
- Effective cell-mediated immune (CMI) response develops two to six weeks after infection
- Failure to develop CMI results in progressive destruction of the lung  progressive primary TB
Complications
1. Foci of scarring may harbour viable bacilli for years, and thus be the nidus of reactivation in times
of immunosuppression
2. Progressive primary  tuberculosis: disease develops without interruption in immunocompromised
individuals eg. AIDS patients with CD4+ counts <200/mm3
- Inability to mount immunological reaction to contain the primary focus
- Absence of characteristic caseating granulomas (non-reactive TB)
- Miliary TB: multiple tubercles evenly distributed thru’out the lung

Latent TB
- Stage inbetween primary and reactivation TB

Secondary tuberculosis (reactivation TB)


- Arises in a previously sensitised host, from reactivation of dormant bacilli when host resistance is
low (only 5% of those with primary disease develop secondary TB)
- Classically localised to the apex of one or both upper lobes (may be due to high oxygen tension)
- Due to hypersensitivity, bacilli excite a prompt and marked tissue response that tends to wall off
the focus (hence the regional LN are less prominently involved in early secondary TB compared to
in early primary TB)
- Cavitation occurs, erosion and dissemination along the airways  sputum positive, person can
spread the disease

Complications
1. Progressive pulmonary  tuberculosis: apical lesion enlarges, erodes into surrounding tissue
- Erosion into bronchus creates a ragged irregular cavity
- Erosion of bld vessels leads to hemoptysis
- Dissemination by blood or lymphatics
2. Miliary pulmonary disease
3. Pleural involvement: effusions, tuberculous empyema or obliterative fibrous pleuritis
4. Lymphadenitis: the most common form of extrapulmonary TB
- Typically occurs in the cervical region (“scrofula”)
5. Endobronchial, endotracheal and laryngeal TB
6. Intestinal tuberculosis
7. Pott disease: TB abscesses in the vertebrae (may spread along tissue planes to form “cold abscesses”
which present as a pelvic lump
8. Systemic military tuberculosis
- Hematogenous spread to other organs esp liver, bone marrow, spleen, meninges, adrenals,
kidneys  fatal without treatment

Clinical features
History
- Symptoms: low grade remitting fever, lassitude, anorexia, night sweats, chronic cough, hemoptysis,
pleuritic chest pain, erythema nodosum
- Symptoms of compression by lymph nodes eg. monophonic wheeze, bronchiectasis, lung collapse
- Symptoms of affected organ systems eg. Headaches and seizures for TB meningitis, paraplegia for Pott
disease
- Risk factors: contact/travel history, crowded living conditions, HIV/immunocompromise

Physical examination
- Consolidation in apices is possible
- Effusion
- Wheeze if there is compression

Investigations
 CXR
- cavitation in the apices of the lung
- calcification
- reticulonodular shadowing (for military TB)
- enlargement of hilar and mediastinal lymph nodes
- cavity with aspergilloma: air crescent sign
(CXR does not give indication of the activity of the disease; is not diagnostic)
 FBC
 LFT
 CRP
 Sputum AFB smear: MTB binds to Ziehl-Neelson stain and resists decolorisation (acid fast)
- positive AFB smear makes a presumptive diagnosis of TB in a high-risk patient, although a positive
stained smear is not specific for M. Tuberculosis
- 50% of AFB positive locals have MOTT (Mycobacteria other than TB)
- most AFB positive foreign workers have MTB
- if the patient is not able to produce sputum, sputum induction with nebulized, hypertonic 3% saline in
a negative-pressure isolation room is an alternative before more invasive procedures (bronchoscopy)
 Sputum culture is the gold standard (culture on Lowenstein Jensen media requires 12 wks; PCR can
provide faster results) * only culture can provide info on drug sensitivity
 Early morning gastric aspiration: most useful in young children where sputum is more difficult to
obtain, and is best performed following at least nine hours of fasting
 Nucleic acid amplification tests (NAAT), can provide rapid diagnostic information to the clinician,
generally within 24 to 72 hours
 Tuberculin skin test: TB antigen is injected intradermally and the cell mediated response at 48-72 hrs
is recorded. A positive test indicates that the patient has immunity (ie, previously exposed or
vaccinated) A strong positive test suggests active disease. False negatives occur in
immunosuppression eg. Miliary TB, AIDS
 In HIV patients, atypical features include sputum smear negative for AFB, false negative tuberculin test
cos of tuberculin anergy, lack of granulomas in tissues

Management
- Notify CDC, refer to TBCU
- Contact tracing
- Advise HIV testing
- Isolation while infectious
- Ishihara colour vision testing before initiating therapy with ethambutol
- Give anti-TB drugs (directly observed therapy to improve compliance) + monitor liver function
- Monitor CXR weekly during treatment, monthly sputum AFB smear and cultures till two consecutive
negative cultures
- Most persons diagnosed with TB are begun on specific treatment before the diagnosis is confirmed by
the laboratory

TB drugs
 Aims of therapy
- Successful treatment requires more than one drug to which the organisms are susceptible
- Sufficient dose
- Sufficient duration
- Compliance  DOT (polyclinic DOT)

 TB drugs
- First line: Isoniazid (H), Rifampicin (R), Pyrazinamide (Z), Ethambutol (E), Streptomycin (S), Amikacin,
Kanamycin
- Pyridoxine is given to reduce peripheral neuropathy induced by i soniazid
- Pyrazinamide is given for the first two months to kill intracellular bacilli
- 6 month treatment
- Titrate according to body weight
- Initial drug regimen is based on knowledge of the likely drug susceptibility.
- Four drugs are used in the initial phase of treatment when the total duration of treatment is six
months, because of the high incidence of isoniazid-resistant organisms in most communities.
 Drug resistant TB
 Initial drug regimens need to be modified in areas with a known high prevalence of MDR-TB
 Development of drug resistance after initial drug sensitivity (secondary drug resistance) occurs in
patients who do not comply with treatment regimens, occurs mainly in HIV patients
 Nosocomial transmission significant
 Use 4 drugs, treat for 2 years
 Follow up for 1 year after eradication
 Second line drugs: Ofloxacin, Ciprofloxacin, Cycloserine, Ethionamide, Azithromycin

Drug side effects


 Rifampicin
- induces liver enzymes  caution in drugs and OCP
- stop if liver enzymes are more than 3x elevated
- orange tears, sweat, sputum, urine
 Isoniazid
- peripheral neuropathy
- skin rash
- hepatitis  stop drug
 Pyrazinamide
- precipitates gout
- liver toxicity
 Ethambutol
- dose related optic retrobulbar neuritis, presents with colour blindness, central scotoma, reduction in
visual acuity
 Streptomycin
- irreversible damage to the vestibular nerve
- allergic reactions are more common

TB and HIV
 TB in an HIV patient is an AIDS defining condition
 4 drugs are used instead of the usual 3
 Adverse reactions are common and the prognosis is poor
 Multiple drug resistance occurs in 6%
 M. avium intracellulare is another mycobacterium that can cause pulmonary infection in AIDS patients

Prevention
 BCG vaccination: live attenuated vaccine  protects against miliary and meningeal TB
 Contact tracing
 Chemoprophylaxis for contacts and for HIV patients
Mantoux test
- used to identify patients with latent TB
- positive tuberculin skin test indicates infection with M. tuberculosis; it does not diagnose active
disease
- intradermal injection of 0.1 ml of PPD
- interpreted 48 to 72 hours after intradermal administration
- transverse diameter of induration should be measured and recorded in millimetres
- False negatives: newly diagnosed TB, HIV, TB meningitis
- Children who have received the BCG vaccine generally demonstrate PPD skin test reactions of 3 to
19 mm several months after vaccination. Most of these reactions wane significantly with time.
Responses indicative of a new infection include: > 10 mm induration in persons less than 35 years
of age or > 15 mm induration in >35 years old
Medicine (Respi) = Pneumonia

Definition
Clinical definition
Pneumonia = acute lower respiratory tract illness associated with fever, symptoms and signs in the chest,
and abnormalities on CXR
Pathologic definition
Pneumonia = inflammation of lung parenchyma characterised by consolidation due to exudate in
the alveolar spaces
[ Distinguish from pneumonitis: inflammation affecting the interstitium, which presents clinically
as “atypical pneumonia”]

Causative organisms
CLASSIFICATION BY ACQUISITION
 Community acquired typical (usually bacterial)  Pneumococcus, H influenzae
 Community acquired atypical  Mycoplasma, Legionella
 Nosocomial (usually bacterial)  Pseudomonas, Staph aureus
 Aspiration

CLASSIFICATION BY AGENT
 Bacterial: Strep pneumoniae
Staph aureus: usually 2o, following viral infection, IVDA (assoc w abscesses)
may be 1o in patients with underlying lung disease
H influenzae: usually 2o, following viral infection
most common cause of acute exacerbation of COPD
Klebsiella: affects debilitated patients esp chronic alcoholics
characteristic thick gelatinous sputum (redcurrant jelly)
Legionella: from water storage systems (chlorination and temp control impt)
affects debilitated patients
diarrhoea is a prominent symptom
Pseudomonas: nosocomial infection in patients with CF, on mech ventilation or
neutropenic (can cause pulm artery invasion and h’rhage/infarction)
Mycoplasma (causes atypical pneumonia)
 Viral: Influenza, CMV (cause atypical pneumonia)
 Fungal: Aspergillus: colonise cavities to form aspergilloma; invasive allergic reactions eg. asthma
Candida: causes lung disease in those with chronic lung disease/immcompromise
hematogenous spread
Cryptococcus: opportunistic infection
localised lesion in the lung which can spread to LN and then to other parts
Histoplasma: affects immcompromised, causes granulomatous inflammation
PCP: when CD4 count < 200/mm3 in AIDS patients
perihilar shadowing, dry cough
BAL and immunofluorescence
 Parasites: cause eosinophilic pneumonia: amoeba, paragonimus
 Drugs/chemicals: cause interstitial pneumonitis

 Aetiology
CLASSIFICATION BY SITE
Lobar pneumonia
 Etiology/epidemiology
- Causative agent usually Strep pneumoniae (90%), Kleb, Staph, H influenzae (high virulence)
- Can occur at any age in healthy people without underlying lung disease
- Usually follows viral infections
Pneumococcus is associated with rusty colored sputum
Bronchopneumonia
 Etiology/epidemiology
- Causative agents: Staph aureus, H influenzae, Strep, Pseudomonas (low virulence organisms)

 Predisposing factors
- Extremes of age
- Immunosuppression/immunocompromised eg. Chronic disease
- Loss of cough reflex eg. Coma, anaesthesia
- Injury to mucociliary apparatus eg. Smoking, viral disease, genetic disease (CF)
- Interference with phagocytosis or bactericidal action eg. Alcohol, smoking
- Splenectomy
- Pulmonary congestion eg. Cardiac failure
- Accumulation of secretions eg. Bronchial obstruction, prolonged bed rest
- In hospital (nosocomial infections)

 Atypical pneumonia
 Etiology/epidemiology
- Causative agents: mycoplasma, chlamydia, viruses (influenza, parainfluenza, RSV, adeno)
- Affects school going children and young adults
 Clinical features
- Presents as first as URTI eg. pharyngitis and flu-like symptoms  laryngitis  tracheobronchitis +
pneumonia (LRTI)
- May have headaches and malaise (typical of mycoplasma), erythema multiforme, arthralgia,
autoimmune haemolytic anaemia, myocarditis, hepatitis, DIC
- Cough, fever, modest sputum production, non-specific CXR changes (transient, ill-defined patches),
WBC count only moderately elevated, non-response to antibiotics
- Because the edema and exudatation are both in a strategic position to cause an alveolocapillary
block, there may be respiratory distress out of proportion to the physical and radiologic findings
- Cold agglutinins, rising antibody titre
 Complications
- ARDS

 Aspiration pneumonia
 Usually in the right middle lobe cos the right bronchus is straighter
 Especially in unconcscious, drunk, epileptic, stroke patients; may f ollow after gen anaesthesia, partial
drowning
 Gastric contents: can cause asphyxia if massive; can cause pulm edema + infection
 Necrotising pneumonia, pursues a fulminant clinical course
 Complications: lung abscess, death

Nosocomial pneumonia
 Common in patients with underlying disease, immsuppression, prolonged antibiotic therapy, invasive
devices/foreign bodies, mechanical ventilation
 Commonest causative organisms: Pseudomonas, S aureus and enterobacteriaceae

Clinical features
History
 Symptoms: Fever, rigors, cough, purulent sputum, malaise, dyspnea, pleuritic chest pain
 Diarrhea (legionella)
 Confusion in elderly
 Preceding viral illness
 Hospitalisation/insitutionalisation
 Smoking/alcohol
 Co-morbidities
Physical examination
 Fever, confusion (in the elderly), tachypnea, tachycardia
 Consolidation: diminished chest expansion, dull percussion note, increased vocal fremitus/resonance,
bronchial breathing
 Pleural rub
 Sputum mug

Differentials (non-infectious)
 Chemical pneumonitis / inflammation due to radiotherapy
 Allergic mechanisms, asthma
 Lung cancer
 COPD

Investigations
 FBC: leukocytosis with left shift, CRP
 Bld c/s
 Viral serology if suspected
 CXR: lobar/patchy consolidation (opacity with air bronchograms)
multicentric, likely hematogenous route  IVDA (Staph aureus)
cavitating: TB, anaerobic, kleb, meliodosis, staph aureus
+/- parapneumonic effusions
- CXR changes lag behind clinical course, hence initial CXR may not show typical changes
- CXR may show consolidation after resolution of symptoms, but should clear by 6 wks
 Sputum for microscopy and c/s, AFB smear and culture, TB PCR
- should have < 10 epithelial cells
- > 25 WBCs are abnormal
- lancet shaped diplococci = S pneumoniae
 Urine antigen for legionella, pneumococcus
 Bronchoscopy for immunocompromised patients
 Pleural fluid for analysis (thoracocentesis) if effusion present

Severity
 Two scoring systems to decide outpatient vs inpatient treatment; also of prognostic value
 CURB 65 score
- Confusion (abbreviated mental test score < 8)
- Urea > 7 mmol/L
- Respiratory rate > 30/min
- BP systolic < 90 mmHg
 0-1: treat as outpatient
 2: inpatient treatment
 3 or more: admit to ICU
 PSI (see attached)
- PORT study (Patient Outcomes Research Team)
- Risk class I: no co-morbidities, normal phy exam and age < 50
- Risk class II – V: points are assigned for different comorbidities and abnormal lab findings
 Direct ICU admission if patient is in septic shock requiring vasopressors or intubation

Management
 Outpatient antibiotic treatment: amoxicillin
 Hospitalised patients are generally begun on intravenous antibiotics (ceftriaxone + azi thro OR
levofloxacin). Patients who are improving clinically, hemodynamically stable, and able to take oral
medications can be switched to oral therapy.
 If no improvement within 72 hours, consider an organism that is not covered by the initial antibiotic
regimen, including unusual pathogens or drug-resistant organisms
 Oxygen: nasal cannula/venture mask/ventilation depending on severity
 IV fluids
 Analgesia
 Vaccines
 Chest physiotherapy
 Follow up CXR in 6 weeks

 Antibiotic selection
 Community acquired
- Mild Oral amoxicillin and/or erythromycin, or ciprofloxacin
- Severe IV augmentin or cefuroxime AND erythromycin
- Atypical Clarithromycin (Legionella), tetracycline (Chlamydia), bactrim (PCP)
 Nosocomial
- Gm negs, Pseudomonas, IV aminoglycoside + 3rd gen cephalosporin
IV ciprofloxacin for pseudomonas
Anaerobes IV metronidazole
** if TB cannot be ruled out then do not give quinolones as it may mask the AFB smear

Complications
 Complete resolution is rare in bronchopneumonia  focal fibrosis or bronchiectasis
 Pleural effusion
 Empyema
 Lung abscess
 Respiratory failure
 ARDS
 Sepiticemia
 Brain abscess
 Pericarditis
 Cholestatic jaundice

Causes of poorly resolving pneumonia


 Lung Ca
 Aspiration of foreign body
 Inappropriate antibiotic
Pneumonia Severity Index

Risk factors Points


Demographic factors
Age for men Age (yr)
Age for women Age (yr) - 10
Nursing home resident +10
Coexisting illnesses
Neoplastic disease (active) +30
Chronic liver disease +20
Congestive heart failure +10
Cerebrovascular disease +10
Chronic renal disease +10
Physical examination findings
Altered mental status +20
Respiratory rate ≥ 30/minute  +20
Systolic blood pressure <90 mmH +20
Temperature <35°C or ≥ 40°C  +15
Pulse ≥ 125 beats/minute  +10
Laboratory and radiographic findings
Arterial pH <7.35 +30
Blood urea nitrogen ≥ 30 mg/dL (11 mmol/L)  +20
Sodium <130 mmol/L +20
Glucose ≥ 250 mg/dL  +10
Hematocrit <30 percent +10
Partial pressure of arterial oxygen <60 mmHg or an
+10
oxygen saturation of <90 percent on pulse oximetry.
Pleural effusion +10

Class Score Management Mortality %


I 0 Outpatient 0.1
II <70 Outpatient 0.6
III 71-90 Inpatient to observe 2.8
IV 91-130 Inpatient 8.2
V >130 Inpatient 29.2
 Aetiology
Medicine (Respi) = Asthma  Extrinsic (allergic) asthma
Principles of Diagnosis - Definite external cause
Recognise acute asthma clinically and define status asthmaticus. - Type 1 hypersensitivity reaction
Describe the severe sequelae arising from inadequate acute management. - Elevated serum IgE and eosinophils
Describe the clinical assessment of severity of asthmatic effect. - Mediated by Th2 cells  release IL 4, IL5  IgE synthesis 
Principles of Management sensitization of mast cells
Describe the immediate steps in management: drugs, dosage and mode of - Triggered by allergen exposure in a sensitised individual  mast cell
administration. degranulation
List the clinical parameters which should be monitored after initiation of - Early phase (within 1 hr): release of leukotriene D4, PGE2, histamine
treatment. - Accompanied by reflex bronchoconstriction due to stimulation of
Describe the indications for intubation. vagal receptors
Explain the pathophysiology of acute asthma and the action of the drugs used - Late phase: recruitment of leukocytes: leukotriene B4, platelet
in acute management. activating factor, TNF
Describe the follow-up medical management after the acute attack. - Leukocytes damage the epithelium, reducing production of NO, thus
Prevention causing smooth muscle contraction
Describe the measures to minimize occurrence of severe asthmatic attacks. - Eosinophils perpetuate the inflammation
- Eg. Atopic asthma, occupational asthma, allergic bronchopulmonary
aspergillosis
Characteristics
- Often has family history of allergy/atopy
 3 characteristics:
- Develops in childhood
i) Airflow limitation of changing severity, episodic, reversible
spontaneously or with treatment
 Intrinsic (non-allergic) asthma
ii) Airway hyperresponsiveness to a wide range of stimuli
- Non immune mechanism, asthmatic “diathesis”
iii) Inflammation – eosinophils, T lymphocytes, mast cells, smooth muscle
- IgE antibodies are normal
hypertrophy, edema, mucosal damage
- Triggered by respiratory infection, aspirin, stress, exercise, cold
 3 factors contribute to airway narrowing: i) bronchospasm, ii)
(these factors may also trigger asthma in a person with extrinsic
inflammation and swelling,
asthma cos of his bronchial hypersensitivity)
iii) increased mucus production
- No family history of allergy/atopy
o Intervals between attacks are characteristically free from respiratory
- Develops later in life, “late onset”
difficulty, but subtle deficits can be detected by spirometry

Epidemiology
o Prevalence is increasing
o More common in developed countries

421
Pathogenesis
 Atopy Physical examination
- individuals who readily develop IgE antibodies against common  Tachypnea
materials in the environment  Wheeze, cough
- runs in the family  Hyperinflated chest, hyperresonance, diminished air entry
- childhood exposure to allergens has an influence on IgE production:  Polyphonic wheeze
growing up in a clean environment predisposes towards IgE response  Clinical severity: able to complete sentences? Silent chest, bradycardia,
to allergens PEF < 33%, cyanosis, feeble respiratory effort, confused
 Airway hyperresponsiveness
→Small airway obstruction due to bronchospasm and thick tenacious Investigations
mucous plugs, progressive hyperinflation with air trapping  FBC: eosinophils
 Remodelling  Sputum culture and c ytology: eosinophils
- deposition of matrix proteins beneath the epithelium  Serial peak expiratory flow measurements
- epithelial metaplasia and increase in goblet cells - Diurnal variation >20% on 3 days a week for 2 weeks: marked
- thickened basement membrane morning dipping of peak flow
- smooth muscle hyperplasia  Lung function tests
- Decreased FEV1/FVC ratio and increased residual volume
Precipitating factors - Before bronchodilator, after bronchodilator  variable airflow
 Environmental allergens: pollen, house dust mite, pets limitation, >15% improvement in PEFR after bronchodilator
 Occupational: isocyanates (varnish), flour, animals  CXR: hyperinflation, exclude pneumothorax or allergic
 Atmospheric: cigarette smoke, pollutants bronchopulmonary aspergillosis
 Cold air, exercise (at the end of exercise), emotion  Skin prick test: helps to identify allergens
 Viral infections: rhinovirus, parainfluenza  Histamine or methacholine challenge: to test airway
 Drugs: Aspirin (imbalance in metabolism of arachidonic acid), beta hyperresponsiveness
blockers

Clinical features
History
 Dyspnea (esp expiration), nocturnal cough, wheezing, chest tightness
 Severity, frequency
 Precipitants: Exercise, cold, stress, infection, drugs eg. aspirin
Allergens, occupational exposure, better when on holiday
 Diurnal variation: worse in the morning
 Other atopic disease: eczema, allergic rhinitis
 Family history
 Social history: occupation, impact on lifestyle

422
Management
 Allergen avoidance eg dust mite, smoking, drugs
Classification of asthma  Pharmacologic therapy – depending on severity/frequency of asthma
New GINA guidelines symptoms
- use step-up or step-down approach
 Education of patient and family:
- Check inhaler technique (avoid excess deposition in mouth)
- Use of spacer to increase lung deposition, decrease the need for
coordination
- Compliance with steroid inhalers
 Asthma action plan
- grades patient’s severity of asthma into the green, yellow and red
zones, according to their symptoms + peak flow rates
- describes the dose, frequency and duration of the appropriate
treatment
- main aim of the asthma action plan is to abort exacerbations by rapid
step up of both reliever and preventor
- also prompts the patient to seek urgent hospital treatment in case of
severe exacerbations and/or failure of self medication.
 SMART approach to asthma (Symbicort Maintenence and Reliever
Treatment)
o 2 puffs BD for maintenence
o 4 puffs BD in exacerbation for rapid relief
o 2 puffs BD when symptoms resolve
o Symbicort = budesonide + formoterol
- Better compliance when using a combined inhaler than two separate
inhalers

423
 Asthma drugs
 Beta-2 agonists
- selective for bronchial smooth muscle  relaxation and
bronchodilation
- for symptomatic relief (2 puffs prn)
- effective up to 6 hrs Status asthmaticus
- excessive use (>2 canisters per month) is associated with increased  Poor response to drug therapy after 24 hours
mortality  Signs of respiratory failure: ABG: PaCO2>6kPa, PaO2<8kPa, pH low and
 Anticholinergic (ipratropium bromide) falling
- muscarinic receptor antagonists
 Long acting beta-2 agonists Risk factors for death from asthma
- effective up to 12 hours  Past history of sudden severe exacerbation
- for patients who cannot be controlled on 800mcg/day of ICS  Prior intubation for asthma
 Sodium cromoglycate and nedocromil  Two or more hospitalisations for asthma in the past year
- blocks chloride channel  prevent mast cell activation  Three or more emergency care visits for asthma in the past year
 Inhaled corticosteroids (ICS)  Hospitalisation or an emergency care visit for asthma within the past
- any form of persistent asthma (needing relief meds at least once a month
week) requires steroid inhaler treatment  Use of >2 c anisters per month of inhaled short-acting B2-agonist
- beclometasone, budesonide, fluticasone, triamcinolone  Current use of systemic corticosteroids or recent withdrawal from
- most of the dose is swallowed or exhaled systemic corticosteroids
- adding a long acting beta-2 agonist is more effective than doubling the  Known difficulty perceiving airflow obstruction or its severity
dose of ICS  Comorbidity, as from cardiovascular diseases or chronic obstructive
- side-effects: oral candidiasis, hoarseness, subcapsular cataract,  pulmonary disease
avascular necrosis of the femoral head, osteoporosis, growth  Serious psychiatric disease or psychosocial problems
retardation  Low socioeconomic status
- step down treatment after the condition is under control  Illicit drug use
 Oral corticosteroids  ABG: PaCO2>6kPa, PaO2<8kPa, pH low and falling
- keep the dose as low as possible
- for those who cannot be controlled on ICS
 Leukotriene-receptor antagonists
- add on therapy
- good for aspirin-intolerant asthma

424
Severity of Asthma Attack

Feeble

Silent

<30%

425
Management of an acute attack

Indications for intubation


- progressive respiratory failure
- altered mental status
- hemodynamic instability

Supportive measures:
- oxygen

Monitoring:
- O2 saturation
- ABG
- PEFR
- Level of consciousness

426
Medicine (Respi) = Pulmonary embolism
Principles of Diagnosis
Identify risk factors and acquired conditions predisposing to DVT and PE
Enumerate differential diagnoses of DVT (leg swelling) and PE (collapse, chest pain, dyspnea or hemoptysis).
Clinically differentiate between the clinical syndromes of PE, i .e. pulmonary infarction, submassive PE,
massive PE and chronic PE.
Outline the diagnostic work-up of patients with suspected PE, including non-imaging and imaging methods,
and their limitations.
Principles of Management
Describe the management of PE including –
a) 1° strategies such as anticoagulants, thrombolysis and surgery.
Detailed knowledge of anticoagulants including contraindications, the types of drugs, route of administration,
indications and maintenance doses, lab monitoring, potential interactive actions to take in case of
overanticoagulation
b) Resuscitative and supportive means for shock, RVF, chest pain
Prevention
Describe the recommended antithrombotic or anticoagulation regimes and mechanical measures for
prophylaxis of DVT.
Know the anticoagulation regimes for secondary prevention of PE.
Describe briefly the role of catheter-based strategies, including inferior vena caval interruption for secondary
prevention of massive PE.
Pathophysiology
Describe the pathophysiology of pulmonary embolism, including the concept of ventricular interdependence.

Predisposing factors
 Prolonged bed rest, immobilisation
 Surgery, trauma, fractures
 Previous stroke or thromboembolism
 Congestive cardiac failure
 Disseminated cancer
 Pregnant women
 Antiphospholipid syndrome
 Drugs: OCP
 Smoking
 Genetic: Factor V Leiden mutation, thrombophilias

Pathogenesis
- Thrombi from deep veins of the leg (95% are from the popliteal vein or the veins above it)
- Clots break off and embolise to the lungs
- Large embolus obstructing the main pulmonary artery  increased pulmonary artery pressure due to
blockage of flow + vasospasm due to release of mediators/neurogenic mechanism  leads to
hypoxemia, acute cor pulmonale (when more than 60% of vasculature is obstructed) and death
- Small thrombi may be clinically silent cos they are rapidly removed by fibrinolytic activity, and the
bronchial circulation maintains the viability of the affected parenchyma till this is achieved
- Smaller thrombi continue traveling distally and are more likely to produce pleuritic chest pain, by
initiating an inflammatory response adjacent to the parietal pleura
- Pulmonary infarction may occur rarely (less likely due to dual blood supply)
- Multiple small emboli may lead to pulmonary hypertension  decreased cardiac output
Clinical syndromes of PE
Massive pulmonary embolism
- PE associated with a systolic blood pressure <90 mmHg or a drop in systolic blood pressure of ≥ 40
mmHg from baseline for a period >15 minutes, which is not otherwise explained by hypovolemia,
sepsis, or a new arrhythmia
- a catastrophic entity that often results in acute right ventricular failure and death

Submassive pulmonary embolism


- All PE not meeting the definition of massive PE are considered submassive PE.

Pulmonary infarction
- Infarction only occurs if bronchial circulation is impaired
- The more peripheral the embolic occlusion, the more likely is infarction

Chronic pulmonary embolism


- Occurs when acute PE does not resolve, lasts for years

Clinical features
History
 Acute breathlessness, pleuritic chest pain, hemoptysis, dizziness, syncope
 Risk factors

Physical examination
 Tachypnea, pyrexia, tachycardia, hypotension
 Cyanosis
 Raised JVP, loud P2, 4th heart sound
 Pleural rub or effusion
 Signs of DVT
 Recent surgical scar

Clinical scoring
 Wells score
Previous DVT/PE 1.5
Immobilization or surgery in previous 1 month 1.5 0-2 Low probability
Malignancy 1
3-6 Mod probability
Clinical symptoms of DVT 3
Hemoptysis 1
> 6 High probability
Heart rate >100 1.5
Other diagnosis less likely than PE 3

 Geneva score
Age > 65 1
Previous DVT or PE 3 0-3 Low probability
Surgery (under GA) or lower limb fracture within 1 month 2
Active malignant condition or cured in < 1 yr 2 4-10 Mod probability
Unilateral lower limb pain 3 > 10 High probability
Pain on lower limb deep venous palpation and unilateral edema 4
Hemoptysis 2
Heart rate 75-94 bpm 3
Heart rate ≥ 95 bpm 5
Investigations
Nonspecific lab findings
 ESR raised
 BNP raised
 Trop T raised
Specific investigations
 CXR
- Normal
- Atelectasis
- Oligemia of affected segment
- Dilated pulmonary artery
- Small effusion
- Wedge shaped opacities
 ECG
- S1Q3T3 pattern, right ventricular strain, new incomplete RBBB (classical but rare)
- Atrial arrhythmias
- T wave inversion, ST changes
 D dimer: degradation product of cross-linked fibrin
- Sensitivity 95%
 V/Q perfusion scan: look for perfusion defects without corresponding ventilation defects
 CT pulmonary angiography (gold standard)/ “spiral CT”
 Lower limb Doppler ultrasound

Differential diagnoses
PE
 Acute coronary syndrome
 COPD
 Myocarditis

DVT
 Cellulitis
 Superficial thrombophlebitis
 Algorithm for diagnosis of PE

Suspicion of PE

Perform Wells/Geneva
score

Low/intermediat  High probability


e probability

Start tx
D Dimer assay

CTPA
D Dimer D Dimer
negative positive
CTPA CTPA
negative positive
No tx CTPA

Lower limb Continue tx


CTPA CTPA DVT scan
negative positive

No tx Start tx Scan Scan


negative positive

Stop tx Continue tx
Management
 Assess ABCs
 Stabilise the patient
- supportive measures eg. supplemental O2
- cautiously administer intravenous fluids (avoid ppting right heart failure)
- vasopressor therapy
 Anticoagulation
- Reduces mortality by preventing recurrent PE
- In those with high probability of PE, start anticoagulation before investigations
- Greatest efficacy if therapeutic heparin levels are initiated within 24 hours
- In hemodynamically stable patients with PE, SC LMWH is preferred
- Patients in whom anticoagulation was initiated during the resuscitative period should remain
anticoagulated during the diagnostic evaluation. Anticoagulation should be discontinued of PE
is excluded
- Long-term anticoagulation with warfarin is indicated if PE is confirmed
 Inferior vena caval filter placement should be considered if anticoagulation is contraindicated (patient
has active bleeding), fails, or causes complications (eg, severe bleeding)
- results in less recurrence of PE
- but recurrent DVT was more common among patients who received an IVC filter
 Thrombolysis should be considered once PE is confirmed
- Accelerates the lysis of acute pulmonary emboli
- Increased likelihood of major hemorrhage
- If thrombolysis is chosen, anticoagulation should be temporarily discontinued then resumed
- No mortality benefit, but shown to improve RV function
- Persistent hypotension due to massive PE is a widely accepted indication for thrombolysis
 Embolectomy
- Removal of embolus using catheters or surgically
- When thrombolysis either fails or is contraindicated
- Catheter emboleeectomy: injecting pressurized saline through the catheter's distal tip, which
macerates the embolus. The saline and fragments of clot are then sucked back into an exhaust
lumen of the catheter for disposal
 Preventive management: elastic stockings, leg exercises, ambulation, long term anticoagulation with
warfarin
 Anticoagulation regimes
DRUG THERAPY
 LMWH (fraxiparin, enoxaparin)
- results in lower mortality, fewer recurrent thrombotic events, and less major bleeding than UFH
- greater bioavailability, once or twice daily administration, fixed dosing (ie, dose does not require
adjustment), no required laboratory monitoring, and decreased likelihood of thrombocytopenia
- exception: patients who are pregnant or have severe renal failure require anti-Xa assay monitoring
after administration of SC LMWH
 Unfractionated heparin (continuous iv infusion)
- preferred in patients with persistent hypotension due to massive PE; severe renal failure (aPTT
monitoring is easier than anti-Xa assay)
- target 1.5-2.3 x the control aPTT
- protamine sulphate is the antidote for heparin (cannot fully reverse LMWH’s anti-Xa effects)
 Fondaparinux (new)
- synthetic heparin pentasaccharides that catalyse factor Xa inactivation by antithrombin, without
inhibiting thrombin
- may be a viable alternative to unfractionated heparin
 Warfarin
- risk factors for bleeding: age >75, concurrent aspirin therapy, hypertension, CVA, renal insufficiency,
heart disease, cancer
- Vit K and FFP are antidotes for warfarin

DURATION
First episode of Recurrent episode of
PE/DVT PE/DVT

Reversible risk No identifiable risk Irreversible risk


factor eg. recent factor factor eg. ptn C
surgery deficiency, APS

Warfarin
3-6 months Warfarin Indefinite therapy
6-12 months,
consider indefinite
therapy

- Treatment duration among patients with a first episode of PE or deep vein thrombosis (DVT) is
determined by whether a risk factor can be identified and, if so, whether the risk factor is reversible.
- Reversible risk factor eg, immobilization, surgery, trauma: warfarin for 3-6 months
- No identifiable risk factors ie, idiopathic PE or DVT: at least 6 to 12 months, consider indefinite
anticoagulation
- Irreversible risk factor eg. protein C deficiency, protein S deficiency, factor V Leiden gene mutation: at
least 6 to 12 months, consider indefinite anticoagulation
- Indefinite therapy should be administered to patients with recurrent PE or DVT.

Prognosis
 30% chance of developing a second embolus
 Mortality rate of approximately 30 % without treatment, due mainly to recurrent embolism
 Accurate diagnosis followed by effective therapy with anticoagulants decreases the mortality rate to 2
to 8 %

You might also like